Sei sulla pagina 1di 258

(2nd Edition- March 2017)

Crux of Indian Economy


For IAS Prelims 2017
By CA Atul Mittal

Coverage of Book:
Current Affairs updated upto Feb 2017
Budget 2017
Economic Survey 2017
Crux of NCERTs,
Crux of NIOS,
Past years UPSC examination questions viz:
UPSC- Prelims 1991 to 2016
UPSC- CDS 2009 to 2017
UPSC- ES 2009 to 2017
UPSC- CAPF 2009 to 2016
UPSC- CISF 2009 to 2015
UPSC- NDA 2009 to 2016
UPSC- SCRA 2009 to 2015

For learning economy or any query/feedback Whatsapp


the Author at 7023213423

For Economy updates follow http://iasselfstudy.com/


Whatsapp No 7023213423 http://iasselfstudy.com/ 1

Contents

Chapter Name Page


No No.
1 RBI & Monetary Policy 2
2 Banking 17
3 Finance & Investments 41
4 Taxation 51
5 National Income, Budget & Economic Survey 60
6 Money Market & Stock Market 85
7 Micro Economy 97
8 International Trade & Agreements 104
9 International Institution 123
10 Govt schemes related to Economy 182
11 Miscellaneous Topics related to Economy 199
12 UPSC Previous Years Questions related to 239
Economy not covered in earlier Chapters

Sign given with the topics indicates the following:

* Important
** Very Important
*** Very Very Important
Whatsapp No 7023213423 http://iasselfstudy.com/ 2

Chapter: 1
RBI & Monetary Policy

Establishment, Nationalization, Accounting year *


Establishment April 1, 1935 in Calcutta but permanently moved to Mumbai
in 1937

Nationalization on 1st January, 1949 on the basis of the Reserve Bank of


India (Transfer to Public Ownership) Act, 1948

Act which governs the RBI Reserve Bank of India Act, 1934
functions

Act which governs the Banking Regulation Act, 1949


Financial sector

Financial/Accounting year of July to June


RBI

Financial/Accounting year of April to March


Companies

IAS Prelims 1998


The accounting year of the Reserve Bank of India is
(a) April – March (b) July – June (c) October – September (d) January – December

ES-2012
Banking in India is controlled by :
(a) RBI (b) Union Finance Commission (c) Union Ministry of Finance (d) Union Ministry of Commerce

Objectives, Functions and Organizational structure *


Basic objectives to regulate the issue of Bank Notes

to keeping of reserves with a view to securing monetary stability in India

to operate the currency and credit system of the country

Functions Formulation and execution of monetary policy,

Foreign exchange and reserves management,

Government debt management, (CG & SG)


Whatsapp No 7023213423 http://iasselfstudy.com/ 3

Financial regulation and supervision,

Acting as banker to the banks and to the Government (CG & SG)

Central Board of Directors

Governor

Deputy Governors
RBI
Organizational Executive Directors
structure
Principal Chief General Manager

Chief General Managers

General Managers

Primary Central Board


authority and
responsibility for
the oversight of
the Reserve
Bank is

RBI chief The Governor is the Reserve Bank’s chief executive. The Governor
executive supervises and directs the affairs and business of the RBI

CAPF-2009
Which one of the following is not a function of Reserve Bank of India ?
(a) Banker to the government
(b) Keeping foreign exchange reserve
(c) Issuing of one rupee coin and note
(d) Regulating credit in the country

CISF-LDC-2013
With reference to Reserve Bank of India which of the following statements is/are correct ?
I. It acts as a Banker's Bank
2. It issues one rupee coins
3. It acts as a lender of the last resort to sick industrial units
Select the correct answer using the code given below :
(a) I only
(b) 2 and 3 only
(c) 3 only
(d) I, 2 and 3

IAS Prelims 2004


Consider the following statements:
1. Reserve Bank of India was nationalized on 26 January, 1950.
2. The borrowing programme of the Government of India is handled by the Department of Expenditure,
Ministry of Finance.
Which of the statements given above is/are correct?
(a) 1 only
(b) 2 only
(c) Both 1 and 2
(d) Neither 1 nor 2
Whatsapp No 7023213423 http://iasselfstudy.com/ 4

IAS Prelims 2001


Consider the following statements regarding Reserve Bank of India :
I. It is a banker to the Central Government.
II. It formulates and administers monetary policy.
III. It acts as an agent of the Government in respect of India’s membership of IMF.
IV. It handles the borrowing programme of Government of India.
Which of these statements are correct ?
(a) I and II
(b) II, III and IV
(c) I, II, III and IV
(d) III and IV

IAS Prelims 1991


Which one of the following functions of the Reserve Bank of India is not performed by the other banks ?
(a) Making advances to agriculturists
(b) Dealing in foreign exchange
(c) Industrial Refinancing
(d) Acting as the banker to the Government

IAS Prelims 2012


The Reserve Bank of India (RBI) acts as a bankers’ bank. This would imply which of the following?
1 Other bank retains their deposits with the RBI.
2 The RBI lends funds to the commercial banks in times of need.
3 The RBI advises the commercial banks on monetary matters.
select the correct answer using the codes given below:
(a )2 and 3 only (b )1 and 2 only (c )1 and 3 only (d )1, 2 and 3

IAS Prelims 2013


The Reserve Bank of India regulates the commercial banks in matters of
1 liquidity of assets
2 branch expansion
3 merger of banks
4 winding-up of banks
Select the correct answer using the codes given below.
a. 1 and 4 only
b. 2, 3 and 4 only
c. 1, 2 and 3 only
d. 1, 2, 3 and 4

Monetary policy ***


Responsibility of RBI is vested with the responsibility of conducting monetary policy.
conducting This responsibility is explicitly mandated under the Reserve Bank of
monetary policy India Act, 1934

Primary objective Maintaining price stability while keeping in mind the objective of
growth.

Price stability is a necessary precondition to sustainable growth.

Announcement Bi-monthly
Whatsapp No 7023213423 http://iasselfstudy.com/ 5

Inflation target Inflation target is set by the Govt. in consultation with RBI, once in
every five years.

Inflation target– is measured by the consumer price index-combined


(CPI-C)

Inflation target is 4% (+/-) 2% for the period from August 5, 2016 to


March 31, 2021

6 members MPC will be constituted by the Central Government


Monetary policy
Following is the member of committee:
committee (MPC)
RBI Governor- Chairperson,

Deputy Governor of RBI, in charge of Monetary Policy

One officer of the Reserve Bank of India to be nominated by the


Central Board

Other three members appointed by the CG: (will hold office for a
period of four years)

Shri Chetan Ghate, Professor, Indian Statistical Institute (ISI)

Professor Pami Dua, Director, Delhi School of Economics; and

Dr. Ravindra H. Dholakia, Professor, IIM, Ahmedabad

MPC function The Committee will determine the policy interest rate required to
achieve the inflation target.

Monetary policy The MPC will meet at least four times in a year.
making
The quorum for the meeting of the MPC is four members.

Each member of the MPC has one vote, and in the event of an
equality of votes, the Governor has a second or casting vote.

The questions which come up before the MPC will be decided by


majority of votes by the members present in voting.

The resolution adopted by the MPC will be published after conclusion


of every meeting.

On the 14th day, the minutes of the proceedings of the MPC will be
published

Monetary Policy Once in every six months, the bank will publish a document called the
Report Monetary Policy Report which will explain:

Source of inflation; and

Forecast of inflation for 6-18 months ahead.


Whatsapp No 7023213423 http://iasselfstudy.com/ 6

CDS 2017
Which one of the following indices is now used by the Reserve Bank of India to measure the rate of
inflation in India?
(a) NASDAQ Index
(b) BSE Index
(c) Consumer Price Index
(d) Wholesale Price Index

Instruments of Monetary policy ***


Direct Cash Reserve CRR is a fixed percentage of demand and time liabilities
Instruments Ratio (CRR) (i.e. bank deposits) which banks must maintain as average
daily balance with the RBI.

Example:

Suppose CRR rate is 3% and Bank has deposits of Rs. 100


crores then it has to deposit Rs. 3 crores with RBI.

Statutory SLR is a fixed percentage of bank deposits which banks


Liquidity Ratio must maintain in safe and liquid assets, such as,
(SLR) unencumbered government securities, cash and gold.

Changes in SLR often influence the availability of resources


in the banking system for lending to the private sector.

Higher the CRR and SLR, lower will be the liquidity in the
system as Banks will have lesser money for providing loans.

Example:

Suppose CRR and SLR rate is 3% and 27% respectively.


Bank deposits is Rs. 100 crores then bank can sanction
loans upto 70 crores.

Indirect Repo Rate The (fixed) interest rate at which the Reserve Bank
Instruments (also called provides short-term (overnight) liquidity to banks
Policy rate)
Reduction in Repo rate helps the banks to get money at a
cheaper rate and increase in Repo rate discourages the
banks.

Reverse Repo The (fixed) interest rate (currently 50 bps below the repo
rate) at which the Reserve Bank absorbs short-term
liquidity, generally on an overnight basis, from banks

When RBI increases the reverse repo rate then Banks are
attracted to deposit with RBI for higher return.
Whatsapp No 7023213423 http://iasselfstudy.com/ 7

Bank Rate It is the rate at which the Reserve Bank is ready to buy or
rediscount bills of exchange or other commercial papers.

i.e. at which the Reserve Bank lends to banks for medium-


term purposes).

Increase in the bank rate is the symbol of tightening of RBI


monetary policy. (i.e. Dearer Monetary Policy)

When RBI wants to increase liquidity in the market, it


reduces bank rate. When RBI wants to decrease liquidity in
the market, it increases bank rate.

Open Market These include both outright purchase/sale of government


Operations securities for injection/absorption of durable liquidity,
(OMO) respectively.

In OMO, RBI purchases (or sells) government securities to


the general public to control the supply of money in the
economy.

In case of inflation, it sells the securities to control the flow


of money.

Marginal A facility under which banks can borrow additional amount


Standing of overnight money from the Reserve Bank by dipping into
Facility (MSF) their Statutory Liquidity Ratio (SLR) portfolio up to a limit
(currently 2% of their deposits) at a penal rate of interest,
currently 50 basis points above the repo rate.

This provides a safety valve against unanticipated liquidity


shocks to the banking system.

Market The scheme envisages issue of cash management bills,


Stabilization treasury bills and/or dated securities to absorb excess
Scheme liquidity arising from large capital inflows.
(MSS)
Thus RBI sterilizes the economy against adverse external
shocks.

This operation of RBI is known as sterilization.


Whatsapp No 7023213423 http://iasselfstudy.com/ 8

IF Increase/Dec Liquidity Inflation Monetary policy


rease in Market

CRR/SLR/Repo/Reverse Increases Decreases Decrease Dearer/Tight


Repo/Bank Rate s Monetary Policy

CRR/SLR/Repo/Reverse Decreases Increases Increases Accommodative/Eas


Repo/Bank Rate y monetary policy

IAS Prelims 2014


The terms ‘Marginal Standing Facility Rate’ and ‘Net Demand and Time Liabilities’, sometimes appearing
in news, are used in relation to
A. banking operations
B. communication networking
C. military strategies
D. supply and demand of agricultural products

ES 2017
Consider the following statements regarding Repo rate:
I. It is the rate at which RBI lends money to Commercial Banks generally against Government
Securities
2. It is the rate at which RBI borrows money from Commercial Banks generally against Government
Securities
3. it is the rate at which Commercial Banks keep Deposits with RBI
Which of the above statements is/are correct?
(a) 1 only
(b) 2 only
(c) 3 only
(d) I, 2 and 3

CDS-2010
Consider the following statements with regard to Statutory Liquidity Ratio (SLR) :
1. To meet SLR, commercial banks can use cash only.
2. SLR is maintained by the banks with themselves.
3. SLR restricts the banks' leverage in pumping more money into the economy.
Which of the statements given above is/ are correct?
(a) 1, 2 and 3
(b) 1 and 3 only
(c) 2 and 3 only
(d) 2 only

CAPF-2013
Consider the following statements :
1. Repo rate is the interest rate at which RBI lends to commercial banks for short period.
2. Reverse repo rate is the interest rate which RBI pays to commercial banks on short term
deposits.
3. Gap between repo rate and reverse repo rate has been declining in India in the recent
past.
Which of the statements given above is/are not correct?
(a) 1
(b) 2 only
(c) 3 only
(d) 2 and 3
Whatsapp No 7023213423 http://iasselfstudy.com/ 9

CISF-LDC-2013
What is "bank rate", quite often in the news?
(a) It is the rate at which RBI lends to Commercial Banks
(b) It is the rate at which commercial banks lend to general public
(c) It is the rate at which World Bank lends to governments
(d) It is the rate at which commercial banks lend to corporate houses

CISF-LDC-2014
The term “Repo Rate” sometimes found in news is related to :
(a)Banking System
(b) Demographic trends
© Foreign trade
(d) Human fertility

IAS Prelims 2010


Which of the following terms indicates a mechanism used by commercial banks for providing credit to
the government?
(a) Cash Credit Ratio
(b) Debt Service Obligation
(c) Liquidity Adjustment Facility
(d) Statutory Liquidity Ratio

IAS Prelims 2010


When the Reserve Bank of India announces an increase of the Cash Reserve Ratio, what does it mean?
(a)Commercial banks will have less money to lend
(b)Reserve Bank of lndia will have less money to lend
© Union Government will have less money to lend
(d) The commercial banks will have more money to lend

IAS Prelims 1992


‘Interest Rate Policy’ is a component of
(a) Fiscal Policy
(b) Monetary Policy
(c) Trade Policy
(d) Direct Control

IAS Prelims 1993


Variable reserve rates and Open Market Operations are instruments of
(a) Fiscal Policy
(b) Monetary Policy
(c) Budgetary Policy
(d) Trade Policy

IAS Prelims 1995


Which one of the following is not an instrument of selective credit control in India ?
(a) Regulation of consumer credit
(b) Rationing of credit
(c) Margin requirements
(d) Variable cost reserve ratios

IAS Prelims 1995


Bank Rate implies the rate of interest
(a) paid by the Reserve Bank of India on the Deposits of Commercial Banks
(b) charged by Banks on loans and advances
Whatsapp No 7023213423 http://iasselfstudy.com/ 10

(c) payable on Bonds


(d) at which the Reserve Bank of India discounts the Bills of Exchange

IAS Prelims 1998


The banks are required to maintain a certain ratio between their cash in hand and total assets. This is
called
(a) SBR (Statutory Bank Ratio)
(b) SLR (Statutory Liquid Ratio)
(c) CBR (Central Bank Reserve)
(d) CLR (Central Liquid Reserve)

IAS Prelims 2015


With reference to India economy, consider the following:
1. Bank rate
2. Open market operations
3. Public debt
4. Public revenue
Which of the above is/are component/components of Monetary Policy?
a) 1 only
b) 2, 3 and 4
c) 1 and 2
d) 1, 3 and 4

IAS Prelims 2015


When the Reserve Bank of India reduces the Statutory Liquidity by 50 basis points, which of the
following is likely to happen?
a) India’s GDP growth rate increases drastically
b) Foreign Institutional Investors may bring more capital into our country
c) Scheduled Commercial Banks may cut their lending rates
d) It may drastically reduce the liquidity to the banking system.

IAS Prelims 2014


In the context of Indian economy; which of the following is/are the purpose/purposes of ‘Statutory
Reserve Requirements’?
1. To enable the Central Bank to control the amount of advances the banks can create
2. To make the people’s deposits with banks safe and liquid
3. To prevent the commercial banks from making excessive profits
4. To force the banks to have sufficient vault cash to meet their day-to-day requirement

Select the correct answer using the code given below.

A. 1 only
B. 1 and 2 only
C. 2 and 3 only
D. 1, 2, 3 and 4

IAS Prelims 2013


An increase in the Bank Rate generally indicates that the
a. Market rate of interest is likely to fall
b. Central Bank is no longer making loans to commercial banks
c. Central Bank is following an easy money policy
d. Central Bank is following a tight money policy
Whatsapp No 7023213423 http://iasselfstudy.com/ 11

IAS Prelims 2013


In the context of Indian economy, Open Market Operations refers to
a. borrowing by scheduled banks from the RBI
b. lending by commercial banks to industry and trade
c. purchase and sale of government securities by the RBI
d. None of the above

IAS Prelims 2011


The lowering of bank rate by the reserve bank of India leads to ?
(a) More liquidity in the market.
(b) Less liquidity in the market.
(c) No change in the liquidity in the market.
(d) Mobilization of more deposits by commercial banks.

Marginal Cost of Funds based Lending Rate (MCLR) *

Background Earlier, loans were priced at a spread over the Base Rate.

Base Rate is the minimum rate of interest for all loans and spread
is the margin of bank based on risk associated with loans.

Bank were using different cost methodology to compute base rate.

When RBI cut interest rates many times, Banks were reluctant to
pass on these rate cuts to borrowers giving excuse that they have
old deposits for which the interest rate remains high.

To counter this, RBI has introduced MCLR so that banks link their
lending rates to marginal funding costs (i.e cost of fresh or
incremental borrowings from public).

Effective from As per RBI, All rupee loans sanctioned and credit limits renewed
w.e.f. April 1, 2016 will be priced with reference to the Marginal
Cost of Funds based Lending Rate (MCLR) .

It will be a tenor linked internal benchmark rate for loans

Purpose to improve transparency in the methodology followed by banks for


determining interest rates on loans

Actual lending rates Actual lending rates will be determined by adding the components
of spread to the MCLR.

Monthly review Banks will review and publish their MCLR of different maturities
every month on a pre-announced date.
Whatsapp No 7023213423 http://iasselfstudy.com/ 12

IAS Prelims 2016


What is/are the purpose/purposes of the `Marginal Cost of Funds based Lending Rate (MCLR)’
announced by RBI?
1. These guidelines help improve the transparency in the methodology followed by banks for
determining the interest rates on advances.
2. These guidelines help ensure availability of bank credit at interest rates which are fair to the
borrowers as well as the banks.
Select the correct answer using the code given below.
(a) 1 only
(b) 2 only
(c) Both 1 and 2
(d) Neither 1 nor 2

Basis Point

100 basis point means 1%. It is used for measuring change in interest rate.

Subsidiaries of RBI

Fully-owned National Housing Bank (NHB),


Subsidiaries
Deposit Insurance and Credit Guarantee Corporation (DICGC),

Bharatiya Reserve Bank Note Mudran Private Limited (BRBNMPL)

Others RBI also has a majority stake in the National Bank for Agriculture and Rural
Development (NABARD).

IAS Prelims 2007


The National Housing Bank was set up in India as a wholly owned subsidiary of which one of the following
?
(a) State Bank of India
(b) Reserve Bank of India
(c) ICICI Bank
(d) life Insurance Corporation of India

IAS Prelims 2004


Consider the following statements:
1. The National Housing Bank, the apex institution of housing finance in India, was set up as a wholly
owned subsidiary of the Reserve Bank of India.
2. The Small Industries Development Bank of India was established as a wholly owned subsidiary of the
Industrial Development Bank of India.
Which of the statements given above is/ are correct?
(a) 1 only
(b) 2 only
(c) Both 1 and 2
(d) Neither 1 nor 2
Whatsapp No 7023213423 http://iasselfstudy.com/ 13

Government Security
Definition A Government security is a tradable instrument issued by the
Central Government or the State Governments.

It acknowledges the Government’s debt obligation.

Short term securities maturity period of less than one year (e.g. treasury bills)

Long term securities maturity period of one year or more (e.g. Government bonds or
dated securities with original).

Issue by CG Central Government issues both, treasury bills and bonds or dated
securities

Issue by SG State Governments issue only bonds or dated securities, which are
called the State Development Loans (SDLs).

Gilt-edged securities Government securities carry practically no risk of default and,


hence, are called risk-free gilt-edged securities

IAS Prelims 2000


Gilt edged market means
(a) bullion market
(b) market of Government securities
(c) market of guns
(d) market of pure metals

Cash Management of Government


Minimum balance by Central Government is required to maintain a minimum cash
CG balance with the Reserve Bank.

Currently, this amount is Rs.10 crore on a daily basis and Rs.100


crore on Fridays.

Minimum balance by All the State Governments are required to maintain a minimum
SG balance with the Reserve Bank, which varies from state to state
depending on the relative size of the state budget and economic
activity.

Ways and Means To tide over temporary mismatches in the cash flow of receipts
Advances/Overdraft and payments, the Reserve Bank provides Ways and Means
Advances/Overdraft to the State Governments
Whatsapp No 7023213423 http://iasselfstudy.com/ 14

Cash Management Bills


Cash Management Bills are issued to meet the temporary cash flow mismatches of the
Government.

Promissory note
is a signed document containing a written promise to pay a stated sum to a specified person
or the bearer at a specified date or on demand. For example Bank note.

Foreign exchange reserves ***


The foreign exchange reserves include (as per decreasing order)
- Foreign currency assets (FCA),
- Gold
- Reserve Tranche Position (RTP) in the IMF and
- Special Drawing Rights (SDRs)

IAS Prelims 2013


Which one of the following groups of items is included in India’s foreign-exchange reserves?

a. Foreign-currency assets, Special Drawing Rights (SDRs) and loans from foreign countries
b. Foreign-currency assets, gold holdings of the RBI and SDRs
c. Foreign-currency assets, loans from the World Bank and SDRs
d. Foreign-currency assets, gold holdings of the RBI and loans from the World Bank

Special Drawing Rights (SDRs) **


Created by IMF The SDR is an international reserve asset, created by the IMF in
1969 to supplement its member countries’ official reserves.

Value of the SDR The value of the SDR is based on a basket of five major freely
usable currencies—U.S. dollar, Euro, Chinese renminbi (RMB),
Japanese yen, and Pound sterling

Effective from October 1, 2016, the Chinese renminbi (also called


Yuan) was included in the SDR basket.

Freely usable A “freely usable” currency mean a currency that the IMF
currency determines

is widely used to make payments for international transactions, and

is widely traded in the principal exchange markets.


Whatsapp No 7023213423 http://iasselfstudy.com/ 15

IMF lending operations are, in practice, conducted in freely usable


currencies or SDRs.

Position of SDR The SDR is neither a currency, nor a claim on the IMF. Rather, it is
a potential claim on the freely usable currencies of IMF members.

SDRs can be exchanged for freely usable currencies.

Holders of SDRs can obtain these currencies in exchange for their


SDRs

CDS-2012
Special Drawing Rights (SDRs) relate to
1. the World Bank
2. the Reserve Bank of India
3. the World Trade Organization
4. the International Monetary Fund

IAS Prelims-2016
Recently, which one of the following currencies has been proposed to be added to the basket of IMF’s
SDR?
(a) Ruble
(b) Rand
(c) Indian Rupee
(d) Renminbi

IAS PRELIMS 2010


Which of the following is/are treated as artificial currency ?
(a)ADR
(b)GDR
(c) SDR
(d) Both ADR and SDR

IAS PRELIMS 2007


Special Drawing Rights (SDR), the currency of IMF, is in the form of
(a) Paper currency
(b) Book-keeping entry only
(c) Gold
(d) PPP-Dollar
Whatsapp No 7023213423 http://iasselfstudy.com/ 16

RBI publishes “Payment and Settlement Systems in India: Vision-2018” *


Aims The Vision-2018 aims at building best of class payment and settlement
systems for a ‘less-cash’ India

5 Cs The broad contours of Vision-2018 revolve around 5 Cs – coverage,


convenience, confidence, convergence and cost

4 strategic To achieve these, Vision-2018 will focus on four strategic initiatives such as
initiatives responsive regulation, robust infrastructure, effective supervision and
customer centricity.

RBI Publications
1. Financial stability report- Half yearly
2. Monetary policy report- Half yearly
3. Report on foreign exchange reserves- Half yearly
4. Bi-monthly Policy Statement
Whatsapp No 7023213423 http://iasselfstudy.com/ 17

Chapter -2

Banking

Banking

Calcutta presidency bank was the First bank in India established by East India company in
1806.

Function of commercial banks


Primary Accepting deposit and Providing loans

Secondary Collection and payment of various items e.g. Cheques, Bills

Purchase and sell of securities

Remittance of money

Purchase and sell of foreign exchange

Acting as executors and trustees of wills

Underwriting of shares

Lockers facility

Travelers’ cheque and letter of credit

CDS-2013
The main functioning of the banking system is to :
(a) accept deposits and provide credit
(b) accept deposits and subsidies
(c) provide credit and subsidies
(d) accept deposits, provide credit and subsidies

IAS Prelims 2010


Consider the following statements :
The functions of commercial banks in India include
1. Purchase and sale of shares and securities on behalf of customers.
2. Acting as executors and trustees of wills.
Which of the statements given' above is/are correct?
(a.) 1 only
(b.) 2 only
(c.) Both 1 and 2
(d.) Neither 1 nor 2
Whatsapp No 7023213423 http://iasselfstudy.com/ 18

Deposits in Bank
Demand deposits Saving account deposit and current account deposits are called
demand deposits as depositor can withdraw money as and when
required.

Term deposits Recurring deposit and fixed deposits are called Term deposits as
depositor can withdraw money after a certain period

Protection of Small Depositors *


DICGC The Reserve Bank has set up Deposit Insurance and Credit Guarantee
Corporation (DICGC) to protect the interest of small depositors, in case of
bank failure.

Coverage DICGC provides insurance cover to all bank depositors upto Rs.1 lakh per
depositor per bank

Liquidity order
Currency
Demand deposits in Banks
Savings deposits in Banks
Term deposits in Banks

IAS Prelims 2013


Consider the following liquid assets:
1.Demand deposits with the banks
2.Time deposits with the banks
3.Savings deposits with the banks
4.Currency
The correct sequence of these assets in decreasing order of liquidity is
a. 1-4-3-2
b. 4-3-2-1
c. 2-3-1-4
d. 4-1-3-2
Whatsapp No 7023213423 http://iasselfstudy.com/ 19

Non-performing Assets of Banks *


Classification In order to maintain the quality of loans and advances, Reserve Bank
requires banks to classify their loans as performing and non-performing
assets (NPA)

NPA An asset becomes non performing when it ceases to generate income for
the bank i.e. when dues are not paid within 90 days.

NPAs has been growing in public sector banks over the years

NPA relation NPAs growth is inversely related to the GDP growth. Decline in GDP
with GDP growth leads to rise in NPAs growth

Priority Sector Lending **


Definition Priority sector refers to those sectors of the economy which may not get
timely and adequate credit in the absence of this special dispensation

Different Priority Sector includes the following categories:


categories
(i) Agriculture (e.g. Farm credit, Agriculture infrastructure and Ancillary
under priority
activities.)
sector
(ii) Micro, Small and Medium Enterprises
(iii) Export Credit
(iv) Education
(v) Housing
(vi) Social Infrastructure
(vii) Renewable Energy
(viii) Others (e.g. Weaker sections)

Lending target Banks are required to lend 40% of their loan to priority sector

Priority Sector PSLCs are a mechanism to enable banks to achieve the priority sector
Lending lending target by purchase of these instruments in the event of shortfall.
Certificates
This also incentivizes surplus banks as it allows them to sell their excess
(PSLCs)
achievement over targets thereby enhancing lending to the categories
under priority sector.

IAS Prelims 2013


Priority Sector Lending by banks in India constitutes the lending to
a. agriculture
b. micro and small enterprises
c. weaker sections
d. All of the above
Whatsapp No 7023213423 http://iasselfstudy.com/ 20

Cash credit facilities


Purpose Cash Credit facility is granted to the customers to bridge working capital gap

The working capital funds are generally required for purchase of raw materials,
storage, Salary & Wages etc.

Security Cash Credit (CC) is granted against hypothecation of stock such as raw
materials, work-in-process, finished goods and stock-in-trade, including stores
and spares

Separate A separate cash credit account is opened with a bank to avail cash credit
account facility.

Know Your Customer guidelines *


About KYC It is a process by which banks obtain information about the identity
and address of the customers

Benefit This process helps to ensure that banks’ services are not misused

When KYC required The KYC procedure is to be completed by the banks while opening a
bank account.

Banks are also required to periodically update their customers’ KYC


details.

Documents required The Government of India has notified six documents as ‘Officially
Valid Documents’ (OVDs) for the purpose of producing proof of
identity. These six documents are Passport, Driving Licence, Voters’
Identity Card, PAN Card, Aadhaar Card issued by UIDAI and NREGA
Job Card.

If these documents also contain your address details, then it would


also be accepted as ‘proof of address’.

Bank Overdraft facility


Purpose to manage short-term cash flow problems

Limit An overdraft limit is determined based on the credit worthiness of a


borrower and the availability of collateral or a guarantee

How it works Bank permits an account holder to use or withdraw more than they
have in their account
Whatsapp No 7023213423 http://iasselfstudy.com/ 21

Land development banks *

They provide long term credit facilities to the farmers for agriculture development project

Regional Rural Banks *


Established through Regional Rural Banks (RRBs) were established in 1975 under the
provisions of the Ordinance promulgated on the 26th September
1975 and followed by Regional Rural Banks Act, 1976

Objective to develop the rural economy and to create a supplementary channel


to the 'Cooperative Credit Structure' with a view to enlarge
institutional credit for the rural and agriculture sector

Sponsored by The Government of India, the concerned State Government and the
bank, which had sponsored the RRB contributed to the share capital
of RRBs in the proportion of 50%, 15% and 35%, respectively.

Area of operation The area of operation of the RRBs is limited to notified few districts in
a State.

Function The RRBs mobilise deposits primarily from rural/semi-urban areas


and provide loans and advances mostly to small and marginal
farmers, agricultural labourers , rural artisans and other segments of
priority sector.

CAPF-2011
Which of the following statements regarding Regional Rural Banks (RRB) in India is/are correct?
1. The basic aim of selling up the RRBs is to develop rural economy.
2. The area of RRBs is limited to a specific region comprising one or more districts.
3. RRBs are sponsored by Commercial Banks.
Select the correct answer using the code given below :
(a) 1 only
(b) 1 and 2 only
(c) 2 and 3 only
(d) 1, 2 and 3
Whatsapp No 7023213423 http://iasselfstudy.com/ 22

National bank for Agriculture and Rural development (NABARD) *


Establishment NABARD was established on 12 July 1982 under THE NATIONAL BANK
FOR AGRICULTURE AND RURAL DEVELOPMENT ACT, 1981

Apex institution It is a Apex institution for financing the Agriculture and Rural
development

Re-financing It also provides re-financing facility to regional rural banks, cooperative


facility banks, commercial banks etc for the promotion of activities in the rural
areas.

CDS-2013
Which bank is limited to the needs of agriculture and rural finance ?
(a) SBI (b) NABARD (c) IFC (d) RBI

Small industries development bank of India (SIDBI) *


Establishment SIDBI was established in April 1990 as a wholly owned subsidiary of
Industrial Development Bank of India (IDBI) under The Small
Industries Development Bank of India Act, 1989

Purpose It acts as the Principal Financial Institution for the Promotion,


Financing and Development of the Micro, Small and Medium
Enterprise (MSME) sector and for Co-ordination of the functions of the
institutions engaged in similar activities

Present Currently shares of SIDBI are held by 34 institutions/public sector


shareholding banks/insurance companies owned or controlled by the Central Govt
and Govt of India.

India Aspiration to boost the starts-up Fund-of-funds ecosystem in the country, SIDBI
Fund (IAF) launched India Aspiration Fund with an initial corpus of Rs.2,000 crore
Whatsapp No 7023213423 http://iasselfstudy.com/ 23

Fund of Funds for Start-ups (FFS) *


Established by Govt. of India

Operated and Small Industries Development Bank of India (SIDBI)


managed by

Genesis The FFS originates from the Start up India Action Plan, an initiative of
Department of Industrial Policy & Promotion (DIPP)

Corpus Fund size is Rs.I0,000 crore and would be built over the 14th and 15th
Finance Commission cycles till 2025.

Investment FFS) would not invest directly in Startups, but would participate in the
capital of Alternate Investment Funds (AIF) registered with Securities and
Exchange Board of India (SEBI) for investing in equity and equity linked
instruments of various Startups at early stage, seed stage and growth
stages

Micro Units Development & Refinance Agency Ltd (Mudra Bank) *


Set up by Government of India

Wholly owned MUDRA has been initially formed as a wholly owned subsidiary of
subsidiary Small Industries Development bank of India (SIDBI)

Goal funding the unfunded

Micro Finance Micro Finance is an economic development tool whose objective is to


provide income generating opportunities to the people at the bottom
of the pyramid (low-income groups).

Roles and MUDRA would be responsible for refinancing all Last Mile Financiers
responsibilities of such as Non-Banking Finance Companies, Societies, Trusts, Section 8
MUDRA Companies [formerly Section 25], Co-operative Societies, Small
Banks, Scheduled Commercial Banks and Regional Rural Banks which
are in the business of lending to micro/small business entities
engaged in manufacturing, trading and services activities.

MUDRA would also partner with State/Regional level financial


intermediaries to provide finance to Last Mile Financier of small/micro
business enterprises.

Pradhan Mantri MUDRA was given the responsibility of monitoring the Pradhan Mantri
Mudra Yojana Mudra Yojana (PMMY) by collecting the information on regular basis
Whatsapp No 7023213423 http://iasselfstudy.com/ 24

MUDRA loan MUDRA loan is available through Banks/NBFCs/MFIs for all kind of
manufacturing, trading and service sector activities.

Loans are categorised into Shishu, Kishor and Tarun.

Shishu : covering loans upto 50,000/-

Kishor : covering loans above 50,000/- and upto 5 lakh

Tarun : covering loans above 5 lakh to 10 lakh

Shishu’, ‘Kishor’ and ‘Tarun’ to signify the stage of growth /


development and funding needs of the beneficiary micro unit /
entrepreneur

CDS-2016
Shishu, Kishor and Tarun are the schemes of:
(a) Regional Rural Banks.
(b) Micro Units Development & Refinance Agency Ltd. (MUDRA).
(c) Small Industries Development Bank of India.
(d) Industrial Development Bank of lndia.

IAS Prelims 2011


Microfinance is the provision of financial services to people of low-income groups. This includes both
the consumers and the self-employed. The service/services rendered under micro- finance is/are :
1. Credit facilities.
2. Savings facilities.
3. Insurance facilities.
4. Fund transfer facilities.
Select the correct answer using the codes given below the lists ?
(a) 1 only.
(b) 1 and 4 only.
(c) 2 and 3 only.
(d) 1, 2 ,3 and 4.

IAS Prelims 2016


Pradhan Mantri MUDRA Yojana is aimed at
(a) bringing the small entrepreneurs into formal financial system
(b) providing loans to poor farmers for cultivating particular crops
(c) providing pensions to old and destitute persons
(d) funding the voluntary organizations involved in the promotion of skill development and
employment generation
Whatsapp No 7023213423 http://iasselfstudy.com/ 25

State Bank of India *


Establishment RBI acquired a controlling interest in the Imperial Bank of India pursuant
to the provisions of the State Bank of India Act 1955.

On 1 July 1955, the Imperial Bank of India became the State Bank of
India.

Associates 1. State Bank of Bikaner and Jaipur

2. State Bank of Hyderabad

3. State Bank of Mysore

4. State Bank of Patiala

5. State Bank of Travancore

Merger SBI to be merged with associates and Bharatiya Mahila Bank

Benefit of rationalization of resources,


merger
reduction of costs,

better profitability,

lower cost of funds leading to better rate of interests for public at large,

improved productivity and customer services.

Merger will also ensure that due to size and scale of economy, SBI will be
able to better handle ensuing competition from new Banks.

Export-Import Bank of India (EXIM Bank)


Establishment Established in 1982 under the Export-Import Bank of India Act 1981

Function It is a premier export finance institution of the country

HQ Mumbai
Whatsapp No 7023213423 http://iasselfstudy.com/ 26

Nationalized banks in India (19 Banks at present)


Ist nationalization 14 major commercial Banks nationalized on 19thJuly 1969.

1. Allahabad Bank

2. Bank of Baroda

3. Bank of India

4. Bank of Maharashtra

5. Canara Bank

6. Central Bank of India

7. Dena Bank

8. Indian Bank

9. Indian Overseas Bank

10. Punjab National Bank

11. Syndicate Bank

12. UCO Bank

13. Union Bank of India

14. United Bank of India

2nd nationalization 6 more commercial Banks nationalized in 1980.

1. Andhra Bank

2. Corporation Bank

3. New Bank of India*

4. Oriental Bank of Commerce

5. Punjab & Sindh Bank

6. Vijaya Bank

*Punjab National Bank acquired New Bank of India in 1993

IAS Prelims 2006


Which one of the following Indian banks is not a nationalized bank ?
(a) Corporation Bank
(b) Dena Bank
(c) Federal bank
(d) Vijaya Bank
Whatsapp No 7023213423 http://iasselfstudy.com/ 27

Non-Banking Financial Companies (NBFCs)


Definition A Non-Banking Financial Company (NBFC) is a company registered
under the Companies Act, 1956 engaged in the business of loans and
advances, acquisition of shares/stocks/bonds/debentures/securities
issued by Government or local authority or other marketable
securities of a like nature, leasing, hire-purchase, insurance business,
chit business but does not include

any institution whose principal business is that of agriculture activity,


industrial activity, purchase or sale of any goods (other than
securities) or providing any services and sale/purchase/construction
of immovable property.

A non-banking institution which is a company and has principal


business of receiving deposits under any scheme or arrangement in
one lump sum or in installments by way of contributions or in any
other manner, is also a non-banking financial company (Residuary
non-banking company).

NBFC different from NBFCs lend and make investments and hence their activities are akin
Banks to that of banks; however there are a few differences as given below:

i. NBFC cannot accept demand deposits;

ii. NBFCs do not form part of the payment and settlement system and
cannot issue cheques drawn on itself;

iii. Deposit insurance facility of Deposit Insurance and Credit


Guarantee Corporation is not available to depositors of NBFCs, unlike
in case of banks.

IAS Prelims 2010


With reference to the Non-banking Financial Companies (NBFCs) in India, consider the following
statements :
1. They cannot engage in the acquisition of securities issued by the government.
2 . They cannot accept demand deposits like Saving Account.
Which of the statements given above is/are correct?
(a) 1 only
(b) 2 only
(c) Both 1 and 2
(d)Neither 1 nor2

IAS Prelims 1994


The difference between a bank and a Non Banking Financial Institution (NBFI) is that
(a) a bank interacts directly with customers while an NBFI interacts with banks and governments
(b) a bank indulges in a number of activities relating to finance with a range of customers, while an NBFI
is mainly concerned with the term loan needs of large enterprises
(c) a bank deals with both internal and international customers while an NBFI is mainly concerned with
the finances of foreign companies
(d) a bank’s main interest is to help in business transactions and savings/ investment activities while an
NBFI’s main interest is in the stabilization of the currency
Whatsapp No 7023213423 http://iasselfstudy.com/ 28

The Anchor banks (Consolidation of public sector banks) *


Meaning Government will identify six-ten public sector banks named as ‘the anchor
banks’ that will take and drive the consolidation process among the public
sector banks.

Reason for NPA of Public sectors banks are rising.


consolidation
Competition from private and new Banks

So it is better to have strong banks rather than having numerically large


number of banks.

Opposition Bankers had opposed the idea of consolidation among public sector banks
by banks on the ground that the financial health of most of the banks is poor.

Hence, no bank is ready to absorb a weak bank.

Merger of Merger of the five associate banks of State Bank of India (SBI) and
SBI Bharatiya Mahila Bank will be seen as a test case for banking consolidation

The Kisan Credit Card (KCC) *


Purpose Kisan Credit Card Scheme aims at providing adequate and timely credit
support from the banking system under a single window to the farmers for
their cultivation & other needs.

Smart card The beneficiaries under the scheme will be issued with a Smart card/ Debit
card (Biometric smart card compatible for use in the ATMs/Hand held Swipe
Machines and capable of storing adequate information on farmers identity,
assets, land holdings and credit profile etc)

Monitoring In case of RRBs and Cooperative banks it is monitored by NABARD and


agency
In case of commercial banks it is monitored by RBI.

Electronic Clearing Service (ECS)


ECS-Debit ‘ECS-Debit’ facilitates payment of charges to utility services, such as,
electricity bill, telephone bill, insurance premium and loan installments,
directly by debit to the customer’s account with a bank.

ECS–Credit The ‘ECS–Credit’ enables companies to pay interest or dividend to a large


number of beneficiaries by direct credit of the amount to their bank accounts.
Whatsapp No 7023213423 http://iasselfstudy.com/ 29

The Real Time Gross Settlement (RTGS)


Meaning This is the fastest possible money transfer system. Settlement in “real
time” means transactions are settled as soon as they are processed.

“Gross settlement” means the transaction is settled on one to one basis


without bunching with any other transaction.

Transfer limit Minimum amount to be transferred Rs. 2 lakh. No upper ceiling

Meant for Large value transactions

National Electronics Funds Transfer System (NEFT)


Meaning NEFT operates in hourly batches. The settlement takes place with all
transactions received till the particular cut-off time.

Any transaction initiated after a designated settlement time would have to


wait till the next designated settlement time

Transfer limit There is no limit – either minimum or maximum – on the amount of funds
that could be transferred using NEFT

Core Banking Solution (CBS)


Core Banking Solution (CBS) is networking of branches, which enables Customers to operate
their accounts, and avail banking services from any CBS branch of the Bank, regardless of
where he maintains his account.

The customer is no more the customer of a Branch. He becomes the Bank’s Customer.

IAS Prelims 2016


The term ‘Core Banking Solutions’ is sometimes seen in the news. Which of the following statements
best describes/describe this term?
1. It is a networking of a bank’s branches which enables customers to operate their accounts from any
branch of the bank on its network regardless of where they open their accounts.
2. It is an effort to increase RBI’s control over commercial banks through computerization.
3. It is a detailed procedure by which a bank with huge non-performing assets is taken over by another
bank.
Select the correct answer using the code given below.
(a) 1 only
(b) 2 and 3 only
(c) 1 and 3 only
(d) 1, 2 and 3
Whatsapp No 7023213423 http://iasselfstudy.com/ 30

The Banking Ombudsman Scheme


Purpose The Banking Ombudsman Scheme enables an expeditious and
inexpensive forum to bank customers for resolution of banking
complaints

Appointment of The Banking Ombudsman is a senior official appointed by the Reserve


BO Bank of India

Coverage All Scheduled Commercial Banks, Regional Rural Banks and Scheduled
Primary Co-operative Banks are covered under the Scheme.

Fee The Banking Ombudsman does not charge any fee for filing and
resolving customers’ complaints.

One can file a complaint with the Banking Ombudsman simply by writing
on a plain paper. One can also file it online or by sending an email to the
Banking Ombudsman

Appeal against If one is not satisfied with the decision passed by the Banking
order of BO Ombudsman, one can approach the appellate authority against the
Banking Ombudsmen’s decision.

Appellate Authority is vested with a Deputy Governor of the RBI.

One can also explore any other recourse and/or remedies available to
him/her as per the law.

IAS Prelims 2010


With reference to the institution of Banking Ombudsman in India, which one of the statements is not
correct?
(a) Banking Ombudsman is appointed by the Reserve Bank of India
(b) Banking Ombudsman can consider complaints from Non-Resident Indians having accounts in India
(c) The orders passed by the Banking Ombudsman, are final and binding on the parties concerned
(d.)The service provided by the Banking Ombudsman is free of any fee

Bancassurance
Bancassurance, i.e., banc + assurance, refers to banks selling the insurance products.

Reverse Mortgage
A reverse mortgage enables a senior citizen to receive a regular/monthly income from a lender
(a bank or a financial institution) against the mortgage of his home.

The borrower continues to reside in the property till the end of his life.
Whatsapp No 7023213423 http://iasselfstudy.com/ 31

Letter of Credit
About L/C Letter of credit is an undertaking by a bank (opening / issuing bank) made
to the seller (beneficiary) on behalf of the buyer (applicant) to pay a
certain amount if the specified documents presented are as per terms of
the letter of credit.

e.g. ABC Ltd export goods to XYZ Ltd in London but want guarantee of
payment.

In this case XYZ Ltd direct his bank to open L/C on behalf of ABC Ltd and
handover the L/C documents to ABC Ltd.

After the goods are delivered to XYZ Ltd as per terms and conditions
mentioned in L/C, ABC Ltd. submits the L/C documents to his bank which
forward the documents to L/C issuing banks and receive the payment.

Benefit of L/C Party cannot deny payment once the L/C conditions are satisfied

Use L/C is widely used in international trade (Import-Export) but now it is


being used in domestic trade also

Bank guarantee
A Bank guarantee is a promise from a bank that if the buyer of BG does not fulfill the
obligations under the contract then Bank will make payment mentioned in the BG to
beneficiary. i.e. beneficiary can encash the BG.

Small Finance Banks


Objective to promote financial inclusion

Scope of activities The small finance bank will primarily undertake

basic banking activities of acceptance of deposits and lending to


unserved and underserved sections including

small business units, small and marginal farmers, micro and small
industries and unorganized sector entities.

Capital minimum paid-up equity capital for small finance banks is Rs. 100
requirement crore

India’s first small Jalandhar-based Capital Small Finance Bank Ltd


finance bank
Whatsapp No 7023213423 http://iasselfstudy.com/ 32

Payments Banks
Objective to promote financial inclusion by Providing

(i) small savings accounts and (ii) payments/remittance services

to migrant labour workforce,

low income households,

small businesses,

Other unorganized sector entities and other users.

Scope of activities Acceptance of demand deposits. Payments bank will initially be


restricted to holding a maximum balance of Rs. 100,000 per individual
customer.

Issuance of ATM/debit cards. Payments banks, however, cannot issue


credit cards.

Payments and remittance services through various channels.

Distribution of non-risk sharing simple financial products like mutual


fund units and insurance products, etc.

The payments bank cannot undertake lending activities

Capital Minimum paid-up equity capital for payments banks shall be Rs. 100
requirement crore.

India’s first Airtel Payments Bank


payments bank

IAS Prelims 2016


The establishment of ‘Payment Banks’ is being allowed in India to promote financial inclusion. Which of
the following statements is/are correct in this context?
1. Mobile telephone companies and supermarket chains that are owned and controlled by residents are
eligible to be promoters of Payment Banks.
2. Payment Banks can issue both credit cards and debit cards.
3. Payment Banks cannot undertake lending activities.
Select the correct answer using the code given below.
(a) 1 and 2 only
(b) 1 and 3 only
(c) 2 only
(d) 1, 2 and 3

India Post Payments Bank *


The India Post Payments Bank (IPPB) has been incorporated as a Public Limited Company
under the Department of Posts (Ministry of communication) with 100% GOI equity.
Whatsapp No 7023213423 http://iasselfstudy.com/ 33

New Bank License to Bandhan Bank & IDFC


Bandhan Bandhan Bank is the first commercial bank from Eastern India to get RBI
Bank clearance since independence

IDFC RBI had also granted banking licence to the the Infrastructure Development
Finance Company (IDFC) Ltd.

No new Indian bank has been formed since Yes Bank in 2004.

White Label ATM


Objective to promote financial inclusion

Need While, there has been year-on-year growth in the number of ATMs, yet their
deployment has been predominantly in Tier I & II centres.

To expand the reach of ATMs in Tier III to VI centres, non-banks entities were
also allowed to set up ATMs, and such ATMs are known as White Label ATMs.

SBI and ICICI Bank Ltd. as Domestic Systemically Important Banks (D-SIBs)
What is D-SIBs D-SIBs are perceived as banks which are equivalent of too-big-to-fail in
other countries

Requirement A few banks assume systemic importance due to their size, cross-
jurisdictional activities, complexity, lack of substitutability and
interconnectedness.

The disorderly failure of these banks has the propensity to cause


significant disruption to the essential services provided by the banking
system, and in turn, to the overall economic activity.

Impact Banks falling in the D-SIB category need to set aside more capital per
loan than their peers to prevent a contagion effect which can potentially
weigh down other banks in the financial system if there is a crisis.

When RBI RBI discloses the names of banks designated as D-SIBs every year in
announces D- August
SIBs
Whatsapp No 7023213423 http://iasselfstudy.com/ 34

7-Point Reform Agenda called Indradhanush for Public Sector Banks


Appointments The Government has decided to separate the post of Chairman and
Managing Director

Setting up of Bank The BBB will be a body of eminent professionals and officials.
Board Bureau

Capitalization Government of India will infuse Rs.70,000 crores out of budgetary


allocations for four years as per the figures given below:

Financial Year 2015 -16 - Rs. 25,000 crore

Financial Year 2016-17 - Rs. 25,000 crore

Financial Year 2017-18 - Rs. 10,000 crore

Financial Year 2018-19 - Rs. 10,000 crore

SBI will get the highest allocation.

De-stressing PSBs The infrastructure sector and core sector have been the major
recipient of PSBs funding during the past decades.

But due to several factors such as delay in obtaining permits /


approvals from various governmental and regulatory agencies, land
acquisition, lack of availability of fuel etc., projects are increasingly
stalled/stressed thus leading to NPA burden on banks.

Project Monitoring Group (Cab.Sectt.) / Respective Ministries will


pursue with concerned agencies to facilitate issue of pending
approval/permits expeditiously.

Empowerment There will be no interference from Government and Banks are


encouraged to take their decision independently

Framework of A new framework of Key Performance Indicators (KPIs) to be


Accountability measured for performance of PSBs is being announced.

it will be linked to the performance bonus

Governance The process of governance reforms started with “GyanSangam” - a


Reforms conclave of PSBs and FIs
Whatsapp No 7023213423 http://iasselfstudy.com/ 35

Banks Board Bureau (BBB) *


Purpose With a view to improve the Governance of Public Sector Banks (PSBs),
the Government decided to set up an autonomous Banks Board Bureau

Function The Bureau will recommend for selection of heads - Public Sector Banks
and Financial Institutions and help Banks in developing strategies and
capital raising plans.

Commencement The BBB, which will start functioning from 1st April, 2016

Chairman Shri Vinod Rai, Former CAG of India, appointed as the Chairman of
Banks Board Bureau

A non-cooperative borrower
Definition A non-cooperative borrower is one who does not engage constructively with
by RBI his lender by defaulting in timely repayment of dues while having ability to
pay, thwarting lenders’ efforts for recovery of their dues by not providing
necessary information sought, denying access to assets financed / collateral
securities, obstructing sale of securities, etc.

In effect, a non-cooperative borrower is a defaulter who deliberately stone


walls legitimate efforts of the lenders to recover their dues

High Banks/FIs will therefore be required to make higher provisioning as


provisioning applicable to substandard assets in respect of new loans sanctioned to such
borrowers

National Payments Corporation of India (NPCI) *


About NPCI It is an umbrella organization for all retail payments system in India.

NPCI is a not for profit company which is charged with a responsibility of


guiding India towards being a cashless society.

Establishment It was set up with the guidance and support of the Reserve Bank of India
(RBI) and Indian Banks’ Association (IBA). It is regulated by RBI

Objective The core objective was to consolidate and integrate the multiple systems
with varying service levels into nation-wide uniform and standard business
process for all retail payment systems.

The other objective was to facilitate an affordable payment mechanism to


benefit the common man across the country and help financial inclusion.
Whatsapp No 7023213423 http://iasselfstudy.com/ 36

Products & Services of NPCI ***


RuPay debit card RuPay is India's own domestic card with own payment gateway system.
It’s objective is to offer a domestic, open-loop, multilateral system
which will allow all Indian banks and financial institutions in India to
participate in electronic payments.

Since the transaction processing will happen domestically, it would lead


to lower cost of clearing and settlement for each transaction as
compared to Master and Visa card which are based outside India.

Aadhaar It uses Aadhaar number as a central key for electronically channelizing


Payment Bridge the Government subsidies and benefits in the Aadhaar Enabled Bank
(APB) System Accounts (AEBA) of the intended beneficiaries.

Aadhaar Enabled AEPS is a bank led model which allows online interoperable financial
Payment System inclusion transaction at PoS (MicroATM) through the Business
correspondent of any bank using the Aadhaar authentication.
(AEPS)
The four Aadhaar enabled basic types of banking transactions are as
follows:-

Balance Enquiry
Cash Withdrawal
Cash Deposit
Aadhaar to Aadhaar Funds Transfer
The only inputs required for a customer to do a transaction under this
scenario are:-

IIN (Identifying the Bank to which the customer is associated)

Aadhaar Number

Fingerprint captured during their enrollment

Aadhar pay is a merchant version of AEPS

National Unified *99# is a USSD (Unstructured Supplementary Service Data) based


USSD Platform mobile banking service that brings together diverse ecosystem partners
(NUUP) or *99# such as Banks & TSPs (Telecom Service Providers).
Service
Using *99# service, a customer can access financial services by 36ialing
*99# from his/her mobile registered with the bank.

The service works across all GSM service providers and handsets.

It Works without Internet – Uses voice connectivity

Unstructured Supplementary Service Data (USSD) is a technology


unique to GSM (Global System for Mobile Communications) handsets
Whatsapp No 7023213423 http://iasselfstudy.com/ 37

Query Service on *99*99# is a USSD based value added service that facilitates the
Aadhaar Mapper customers to check the status of his/her Aadhaar number
(QSAM) or seeding/linking in the bank account.
*99*99# Service

BHIM App Bharat Interface for Money is an app that lets you make easy and quick
(Bharat Interface payment transactions using UPI. It is easier than Wallets!
for Money)
You will not have to fill-out those tedious bank account details again
and again. You can easily make direct bank to bank payments and
instantly collect money using just Mobile number or Payment address.

All payments over Bharat Interface for Money are linked to your bank
account and transaction can be completed within few seconds.

There are no charges for making transaction through Bharat Interface


for Money.

Note - Your bank might however levy a nominal charge as UPI or IMPS
transfer fee which is not under our control.

Your account need to be enabled for mobile banking to use Bharat


Interface for Money. Your mobile number shall have to be registered
with the Bank.

To enable transfers directly using your bank account, your bank needs
to be live on UPI (Unified Payment Interface) platform. All the banks,
which are currently live on UPI, have been listed in the Bharat Interface
for Money app.

Currently, Bharat Interface for Money supports linking of one Bank at a


time.

Unified Payment Interface (UPI) is an instant payment system


developed by the National Payments Corporation of India (NPCI), an
RBI regulated entity. UPI is built over the IMPS infrastructure and allows
you to instantly transfer money between any two parties’ bank
accounts.

All payments are instant and 24/7, regardless of your bank's working
hours!

Can I send money to a friend not registered on Bharat Interface


for Money?

Yes. Payment can be made via (IFSC, Account number) or (MMID,


Mobile number) if the person is not registered on Bharat Interface for
Money.

BharatQR code Bharat QR Code is a Quick Response (QR) code supporting Visa,
MasterCard and Rupay cards.
Whatsapp No 7023213423 http://iasselfstudy.com/ 38

Bharat QR code enables merchants to accept electronic payments


without the need for a POS machine.

Payments using Bharat QR code:

Customers with Mobile app click on Bharat QR to scan the QR code at


the merchant establishment.

Pockets app automatically enables the camera in the phone.

Customers will be presented with a payment screen displaying


merchant details and amount to pay.

Customers can select from any of their Visa, MasterCard or Rupay cards
for payment.

Transaction is completed and the customer sees a success screen. The


merchant receives a notification on his phone for the transaction.

Immediate IMPS is an innovative real time payment service that is available round
Payment Service the clock (24x7) including on holidays.
(IMPS)
It empowers customers to transfer money instantly through banks.

An unbanked customer can initiate IMPS transaction using the services


of RBI authorized Prepaid Payment Instrument Issuers (PPI).

IMPS is an immediate fund transfer service, after initiating the payment


request payment cannot be stopped or cancelled

a customer can do IMPS transaction:

• Using Beneficiary Mobile no. and MMID or

• Using Beneficiary Account no. and IFS Code or

• Using Beneficiary Aadhaar Number

National NFS is the largest network of shared automated teller machines (ATMs)
Financial Switch in India.
(NFS)
It was designed, developed and deployed with the aim of inter-
connecting the ATMs in the country and facilitating convenience
banking.
Whatsapp No 7023213423 http://iasselfstudy.com/ 39

Mobile Money Identifier (MMID) *


Issued by It is a 7 digit number, issued by banks

Use MMID is one of the input which when clubbed with mobile number
facilitates fund transfer.

Combination of Mobile no. & MMID is uniquely linked with an Account


number and helps in identifying the beneficiary details.

One account one Every bank account has only one MMID
MMID

Linking of different Different MMID’s can be linked to same Mobile Number


bank account

CISF-LDC-2015
'RuPay' is the name of a
(a) Debit Card
(b) Digital Currency
(c) Pension Scheme
(d) Post Office Savings Scheme

ES-2016
“RuPay”, recently in the news, is the name of a :
(a) Debit card
(b) Digital currency
(c) Super computer
(d) Tax benefit

Indian Financial System Code (IFSC)


Issued by RBI

Issued to Bank-Branch

Purpose It identifies a bank-branch where online payment is to be credited in


beneficiary account

Nature This is an alpha-numeric 11 digit code with the first 4 alpha characters
representing the bank, and the last 6 characters representing the branch.
The 5th character is 0 (zero).

e.g. IFSC code of HDFC Chandni Chowk-Delhi Branch is HDFC0000553


where 000553 is branch code
Whatsapp No 7023213423 http://iasselfstudy.com/ 40

LOAN TO VALUE RATIO


About LTV LTV denotes the amount banks can finance to a borrower for a
property purchase.

A 90% LTV indicates that the buyer will have to pay only 10% of the
property value and the rest can be financed through banks.

RBI guideline RBI allowed a loan-to-value ratio (LTV) of up to 90% for home loans
of Rs.30 lakh or less

For properties above Rs 30 lakh and up to Rs 75 lakh, the LTV will be


up to 80 per cent and

those above Rs 75 lakh, it will be 75 per cent.

Islamic or Sharia banking


Islamic or Sharia banking is based on the principles of not charging interest, which is
prohibited under Islam.

The Reserve Bank of India (RBI) has proposed opening of “Islamic window” in conventional
banks for “gradual” introduction of Sharia-compliant or interest-free banking in the
country.

Bad Bank *
Public sector banks (PSBs), where the bulk of the bad loans reside, cannot raise enough
capital to fund credit growth which in turn affect the Growth rate of India.

The finance ministry is considering to set up of a ‘bad bank’ that will absorb the non-
performing assets (NPAs) of public sector banks and help them clean up their books.

Merchant Discount Rate (MDR) **


The commission charged by the bank to the Merchant (Trader/Service Provider) for
providing debit and credit card services through POS terminal.

It is also termed as Merchant Service Fee (MSF).


Whatsapp No 7023213423 http://iasselfstudy.com/ 41

Chapter 3:

Finance & Investments

Foreign Direct Investments (FDI) and Foreign Institutional Investors


(FIIs) *
FDI When a foreign company invests in India directly by setting up a
wholly owned subsidiary or getting into a joint venture and
conduct business in India.

FII When foreign investors invest in the shares/bonds of a company


that is listed in India

Nature FDI invests in new production activities hence help in economic


development. Whereas FIIs invest mainly in capital market for
short term

FDI inflow Major portion of FDI in India is from Mauritius as it has double
taxation avoidance agreement with India. Singapore is on second
position.

Double Taxation DTAA is a tax treaty signed between two countries to avoid dual
Avoidance taxation on same income.
Agreement
It makes a country an attractive investment destination.

But even legitimate investors were routing investments through


Mauritius and Singapore to sidestep taxation.

Hence India has amended DTAA with Mauritius, Singapore and


Cyprus to curb tax evasion.

Amendment provides for source based taxation of capital gains


arising from transfer of shares on or after 1st April, 2017 in a
company resident in India instead of residence based taxation

IAS Prelims 2010


A great deal of Foreign Direct Investment (FDI) to India comes from Mauritius major and mature
economies like UK and France. Why?
(a.) India has preference, for certain countries as regards receiving FDI
(b.) India has double taxation avoidance agreement with Mauritius
Whatsapp No 7023213423 http://iasselfstudy.com/ 42

(c.) Most citizens of Mauritius have ethnic identity with India and so they feel secure to invest in
India
(d.) Impending dangers of global climatic change prompt Mauritius to make huge investments in
India

IAS Prelims 2011


Both foreign direct investment (FDI) and foreign institutional investor (FII) are related to
investment in a country. Which one of the following statements best represents an important
difference between the two?
(a) FII helps bring better management skills and technology. While FDI only brings in capital.
(b) FII helps in increasing capital availability in general, while FDI only targets specific sectors.
(c) FDI flows only into the secondary market, in general, while FII targets primary market.
(d) FII is considered to be more stable than FDI.

IAS Prelims 2002


Global capital flows to developing countries increased significantly during the nineties. In view of
the East Asian financial crisis and Latin American experience, which type of inflow is good for the
host country?
(a) Commercial loans
(b) Foreign Direct Investment
(c) Foreign Portfolio Investment
(d) External Commercial Borrowings

Joint Declaration for Automatic Exchange of Information with Switzerland


It will now be possible for India to receive from September, 2019 onwards, the financial
information of accounts held by Indian residents in Switzerland for 2018 and subsequent
years, on an automatic basis

Green bond
A green bond is like any other regular bond but with one key difference, the money raised
by the issuer are earmarked towards financing `green' projects

Zero Coupon bond


Zero coupon bonds are bonds with no coupon payments

These are issued at discount and repayment are made at par value.

Eg. Face value of bond is Rs. 100. Issued at Rs. 90 and repayable at Rs. 100
Whatsapp No 7023213423 http://iasselfstudy.com/ 43

Disinvestment Policy
Strategic Sale of In the strategic sale of a company, the transaction has two
PSU elements:

 Transfer of a block of shares to a Strategic Partner and

 Transfer of management control to the Strategic Partner

Salient features - Public Sector Undertakings are the wealth of the Nation and to
of the Policy ensure this wealth rests in the hands of the people, promote public
ownership of CPSEs;

-While pursuing disinvestment through minority stake sale in listed


CPSEs, the Government will retain majority shareholding, i.e. at
least 51% of the shareholding and management control of the Public
Sector Undertakings;

-Strategic disinvestment by way of sale of substantial portion of


Government shareholding in identified CPSEs upto 50% or more,
along with transfer of management control.

Main objectives  To reduce the financial burden on the Government


of disinvestment
 To improve public finances

 To introduce, competition and market discipline

 To fund growth

 To encourage wider share of ownership

 To depoliticize non-essential services

Committees NITI Aayog had constituted a Committee under the Chairmanship


of Dr. Arvind Panagariya, Vice Chairman, NITI Aayog on sick/loss-
making/non-performing Central Public Sector Enterprises (CPSEs)

The committee noted that financial performance of 74 CPSEs has


been unsatisfactory and a number of these have requested for and
received periodic support from budgetary resources.

Over the years, this has become a significant drain on limited


resources of the government.

The committee has made the recommendations of closure/winding


up of 26 such CPSEs.

IAS Prelims 2011


Why is the government of India disinvesting its equity in the central public sector enterprises
(CPSEs) ?
1. The government intends to use the revenue earned from the disinvestment mainly to pay back
the external debt.
Whatsapp No 7023213423 http://iasselfstudy.com/ 44

2. The government no longer intends to retain the management control of the CPSEs.
Which of the statements given above is/are correct ?
(a) 1 only.
(b) 2 only.
(c) Both 1 and 2.
(d) Neither 1 nor 2.

Foreign Investment Promotion Board *


Administrative Department of Economic Affairs, Ministry of Finance
Ministry

About FIPB The Foreign Investment Promotion Board (FIPB) offers a single
window clearance for applications on Foreign Direct Investment
(FDI) in India that are under the approval route.

The sectors under automatic route do not require any prior


approval from FIPB and are subject to only sectoral laws.

Recommendations FIPB is responsible for processing of FDI proposals and making


to Govt. recommendations for Government approval.

Sanctioning power Ministry of The Minister of Finance would consider the


Finance recommendations of FIPB on proposals with
total foreign equity inflow of upto Rs. 5000
crore.

Cabinet Recommendations of FIPB on proposals with


Committee on total foreign equity inflow of more than Rs.
Economic Affairs 5000 crore would be placed for consideration of
(CCEA) CCEA.

The CCEA would also consider the proposals


which may be referred to it by the
FIPB/Minister of Finance

Budget 2017 Foreign Investment Promotion Board to be abolished in 2017-18

Finance combined with technology or FinTech *

FinTech is an industry comprising companies generally startups that use technology to

offer financial services


Whatsapp No 7023213423 http://iasselfstudy.com/ 45

Seed capital
Seed capital is the initial funding required to start a new business.

Angel investors
Angel investors provide the initial support and sometimes mentorship to startups.

They are usually experienced entrepreneurs and typically use their own money.

Venture Capital Fund *

A fund set up for the purpose of investing in startup businesses.

Venture Capitalist are firms or companies that use other people's money.

IAS Prelims 2014


What does venture capital mean?
A. A short-term capital provided to industries
B. A long-term start-up capital provided to new entrepreneurs
C. Funds provided to industries at times of incurring losses
D. Funds provided for replacement and renovation of industries

Death Valley Curve


Used in Venture capital

About it It refers to period from when a startup raises an initial capital till it
starts generating revenues.

Initial costs of almost all the start-ups are very high.

This is called Death Valley curve as the startup is most vulnerable


to death because of additional capital requirements as income has
not yet been generated.

Thus startup has to manage itself during the Death Valley phase.
Whatsapp No 7023213423 http://iasselfstudy.com/ 46

Chit Fund Company *


Registered by State Governments under the Chit Funds Act, 1982 – an Act
administered by the Ministry of Finance but with responsibilities of
implementation resting with the States

Power of Power to investigate and prosecute the Chit Fund Company lies
investigation with the State Governments.

Investigation under Serious Fraud Investigation Office (SFIO) under the Ministry of
Companies act Finance can investigate the Chit Fund companies on violations of
provision of Companies act, 2013

Shell Companies ***


Meaning Companies which does not conduct any operations and indulge in
money laundering

Status in India In India, There are about 15 lakh registered companies in India; and
only 6 lakh companies file their Annual Return.

This means that large number of these companies may be indulging in


financial irregularities

Characteristics Shell Companies’ are characterized by nominal paid-up capital, high


of Shell reserves & surplus on account of receipt of high share premium,
companies investment in unlisted companies, no dividend income, high cash in
hand, private companies as majority shareholders, low turnover &
operating income, nominal expenses, nominal statutory payments &
stock in trade, minimum Fixed Asset.
Whatsapp No 7023213423 http://iasselfstudy.com/ 47

Different PPP models

The Build Operate and Transfer BOT (Annuity) Model


Cost & Maintenance Developer meets the construction cost and the maintenance
expenditure.

Recovery from Developer recovers his cost and return on his investment out of
Annuities the annuities payable by the Govt./NHAI every year

BOT (Toll) Model


Cost & Maintenance Developer meets the construction cost and the maintenance
expenditure.

Recovery from Toll Developer recovers his cost and return on his investment out of
the future toll collection.

Engineering, Procurement and Construction (EPC) Model


Cost Cost is borne by the government

Developers’ scope Construction of road and raise the running bill on Govt for payment.

The Hybrid Annuity Model (HAM) **


About HAM The HAM is a mix of EPC and BOT model

Govt Share National Highways Authority of India (NHAI) will pay 40% of the
project cost in the five equal installment linked to project
completion milestone

Developer share Developer has to bear the remaining 60% cost through a
combination of equity and debts and construct the highway.
Whatsapp No 7023213423 http://iasselfstudy.com/ 48

On project Authority shall pay 60% cost to developer through Semi- annual
completion annuity payment.

Project cost shall be inflation indexed.

Interest shall be paid on reducing balance of cost.

Interest rate shall be bank rate plus 3%

Toll collection Toll collection shall be the responsibility of the NHAI.

Maintenance Developer shall be responsible for maintenance till the end of


concession period for which payment will be made

Concession period Construction period plus operation period of 15 years

Swiss Challenge Method (SCM) a model to develop unsolicited proposals *


About SCM It allows other parties to make better offers for a project and the project
proponent then has the right to counter-match any superior offers

Process Project proponent shall submit unsolicited/innovative proposal

Authority will provide approval after examination of the proposal

An open bidding shall be done by the concerned authority inviting bids


from various organizations

If the competing bidder provides the best financial offer, the project
proponent shall be given an opportunity to match the counter proposal

Viability Gap funding scheme *


About VGF Govt provides financial support in the form of capital grants, one time or
deferred, to infrastructure projects undertaken through PPPs with a view
to make them commercially viable

Administered The Scheme for Financial Support to PPPs in Infrastructure (Viability Gap
by Funding scheme) of the Government of India is administered by the
Ministry of Finance

Funding % The Government of India provides total Viability Gap Funding up to 20%
of the total project cost; at the stage of project construction.

The Government or statutory entity that owns the project may, provide
additional grants out of its budget up to further 20% of the total project
cost.
Whatsapp No 7023213423 http://iasselfstudy.com/ 49

Mezzanine financing
Mezzanine financing is a debt capital that gives the lender the right to convert the loan into
equity in case of non-repayment

Crowd Funding
Crowd Funding generally refers to a method of funding a project or venture through small
amounts of money raised from a large number of people, typically through a portal acting
as an intermediary.

Crowd funding is equity, debt based and fund based. it falls under the purview of capital
markets regulator (SEBI)

P2P lending *
Online It is the use of an online platform that matches lenders with borrowers
platform in order to provide unsecured loans

interest rate interest rate may be set by the platform or by mutual agreement
between the borrower and the lender

Charges by The platforms do the credit scoring and make a profit from arrangement
platform fees and not from the spread between lending and deposit rates as is
the case with normal financial intermediation

Benefit of P2P for borrowers- lower interest rates than those offered by money
lending lenders/unorganized sector and

for lenders- higher returns than what conventional investment


opportunities offer

Regulator Bank

Predatory Pricing-Reliane Jio **


Meaning The predator sets its prices so low for a sufficient period of time that its
competitors leave the market and others are deterred from entering

Reliance Jio Reliance Jio has offered free voice and data offer.

Other telecom operators have moved to Commission of India (CCI)


accusing the Mukesh Ambani-led Reliance Jio of predatory pricing, and
anti-competitive behavior.
Whatsapp No 7023213423 http://iasselfstudy.com/ 50

Public Financial Management System (PFMS) *


About it PFMS is a web-based online transaction system for payment, accounting
and reconciliation of Government transactions.

Objective The primary objective of PFMS is to establish an efficient fund flow


system and expenditure network

Benefit PFMS helps in ensuring ‘Just in Time’ releases and monitor the end
uses of funds

Uses Govt has decided to use the PFMS to cover all transactions in respect of
Central Sector (CS) and Central Assistance to State Plan (CASP)
schemes.

Administered Controller General of Accounts in the Department of Expenditure,


by Ministry of Finance
Whatsapp No 7023213423 http://iasselfstudy.com/ 51

Chapter 4:

Taxation

Goods and Service Tax (GST) ***


Road map The idea of moving towards the GST was first mooted in the Budget for
2006-07.

101st Constitution Amendment Act coming into force on 8th September,


2016 and notification of the GST Council on 15th September – the road
to GST rollout is clear.

Government is keen on introducing GST-the biggest indirect tax reform,


with effect from 01 April 2017

New deadline After missing April deadline, now GST to be implemented from July 1
for GST

Biggest to train the indirect tax officials of both Centre and State, as well as the
challenges trade on the concepts, processes and procedures of GST.

National Academy of Customs, Excise & Narcotics (NACEN), the apex


training institution for capacity building in indirect taxation under the
Central Board of Excise and Customs, has been mandated to impart
training on GST to Central and State Government officers.

What is GST It is a destination based tax on consumption of goods and services.

It is proposed to be levied at all stages right from manufacture up to


final consumption with credit of taxes paid at previous stages available
as setoff.

In a nutshell, only value addition will be taxed and burden of tax is to be


borne by the final consumer.

Concept of The tax would accrue to the taxing authority which has jurisdiction over
destination the place of consumption which is also termed as place of supply
based tax on
consumption

Existing taxes The GST would replace the following taxes:


are proposed to
(i) taxes currently levied and collected by the Centre:
be subsumed
under GST Excise & Custom duty

Service Tax
Whatsapp No 7023213423 http://iasselfstudy.com/ 52

Central Surcharges and Cesses related to supply of goods and services

(ii) State taxes that would be subsumed under the GST are:

State VAT

Central Sales Tax

Luxury Tax

Entry Tax (all forms)

Entertainment and Amusement Tax (except when levied by the local


bodies)

Taxes on advertisements

Purchase Tax

Taxes on lotteries, betting and gambling

State Surcharges and Cesses related to supply of goods and services

Exclusion Taxes on entertainments and amusements to the extent levied and


collected by a Panchayat or a Municipality or a Regional Council or a
District Council shall not be subsumed under GST.

The local bodies of States could continue to levy such taxes.

GST Council The GST Council shall make recommendations to the Union and States
on the taxes, cesses and surcharges levied by the Centre, the States and
the local bodies which may be subsumed in the GST.

Applicability of The GST shall be levied on all goods and services except alcoholic liquor
GST for human consumption

Status of Petroleum & petroleum products would be subject to GST.


Petroleum &
However, it has been decided that five products, viz. petroleum crude,
petroleum
motor spirit (petrol), high speed diesel, natural gas and aviation turbine
products
fuel would be kept out of the purview of GST in the initial years of
implementation

Status of Tobacco and tobacco products would be subject to GST.


Tobacco and
In addition, the Centre would have the power to levy Central Excise duty
Tobacco
on these products
products

Power to make Parliament will have power to make laws with respect to GST imposed by
law the Union (CGST) and the State Legislatures will have power to make
laws with respect to GST imposed by the States (SGST)
Whatsapp No 7023213423 http://iasselfstudy.com/ 53

Type of GST It would be a dual GST with the Centre and States simultaneously
levying it on a common tax base.

The GST to be levied by the Centre on intra-State supply of goods and /


or services would be called the Central GST (CGST) and

that to be levied by the States would be called the State GST (SGST).

Similarly Integrated GST (IGST) will be levied and administered by


Centre on every inter-state supply of goods and services.

Why is Dual India is a federal country where both the Centre and the States have
GST required been assigned the powers to levy and collect taxes through appropriate
legislation.

A dual GST will, therefore, be in keeping with the Constitutional


requirement of fiscal federalism

Benefits The GST will create a common Indian market, improve tax compliance
and governance, and boost investment and growth

Who will decide The CGST and SGST would be levied at rates to be jointly decided by the
rates for levy of Centre and States. The rates would be notified on the recommendations
GST? of the GST Council

GST Council A GST Council would be constituted comprising the Union Finance
Minister (who will be the Chairman of the Council), the Minister of State
(Revenue) and the State Finance/Taxation Ministers

Decisions be Every decision of the GST Council shall be taken at a meeting by a


taken by GST majority of not less than 3/4th of the weighted votes of the Members
Council present and voting.

The vote of the Central Government shall have a weightage of 1/3rd of


the votes cast and the votes of all the State Governments taken together
shall have a weightage of 2/3rd of the total votes cast in that meeting.

One half of the total number of members of the GST Council shall
constitute the quorum at its meetings.

Who is liable to Tax is payable by the taxable person on the supply of goods and/or
pay GST services.

Tax payers with an aggregate turnover in a financial year up to [Rs.20


lakhs] would be exempt from tax. (Rs. 10 lakhs for Special Category
States e.g. North east states)

Imports Imports of Goods and Services will be treated as inter-state supplies and
IGST will be levied on import of goods and services into the country.
Whatsapp No 7023213423 http://iasselfstudy.com/ 54

The incidence of tax will follow the destination principle and the tax
revenue in case of SGST will accrue to the State where the imported
goods and services are consumed.

Exports Exports will be treated as zero rated supplies.

No tax will be payable on exports of goods or services, however credit of


input tax credit will be available and same will be available as refund to
the exporters

GSTN GSTN stands for Goods and Service Tax Network (GSTN).

A Special Purpose Vehicle called the GSTN has been set up to cater to
the needs of GST.

The GSTN shall provide a shared IT infrastructure and services to Central


and State Governments, tax payers and other stakeholders for
implementation of GST.

Compensation CG has assured state govts of compensation for any revenue loss due to
to states introduction of GST for a period of five years.

GST rate Bands of rates of goods under GST shall be 5%, 12%, 18% and 28%.

In addition, there would be a category of exempt goods.

Further, a cess would be levied on certain goods such as luxury cars,


aerated drinks, pan masala and tobacco products, over and above the
rate of 28% for payment of compensation to the States.

Upper cap on The GST Council has decided to keep the upper cap higher at 40% (20%
GST rate CGST and 20% SGST) so that in future in case of need to hike tax rate,
there is no need to approach Parliament for a nod and the GST Council
can raise it

CDS 2017
Which one of the following statements is correct in relation to the GST Bill passed by the Rajya Sabha
in August 2016?
(a) It will replace all central taxes, duties, etc., only by a single tax.
(b) It will subsume central as well as State taxes, duties, etc.
(c) GST will be levied on alcoholic liquor for human consumption at a uniform rate of 25 percent.
(d) Petroleum and petroleum products shall not be subjected to the levy of GST

CDS 2017
Goods and Services Tax likely to be levied in India is not a
(a) gross value tax
(b) value-added tax
(c) consumption tax
(d) destination-based tax
Whatsapp No 7023213423 http://iasselfstudy.com/ 55

ES 2017
Consider the following statements regarding GST :
1. The GST Bill 2014 has the purpose to improve the Value Added Tax on Goods and Services
2. It can be imposed differently in different States
3. It is a Comprehensive Tax imposed nationwide irrespective of any State concerned
4. It is a significant step in the reform of Indirect Taxation in India

Which of the above statements are correct?


(a) I, 2 and 3
(b) 1, 2 and 4
(c) 2, 3 and 4
(d) 1, 3 and 4

Methods of taxation
Regressive tax system If tax rate is gradually reduced due to increase in income

Proportional tax When tax rate remain constant


system

Progressive tax system If tax rate is gradually increased due to increase in income (India)

CDS-2015
Which one of the following represents a progressive tax structure?
(a) Tax rate is the same across all Incomes
(b) Tax rate increases as income Increases
(c) Tax rate decreases as income Increases
(d) Each household pays equal amount of tax

SCRA-2011
Income tax in India is
(a) progressive
(b) regressive
(c) proportional
(d) based on benefit principle

IAS Prelims 1996


A redistribution of income in a country can be best brought about through
(a) progressive taxation combined with progressive expenditure
(b) progressive taxation combined with regressive expenditure
(c) regressive taxation combined with regressive expenditure
(d) regressive taxation combined with progressive expenditure
Whatsapp No 7023213423 http://iasselfstudy.com/ 56

Minimum Alternative Tax (MAT) *


Objective of At times it happens that a company has generated income during the
levying MAT year, but by taking the advantage of various provisions of Income-tax
Law (like exemptions, deductions, depreciation, etc.), it has reduced its
tax liability or has not paid any tax at all.

The objective of introduction of MAT is to bring into the tax net "zero tax
companies" which in spite of having earned substantial book profits and
having paid handsome dividends, do not pay any tax due to various tax
concessions and incentives provided under the Income-tax Law

MAT calculation MAT is calculated at

18.5% (plus surcharge and cess as applicable) on the book profit (i.e.
profit shown in the profit and loss account)

Tax liability As per the concept of MAT, the tax liability of a company will be higher of
the following Normal tax liability (as per applicable tax provisions) or MAT

MAT credit If in any year the company pays liability as per MAT, then it is entitled to
claim credit of MAT paid over and above the normal tax liability in the
subsequent year(s)

The credit can be adjusted in the year in which the liability of the
company as per the normal provisions is more than the MAT liability

however, the MAT credit can be carried forward only for a period of 15
years after which it will lapse

Applicability of MAT provisions shall not be applicable to a foreign company if —


MAT on foreign
the foreign company is a resident of a country having DTAA with India
companies
and such foreign company does not have a permanent establishment in
India, or

the foreign company is a resident of a country which does not have a


DTAA with India and such foreign company is not required to seek
registration under section 592 of the Companies Act 1956 or section 380
of the Companies Act 2013.

Alternate Minimum Tax (AMT)


MAT applies to companies and AMT applies to a person other than a company

Rate of AMT, AMT is levied @ 18.5% (plus surcharge and cess as applicable) of adjusted total
income
Whatsapp No 7023213423 http://iasselfstudy.com/ 57

Direct Tax *
Meaning When a person bears the burden as well as makes payment to the
government

Examples 1.Corporation tax :.

2. Income tax :

3. Interest tax :

4. Expenditure tax :.

5. Wealth tax :

6. Gift tax :

7. Estate Duty

8. Land revenue

9. Agriculture tax

10. Hotel receipts tax

CDS-2013
Which of the following are included in the category of direct tax in India ?
1. corporation tax
2. tax on income
3. wealth tax
4. customs duty
5. excise duty

(a) 1, 2 & 3
(b) 1,2,4 & 5
(c) 2 & 3 only
(d) 1,3,4 & 5
Whatsapp No 7023213423 http://iasselfstudy.com/ 58

Indirect Tax *
Meaning When a seller collects the tax from the buyer first and then pays the same to
the government.

In other words, it is the buyer who indirectly pays sales tax to the
government.

Examples 1. Customs duties: These are taxes on imports and exports.

2.Union excise duties: These are taxes on manufacturing goods imposed by


the Union government

3. Service tax:It is a tax on rendering of services

4. Sales tax: It is a tax on sales.

5. State excise duty

6. Stamp & Registration fees

7. Entertainment tax

8. Taxes on Vehicles, Goods & Passengers, Electricity, Purchase of sugarcane

Taxes VAT, Taxes on Land and Buildings, Agriculture income, Mineral rights, Liquors,
imposed sale of Electricity, Tolls, Stamp duty, Entertainment tax, Professional tax,
by State Vehicle tax etc
Govt

IAS Prelims 2001


Consider the following taxes :
I. Corporation tax
II. Customs duty
III. Wealth tax
IV. Excise duty
Which of these is/are indirect taxes ?
(a) I only
(b) II and IV
(c) I and III
(d) II and III

ES-2012
Excise Duty is a tax levied on :
(a) Commodities that are exported
(b) Commodities that are imported
(c) Both the exported and imported commodities
(d) Commodities that are produced and consumed within the country
Whatsapp No 7023213423 http://iasselfstudy.com/ 59

IAS Prelims 2006


Which one of the following is the correct statements?
Service tax is a/an
(a) direct tax levied by the Central Government.
(b) indirect tax levied by the Central Government
(c) direct tax levied by the State Government
(d) indirect tax levied by the State Government

IAS Prelims 2014


The sales tax you pay while purchasing a toothpaste is a
(a) tax imposed by the Central Government
(b) tax imposed by the Central Government but collected by the State Government
(c) tax imposed by the State Government but collected by the Central Government
(d) tax imposed and collected by the State Government
Whatsapp No 7023213423 http://iasselfstudy.com/ 60

Chapter -5

National Income, Budget & Economic Survey

Key economic indicators- GVA & GDP at Constant prices (Base year 2011-
12) ***
Issued Monthly

Issued by The Central Statistics Office (CSO) (Ministry of Statistics and Program
Implementation)

CSO also releases Quarterly Estimates of National Income/GDP at


constant price ( Base 2011-12)

Gross Domestic Market value of all final goods and services taking place within the
Product (GDP) domestic economy during a year.

or

GDP at market price =

Gross Value Added (GVA) at basic price + Indirect tax- Subsidies

Gross National GDP


Product (GNP)
Add: Income earned by the domestic factors of production employed in
the rest of the world. (i.e. Indians or Indian companies abroad)

Less: Income earned by the factors of production of the rest of the


world employed in the domestic economy. ( i.e. profit earned by MNCs
in India e.g. Google, IBM)

Hence, GNP = GDP + Net factor income from abroad.

Net National NNP = GNP – Depreciation


Product (NNP)

NNP at factor All above variables are evaluated at market prices. But market price
cost or National includes indirect taxes. Indirect taxes accrue to the government.
Income ***
National Income= NNP at market prices – Indirect taxes + Subsidies

CDS 2017
The 7.6% growth rate registered by Indian economy during the year 2015-16 is based on
(a) Gross National Product at market prices
(b) Gross Value Added at constant prices
(c) Gross Domestic Product at market prices
(d) Gross Domestic Product at constant prices
Whatsapp No 7023213423 http://iasselfstudy.com/ 61

CDS-2014
National product at factor cost is equal to
(a) Domestic product + Net factor income from abroad
(b) National product at market prices - indirect taxes + subsidies
(c) Gross domestic product - depreciation
(d) National product at market prices + Indirect taxes + subsidies

CDS-2016
Which one of the following statements is correct with respect to the composition of national income in
India?
(a) The share of manufacturing sector has declined .
(b) The share of services sector has increased sharply.
(c) The share of agriculture has remained static.
(d) The share of services sector has declined.

CDS-2012
Which of the following statements is/are, correct?
1. If a country is experiencing increase in its per capita GDP, its GDP must necessarily be growing.
2. 'If a country is experiencing negative inflation, its GDP must be decreasing.
'Select the correct answer using the code given below :
Code:
(a) 1 only
(b) 2 only
(c) Both 1 and 2
(d) Neither 1 nor 2

CDS-2012
National income ignores
(a) sales of a firm
(b) salary of employees
(c) exports of the IT sector
(d) sale of land

CDS-2009
Consider the following statements about Gross Domestic Product :
l. It is the market value of all final goods and services made within the borders of a nation in a year.
2. It is equal to the total expenditures for all final goods and services produced within the country in a
stipulated period of time.
Which of the statements given above is I are correct ?
(a) 1 only
(b) 2 only
(c) Both l and 2
(d) Neither 1 nor 2

SCRA-2014
Gross Domestic Product (GDP) is called 'gross' because its computation does not exclude
(a) depreciation of capital (consumption of capital in production process)
(b) subsidies on consumption of goods
(c) earnings of foreign factors in host country
(d) impact of price rise
Whatsapp No 7023213423 http://iasselfstudy.com/ 62

IAS Prelims 2001


The term National Income represents
(a) Gross National Product at market prices minus depreciation
(b) Gross National Product at market prices minus depreciation plus net factor income from abroad
(c) Gross National Product at market prices minus depreciation and indirect taxes plus subsidies
(d) Gross National Product at market prices minus net factor income from abroad

IAS Prelims 1997


National Income is the
(a) Net National Product at market price
(b) Net National Product at factor cost
(c) Net Domestic Product at market price
(d) Net Domestic Product at factor cost

IAS Prelims 1991


The data collection for national income estimation in India is done by
(a) National Sample Survey Organisation
(b) Finance Ministry of the Govt. of India
(c) Central Statistical Institute/Organisation
(d) Indian Statistical Institute

IAS Prelims 2013


The national income of a country for a given period is equal to the
a. Total value of goods and services produced by the nationals
b. sum of total consumption and investment expenditure
c. sum of personal income of all individuals
d. money value of final goods and services produced

CDS-2014
The value of all final goods and services produced by the normal residents of a country and their
property, whether operating within the domestic territory of the country or outside in a year is termed
as
(a) Gross National Income
(b) Net National Income
(c) Gross Domestic Product
(d) Net Domestic Product
Whatsapp No 7023213423 http://iasselfstudy.com/ 63

Revenue, Fiscal and Primary deficit ***


Revenue deficit Revenue expenditure – Revenue receipts

Effective Revenue Difference between Revenue deficit and Grants for creation of capital
deficit assets

Fiscal deficit Fiscal deficit = Total expenditure – (Revenue receipts+ Non-debt


creating capital receipts)

This indicates the total borrowing requirements of Government from


all sources.

Non-debt creating capital receipts are those receipts which are not
borrowings and, therefore, do not give rise to debt.

For example- recovery of loans and the proceeds from the sale of
PSUs.

Primary deficit Fiscal deficit – Interest payment

Budget deficit Total Expenditure - Total Receipts

IAS Prelims 1999


Assertion (A): Fiscal deficit is greater than budgetary deficit.
Reason (R): Fiscal deficit is the borrowings from the Reserve Bank of India plus other liabilities of the
Government to meet its expenditure.
(a) Both A and R are true, and R is the correct explanation of A
(b) Both A and R are true, but R is not a correct explanation of A
(c) A is true, but R is false
(d) A is false, but R is true

CDS-2013
The concept which tries to ascertain the actual deficit in the revenue account after adjusting for
expenditure of capital nature is termed as
(a) revenue deficit
(b) effective revenue deficit
(c) fiscal deficit
(d) primary deficit

IAS Prelims 2001


Match List I with List II and select the correct answer using the codes given below the Lists :
List I List II
(Term) (Explanation)
I. Fiscal deficit (A) Excess of Total Expenditure over Total Receipts
II. Budget deficit (B) Excess of Revenue Expenditure over Revenue Receipts
III. Revenue deficit (C) Excess of Total Expenditure over Total Receipts less borrowings
IV. Primary deficit (D) Excess of Total Expenditure over Total Receipts less borrowings and
Interest Payments
Codes:
(a) IC, IIA, IIIB, IVD
(b) ID, IIC, IIIB, IVA
(c) IA, IIC, IIIB, IVD
(d) IC, IIA, IIID, IVB
Whatsapp No 7023213423 http://iasselfstudy.com/ 64

IAS Prelims 1992


Deficit financing means
(a) An excess of governments’ current expenditure over its current revenue
(b) An excess of government expenditures minus borrowings other than those from the RBI
(c) An excess of government’s total expenditure over its total revenue
(d) An excess of government revenue over expenditure

IAS Prelims 1994


Fiscal deficit in the Union Budget means
(a) the sum of budgetary deficit and net increase in internal and external borrowings
(b) the difference between current expenditure and current revenue
(c) the sum of monetized deficit and budgetary deficit
(d) net increase in Union Government’s borrowing from the Reserve Bank of India

CAPF-2013
If we deduct grants for creation of capital assets from revenue deficit, we arrive at the concept of
(a) primary deficit (b) net fiscal deficit (c) budgetary deficit (d) effective revenue deficit

IAS Prelims 2013


In India, deficit financing is used for raising resources for
a. economic development
b. redemption of public debt
c. adjusting the balance of payments
d. reducing the foreign debt

Fiscal deficit target of Centre ***


For Financial year 2016-17 3.5% of GDP

For Financial year 2017-18 onwards 3% of GDP

Fiscal deficit targets of States during 2015-20 **


Normal 3% of Gross State Domestic Product (GSDP) as per recommendations of 14th
limit Finance Commission (FFC)

Additional FFC, taking into account the development needs and the current macro-
economic requirement, provided additional headroom to a maximum of 0.5%
over and above the normal limit of 3% in any given year to the States that
have a favourable debt-GSDP ratio and interest payments-revenue receipts
ratio in the previous two years.

However, the flexibility in availing the additional fiscal deficit will be available
to State if there is no revenue deficit in the year in which borrowing limits are
to be fixed and immediately preceding year.
Whatsapp No 7023213423 http://iasselfstudy.com/ 65

Gross Domestic Savings


Meaning Gross Domestic Saving is GDP minus final consumption expenditure. It is
expressed as a percentage of GDP

Composition Gross Domestic Saving consists of:

Household Savings(Rank 1)

Private Corporate Sector Savings (Rank 2)

Public Sector Savings (Rank 3)

THE BALANCE OF PAYMENTS ***

INDIA’S OVERALL BALANCE OF PAYMENTS

A. CURRENT ACCOUNT

1 Exports
2 Imports
3 Balance of Trade (1-2)
4 Invisibles*: *(as no port entry like import-export)
a) Services (e.g. Software services)
b) Transfers (e.g. Remittance, Grant)
c) Income (e.g. Investment income)
5 Current Account Balance (3+4)

B. CAPITAL ACCOUNT

1 Foreign Investment
a) Direct Investment
b) Portfolio Investment
2 External Assistance
3 Commercial Borrowings
4 Short Term Loan (e.g. Suppliers’ credit)
5 Banking Capital
6 Rupee Debt Service
7 Other Capital
8 Total Capital Account (1 to 7)

Overall balance (A+B)

RBI releases data for BoP quarterly basis in Financial year. (e.g. April-June, July-Sept, Oct-Dec
and Jan-Mar)
Whatsapp No 7023213423 http://iasselfstudy.com/ 66

IAS Prelims 1992


Invisible trade is a trade
(a) Of corporate and financial institutions with government
(b) Of government with public institutions
(c) Of government with other countries
(d) Of the services like the banks, marine companies and shipping companies

CISF-LDC-2013
Which one of the following describes Current Account Deficit in India ?
(a) Value of imports is more than value of exports in a year
(b) Value of exports is more than value of imports in a year
(c) Depreciation of rupee value against US dollar during the financial year
(d) A government falling into a debt trap

IAS Prelims 2014


With reference to Balance of Payments, which of the following constitutes/constitute the Current
Account?
1. Balance of trade
2. Foreign assets
3. Balance of invisibles
4. Special Drawing Rights
Select the correct answer using the code given below.
A. 1 only
B. 2 and 3
C. 1 and 3
D. 1, 2 and 4

IAS Prelims 2013


The balance of payments of a country is a systematic record of
a. all import and export transactions of a country during a given period of time, normally a year
b. goods exported from a country during a year
c. economic transaction between the government of one country to another
d. capital movements from one country to another

IAS Prelims 2013


Which of the following constitute Capital Account?
1. Foreign Loans
2. Foreign Direct Investment
3. Private Remittances
4. Portfolio Investment
Select the correct answer using the codes given below.
a. 1, 2 and 3
b. 1, 2 and 4
c. 2, 3 and 4
d. 1, 3 and 4

IAS Prelims 2012


Which of the following would include Foreign Direct Investment in India?
1. Subsidiaries of foreign companies in India
2. Majority foreign equity holding in Indian companies
3. Companies exclusively financed by foreign companies
4. Portfolio investment
Select the correct answer using the codes given below:
a) 1, 2 , 3 and 4
b) 2 and 4 only
Whatsapp No 7023213423 http://iasselfstudy.com/ 67

c) 1 and 3 only
d) 1, 2 and 3 only

CDS-2014
Which of the following does not form part of current account of Balance of Payments ?
(a) Export and import of goods
(b) Export and import of services
(c) Income receipts and payments
(d) Capital receipts and payments

IAS Prelims 2006


The following item consists of two statements, one labeled as the Assertion (A) and the other as
Reason (R).
You are to examine these two statements carefully and select the answers to these item using the code
given below :
Assertion (A): Balance of Payments represents a better picture of a country’s economic transactions
with the rest of the world than the Balance of Trade.
Reason (R) : Balance of Payments takes into account the exchange of both visible and invisible items
whereas Balance of Trade does not.
(a) Both A and R are individually true and R is the correct explanation of A.
(b) Both A and R are individually true but R is not the correct explanation of A.
(c) A is True but R is false
(d) A is false but R is true

BoP Surplus and Deficit **


Current account balance represents deficit or surplus in the balance of payments.

a country that has current account deficit must finance it by selling assets or by borrowing
abroad. Thus, any current account deficit is financed by a net capital inflows.

A current account deficit implies that a country's economy is functioning on borrowed means.

A deficit may be planned for the purpose of helping an economy's development and growth.

IAS Prelims 2011


Consider the following actions which the government can take:
1. Devaluing the domestic currency.
2. Reduction in the export subsidy.
3. Adopting suitable policies which attract greater FDI and more funds from FIIs.
Which of the above action/actions can help in reducing the current account deficit?
(a) 1 and 2
(b) 2 and 3
(c) 3 only
(d) 1 and 3
Whatsapp No 7023213423 http://iasselfstudy.com/ 68

Deficit reduction *
Government deficit can be reduced by an increase in taxes or reduction in expenditure.

In India, the government has been trying to increase tax revenue with greater reliance on direct
taxes (indirect taxes are regressive in nature – they impact all income groups equally).

Govt. is trying to raise money through the sale of shares in PSUs.

However, the major thrust has been towards reduction in government expenditure.

in case of a budget deficit, i.e., when Govt. cannot meet its expenses from the tax revenue. So
it borrows money by selling treasury bills or government securities to RBI, which issues
currency to the government in return. The government then pays for its expenses with this
money. The money thus ultimately comes into the hands of the general public.

If budget deficit is financed by raising money then inflation may rise.

IAS Prelims 2013


Which one of the following is likely to be the most inflationary in its effect?
a. Repayment of public debt
b. Borrowing from the public to finance a budget deficit
c. Borrowing from banks to finance a budget deficit
d. Creating new money to finance a budget deficit

Financial motivators
Financial motivators are used during recession time. e.g. reduce the tax rates, increase the
govt. expenditures.

Due to financial motivators, purchasing power of people increases and therefore demand for
the goods and services also increases..

ES-2013
During the time of economic recession :
(a) interest rate should be increased
(b) taxes should be increased
(c) expenditure on public projects should be increased
(d) interest rate and taxes should be increased

SCRA-2010
During the time of recession
(a) interest rate should be increased
(b) taxes should be increased
(c) Cash Reserve Ratio should be increased
(d) expenditure on public projects should be increased

IAS Prelims 2010


Consider the following actions by the Government:
1 Cutting the tax rates
2. Increasing the government spending
3. Abolishing the subsidies
Whatsapp No 7023213423 http://iasselfstudy.com/ 69

In the context of economic recession, which of the above actions can be considered a part of the "fiscal
stimulus" package?
(a.) 1 and 2 only
(b.) 2 only
(c.) 1 and 3 only
(d.) 1, 2 and 3

Convertibility of the rupee **


Means freely permitting the conversion of rupee to other major currencies and
vice versa

Current India currently has full convertibility of the rupee in current accounts such
account as for exports and imports
convertibility

Capital account India does not have full convertibility of the rupee for capital transactions.
convertibility
There are ceilings on government and corporate debt, external
commercial borrowings and equity.

CAPF-2013
Which of the following statements is correct with respect to the convertibility of Indian rupee?
(a) It is convertible on capital account
(b) It is convertible on current account
(c) It is convertible both on current and capital account
(d) None of the above

IAS Prelims 2002


Consider the following statements:
Full convertibility of the rupee may mean
1. its free float with other international currencies.
2. its direct exchange with any other international currency at any prescribed place inside and outside
the country.
3. it acts just like any other international currency.
Which of these statements are correct?
(a) 1 and 2
(b) 1 and 3
(c) 2 and 3
(d) 1, 2 and 3

IAS Prelims 2000


Consider the following statements :
The Indian rupee is fully convertible
I. in respect of Current Account of Balance of Payment.
II. in respect of Capital Account of Balance of Payment.
III. into gold.
Which of these statements is/are correct ?
(a) I alone
(b) III alone
(c) I and II
(d) I, II and III
Whatsapp No 7023213423 http://iasselfstudy.com/ 70

IAS Prelims 1994 & 2015


Convertibility of the rupee implies
(a) being able to convert rupee notes into gold
(b) allowing the value of the rupee to be fixed by market forces
(c) freely permitting the conversion of rupee to other major currencies and vice versa
(d) developing an international market for currencies in India

IAS Prelims 1996


One of the important goals of the economic liberalisation policy is to achieve full convertibility of the
Indian rupee. This is being advocated because
(a) convertibility of the rupee will stabilize its exchange value against major currencies of the world
(b) it will attract more foreign capital inflow in India
(c) it will help promote exports
(d) it will help India secure loans from the world financial markets at attractive terms

IAS Prelims 1998


Capital Account Convertibility of the Indian Rupee implies
(a) that the Indian Rupee can be exchanged by the authorised dealers for travel
(b) that the Indian Rupee can be exchanged for any major currency for the purpose of trade in goods
and services
(c) that the Indian Rupee can be exchanged for any major currency for the purpose of trading financial
assets
(d) None of the above

The Ministry of Finance **


Five Departments Department of Economic Affairs

Department of Expenditure

Department of Revenue

Department of Investment and Public Asset Management (DIPAM)

Department of Financial Services

Budget is prepared by Budget Division of Department of Economic Affairs

Economic Survey is Economic Division of Department of Economic Affairs


prepared by

IAS Prelims 2010


Which of the following is responsible for the preparation and presentation of Union Budget to
the parliament ?
(a)Department of Revenue (b.) Department of Economic Affairs
(c) Department of Financial Services (d) Department of Expenditure

IAS Prelims 1998


Economic Survey in India is published officially, every year by the
(a) Reserve Bank of India (b) Planning Commission of India
(c) Ministry of Finance, Government of India (d) Ministry of Industries, Government of India
Whatsapp No 7023213423 http://iasselfstudy.com/ 71

KEY TO BUDGET DOCUMENTS- BUDGET 2017-2018 **


Budget The Budget documents presented to Parliament comprise, besides the
documents Finance Minister's Budget Speech, the following:

A. Annual Financial Statement (AFS)

B. Demands for Grants (DG)

C. Appropriation Bill

D. Finance Bill

E. Memorandum Explaining the Provisions in the Finance Bill

F. Macro-economic framework Statement

G. Fiscal Policy Strategy Statement

H. Medium Term Fiscal Policy Statement

I. Medium Term Expenditure Framework Statement

J. Expenditure Profile

K. Expenditure Budget

L. Receipts Budget

M. Budget at a glance

N. Highlights of Budget-Key features

O. Outcome Budget

Applicable The documents shown at Serial A, B, C and D are mandated by Article


provisions 112,113, 114(3) and 110(a) of the Constitution of India respectively.

Documents at Serial F, G, H and I are presented as per the provisions of the


Fiscal Responsibility and Budget Management Act, 2003.

Other documents are in the nature of explanatory statements supporting the


mandated documents.
Whatsapp No 7023213423 http://iasselfstudy.com/ 72

Brief description of the Budget documents:

Budget/Annual Financial Statement (AFS) (Article 112 of the Constitution)*


When budget is On such date as fixed by President
presented

Where presented before both the house of parliament

Responsibility of President
budget
presentation

Details of AFS It shows:

Estimated receipts and expenditure of the Government of India for


2017-18.

Budgeted and revised estimates for 2016-17.

Actual expenditure for the year 2015-16.

Three parts The receipts and disbursements are shown under three parts in which
Government Accounts are kept viz.,

Consolidated Fund,

Contingency Fund and

Public Account.

Separation of Annual Financial Statement distinguishes revenue expenditure from


Revenue other expenditure.
expenditure
The Revenue and the Capital sections together, therefore make the
Union Budget.

CDS-2014
The Annual Financial Statement of the Government of India in respect of each financial year shall be
presented to the House on such day as the
(a) Speaker may direct
(b) President of India may direct
(c) Parliament may decide
(d) Finance Minister may decide

ES-2011
Whose duty is it to cause to be laid before the Parliament the Annual Financial Statement (popularly
known as Budget)?
(a) The President of India
(b) The Prime Minister of India
(c) The Union Minister of Finance
(d) The Union Minister of Parliamentary Affairs
Whatsapp No 7023213423 http://iasselfstudy.com/ 73

Consolidated Fund of India (Article 266 of the Constitution) *


Revenue All revenues received by Government, loans raised by it, and also receipts
from recoveries of loans granted by it form the Consolidated Fund

Expenditure All expenditure of Government is incurred from the Consolidated Fund of


India

Withdrawal No amount can be drawn from the Consolidated Fund without authorization
from Parliament

CDS-2011
In the Union Budget of India, all revenues received by the Government and loans raised by it form part
of
(a) Consolidated Fund of India
(b) Contingency Fund of India
(c) Public Accounts
(d) Balance of Payments

Contingency Fund of India (Article 267 of the Constitution) *


Nature It is an imprest placed at the disposal of the President of India

Corpus Rs.500 crore

Purpose to facilitate Government to meet urgent unforeseen expenditure pending


authorization from Parliament

Post facto Post facto approval is obtained from Parliamentary for such unforeseen
approval expenditure

Recoupment After such post-facto approval, an equivalent amount is drawn from the
Consolidated Fund to recoup the Contingency Fund.

Public Account *
Which type of Moneys held by Government in trust are kept in the Public Account.
money kept in
For example :Provident Funds, Small Savings collections, income of
PA
Government set apart for expenditure on specific objects such as road
development, primary education, other Reserve/Special Funds etc

No need of Public Account funds do not belong to the Government and have to be
Parliamentary finally paid back to the persons and authorities who deposited them, do
authorization not require Parliamentary authorization for withdrawals.
Whatsapp No 7023213423 http://iasselfstudy.com/ 74

IAS Prelims 2015


With reference to the Union Government consider the following statements.
1. The Department of Revenue is responsible for the preparation of Union Budget that is presented to
the parliament
2. No amount can be withdrawn from the Consolidated Fund of India without the authorization of
Parliament of India.
3. All the disbursements made from Public Account also need the Authorization from the Parliament of
India
Which of the following statements given above is/are correct?
a) 1 and 2 only
b) 2 and 3 only
c) 2 only
d) 1, 2 and 3

Demands for Grants ** (Article 113)


Expenditure from Estimated expenditure from the Consolidated Fund of India included
the Consolidated in the Annual Financial Statement are submitted in the form of
Fund in the form of Demands for Grants.
DG

Lok sabha voting Demands for Grants are presented to the Lok Sabha along with the
Annual Financial Statement and required to be voted by the Lok-
Sabha

One Demand for Generally, one Demand for Grant is presented in respect of each
each Ministry Ministry or Department.

However, more than one Demand may be presented for aMinistry or


Department depending on the nature of expenditure

Demand for Union In regard to Union Territories without Legislature, a separate


Territories Demand is presented for each of the Union Territories

Appropriation Bill ** Article 114(3)


Under Article 114(3) of the Constitution, no amount can be withdrawn from the Consolidated
Fund without the enactment of such a law by Parliament.

After the Demands for Grants are voted by the Lok Sabha, Parliament's approval to the
withdrawal from the Consolidated Fund is sought through the Appropriation Bill.
Whatsapp No 7023213423 http://iasselfstudy.com/ 75

Finance Bill **
At the time of presentation of the Annual Financial Statement before Parliament, a Finance Bill
is also presented detailing the imposition, abolition, remission, alteration or regulation of taxes
proposed in the Budget.

Vote on account **
Vote on Account The whole process of the Budget requires a sufficiently long time.

The Lok Sabha is, therefore, empowered to make any grant in


advance in respect of the estimated expenditure for a part of the
new financial year pending completion of the procedure of the
voting on the Demands.

This is termed as 'Vote on Account'.

Purpose The purpose of the 'Vote on Account' is to keep the Government


functioning, pending voting of 'final supply'

Vote on Account not Because the Budget presentation is advanced to the first of
required in the February, 2017.
financial year 2017-
18

CDS -2009
Identify the correct sequence of passing a Budget in the Parliament
(a) Vote on Account - Finance Bill - Appropriation Bill - Discussion on Budget
(b) Finance Bill - Appropriation Bill - Discussion on Budget - Vote on Account
(c) Discussion on Budget - Vote on Account- Finance Bill-Appropriation Bill
(d) Discussion on Budget – Appropriation Bill - Finance Bill - Vote on Account

Outcome Budget *
From the fiscal year 2006-07, every Ministry presents a preliminary Outcome Budget to the
Ministry of Finance, which is responsible for compiling them.

The Outcome Budget is a progress card on what various Ministries and Departments have
done with the outlays in the previous annual budget.

It measures the development outcomes of all Government programs and whether the money
has been spent for the purpose it was sanctioned including the outcome of the fund usage.
Whatsapp No 7023213423 http://iasselfstudy.com/ 76

Guillotine *
Parliament, unfortunately, has very limited time for scrutinizing the expenditure demands of
all the Ministries.

So, once the prescribed period for the discussion on Demands for Grants is over, the Speaker
of Lok Sabha puts all the outstanding Demands for Grants, whether discussed or not, to the
vote of the House.

This process is popularly known as 'Guillotine'.

Cut Motions *
Motions for reduction to various Demands for Grants are made in the form of Cut Motions
seeking to reduce the sums sought by Government on grounds of economy or difference of
opinion on matters of policy or just in order to voice a grievance.

Revenue Budget **
Definition The Revenue Budget consists of the revenue receipts of the
Government and the expenditure met from these revenues

Revenue receipts Tax revenues: taxes and other duties levied by the Union

Non Tax revenues: interest and dividend on investments by the Govt,


fees and other receipts for services rendered by the Govt

Revenue Expenditure which does not result in creation of assets for the Govt,
expenditure is treated as revenue expenditure like normal running of Government
departments and for rendering of various services, making interest
payments on debt, meeting subsidies, grants in aid, etc.

Grants to the State All grants given to the State Governments/Union Territories and
Governments/Union other parties are also treated as revenue expenditure even though
Territories some of the grants may be used for creation of capital assets.

Effective Revenue Revenue Deficit - Grants for Creation of Capital Assets


Deficit (ERD)

CDS 2017
Which one of the following is not a component of Revenue Receipts of the Union Government?
(a) Corporate tax receipts
(b) Dividends and profits
(c) Disinvestment receipts
(d) Interest receipts
Whatsapp No 7023213423 http://iasselfstudy.com/ 77

Capital Budget **
Definition Capital receipts and capital payments together constitute the Capital Budget

Capital Loans raised by the Govt from the public (termed as market loans),
receipts
Borrowings by the Govt from the RBI and other parties through the sale of
Treasury Bills,

Loans received from foreign Govts and bodies,

Disinvestment receipts and

Recoveries of loans from State/Union Territory Govts and other parties

Capital Acquisition of assets like land, buildings, machinery, equipment etc.


payments
Investments in shares, etc., and

Loans and advances granted to the State/Union Territory Govts, Govt


companies, Corporations and other parties

IAS Prelims 2016


Which of the following is/are included in the capital budget of the Government of India?
1. Expenditure on acquisition of assets like roads, buildings, machinery, etc.
2. Loans received from foreign governments
3. Loans and advances granted to the States and Union Territories
Select the correct answer using the code given below.
(a) 1 only
(b) 2 and 3 only
(c) 1 and 3 only
(d) 1, 2 and 3

Economic Survey ***


Issued by Economic Division of Department of Economic Affairs, Ministry of Finance

About ES It highlights the economic trends in the country and facilitates a better
appreciation of the mobilization of resources and their allocation in the Budget.

It reviews the developments in the Indian economy over the previous 12


months, summarizes the performance on major development programmes,
and highlights the policy initiatives of the government and the prospects of the
economy in the short to medium term.

Where The Economic Survey is presented to both houses of Parliament in advance of


presented the Union Budget.

In 2017, Economic survey presented on 31-01-2017 and Union Budget on 01-


02-2017.
Whatsapp No 7023213423 http://iasselfstudy.com/ 78

Highlights of Economic Survey 2016-17 ***


Prepared by Arvind Subramanian, Chief Economic Adviser in the Ministry of finance

Headline inflation CPI based headline inflation remained under control for the third
successive financial year

Core inflation CPI based core inflation remained stable in the current fiscal year
averaging around 5 per cent.

Core inflation= headline inflation minus food and energy items which
are frequently subject to volatile prices

Major contributor pulses


of food inflation

Growth rate of 2015-16 7.6% actual


GDP
2016-17 7.1% estimated

2017-18 6.75 to 7.25% estimated

Driver of GDP Government final consumption expenditure is the major driver of GDP
growth growth in the current year

Growth rate of estimated to grow at 4.1 per cent in 2016-17 as opposed to 1.2 per
Agriculture sector cent in 2015-16;

the higher growth in agriculture sector is not surprising as the


monsoon rains were much better in the current year than the previous
two years.

Growth rate of estimated to moderate to 5.2 per cent in 2016-17 from 7.4 per cent in
industrial sector 2015-16

Growth rate of estimated to grow at 8.9 per cent in 2016-17, almost the same as in
the Service sector 2015-16

Universal Basic ES has advocated the concept of Universal Basic Income (UBI) as an
Income (UBI) alternative to the various social welfare schemes in an effort to reduce
poverty

Prerequisite for a JAM (Jan Dhan, Aadhar and Mobile) and Center- State cost sharing
successful UBI

Twin Balance Over-indebted corporates and


Sheet problem
Bad-loan-encumbered Public Sector Banks

Public Sector A decisive resolution is urgently needed before the TBS problem
Asset becomes a serious drag on growth.
Whatsapp No 7023213423 http://iasselfstudy.com/ 79

Rehabilitation a centralised Public Sector Asset Rehabilitation Agency to be


Agency considered that could take charge of the largest, most difficult cases,
and make politically tough decisions to reduce debt.

Two major passage of the Constitutional Amendment for GST, and


domestic policy
action to demonetize the two highest denomination notes
developments

India’s NPA ratio Higher than any other major emerging market, with the exception of
Russia

Labour migration inter-state labor mobility is significantly higher particularly for females
in India

Property tax Property tax potential is large and can generate additional revenue at
potential for city level
urban local
bodies (ULBs)

For Jobs creation- Apparel (cloth) and leather (shoe) sectors provide immense
pay attention to opportunities for creation of jobs for the weaker sections, especially for
labor-intensive women, and can become vehicles for broader social transformation in
sectors the country

India has an opportunity to push exports since rising wage levels in


China has resulted in China stabilizing or losing market share in these
products.

India is well positioned to take advantage of China’s deteriorating


competitiveness due to lower wage costs in most Indian states.

But space vacated by China is fast being taken over by Bangladesh and
Vietnam in case of apparels and Vietnam and Indonesia in case of
leather and footwear.

Economic Survey recommends reforms in labour and tax policies to


make the Apparel and Leather sector globally competitive

Redistributive Redistributive Resource Transfer or RRT to a state (from the Centre) is


Resource defined as gross devolution to the state adjusted for the respective
Transfers (RRT) state’s share in aggregate GDP.

Gross devolution to the states means share in central taxes, grants,


loans etc.

The top 10 recipients are: Sikkim, Arunachal Pradesh, Mizoram,


Nagaland, Manipur, Meghalaya, Tripura, Jammu and Kashmir, Himachal
Pradesh and Assam.

Annual per capita RRT flows for all North-Eastern States (except
Assam) and Jammu & Kashmir have exceeded the annual per-capita
Whatsapp No 7023213423 http://iasselfstudy.com/ 80

consumption expenditure that defines the all-India poverty lines,


especially the rural line

The Economic Survey suggests providing a part of the RRTs or to


redistribute the gains from use of natural resources as a Universal
Basic Income (UBI) directly to households in relevant states
which receive large RRT flows and are more reliant on natural resource
revenues.

Key Features of Budget 2017-2018 (February 01, 2017) ***


3 major reforms in First, presentation of Budget advanced to 1st February to enable the
Budget Ministries to operationalize all activities from the commencement of
the financial year.

Second, merger of Railways Budget with General Budget to bring


Railways to the Centre stage of Government’s Fiscal Policy and

Third, removal of plan and non plan classification of expenditure to


facilitate a holistic view of allocations for sectors and ministries.

Transformational Passage of the Constitution Amendment Bill for GST


reforms in last
Demonetisation of high denomination bank notes
year
Enactment of the Insolvency and Bankruptcy Code;

amendment to the RBI Act for inflation targeting;

enactment of the Aadhar bill for disbursement of financial subsidies


and benefits

Agenda for 2017- Transform, Energise and Clean India” – TEC India
18
TEC India seeks to

Transform the quality of governance and quality of life of our people;

Energise various sections of society, especially the youth and the


vulnerable, and enable them to unleash their true potential; and

Clean the country from the evils of corruption, black money and non-
transparent political funding

For Farmers committed to double the income in 5 years by March 2022;

Target for agricultural credit in 2017-18 has been fixed at a record


level of Rs. 10 lakh crores
Whatsapp No 7023213423 http://iasselfstudy.com/ 81

Coverage under Fasal Bima Yojana scheme will be increased from


30% of cropped area in 2016-17 to 40% in 2017-18 and 50% in
2018-19

Rural Population Aim to bring one crore households out of poverty and to make 50,000
Gram Panchayats poverty free by 2019, the 150th birth anniversary
of Gandhiji.

target to complete 1 crore houses by 2019 for the houseless and


those living in kutcha houses under Pradhan Mantri Awaas Yojana –
Gramin

100% village electrification by 1st May 2018 under Deen Dayal


Upadhyaya Gram Jyoti Yojna

MGNREGA allocation to be the highest ever at Rs. 48,000 crores in


2017-18

For youth National Testing Agency to be set-up as an autonomous and self-


sustained premier testing organisation to conduct all entrance
examinations for higher education institutions.

Poor and the Mahila Shakti Kendra will be set up with an allocation of Rs. 500
underprivileged crores in 14 lakh ICDS Anganwadi Centres.

This will provide one stop convergent support services for empowering
rural women with opportunities for skill development, employment,
digital literacy, health and nutrition

Under Maternity Benefit Scheme Rs. 6,000 each will be transferred


directly to the bank accounts of pregnant women who undergo
institutional delivery and vaccinate their children

Affordable housing to be given infrastructure status

The National Housing Bank (NHB) will refinance individual housing


loans of about Rs. 20,000 crore in 2017-18

Two new All India Institutes of Medical Sciences to be set up in


Jharkhand and Gujarat

Elimination target Government has prepared an action plan to eliminate Kala-Azar and
for various disease Filariasis by 2017, Leprosy by 2018, Measles by 2020 and
Tuberculosis by 2025 is also targeted

Railway safety For passenger safety, a Rashtriya Rail Sanraksha Kosh will be created
with a corpus of Rs. 1 lakh crores over a period of 5 years

Unmanned level crossings on Broad Gauge lines will be eliminated by


2020
Whatsapp No 7023213423 http://iasselfstudy.com/ 82

By 2019, all coaches of Indian Railways will be fitted with bio toilets

Villages By the end of 2017-18, high speed broadband connectivity on optical


fibre will be available in more than 1,50,000 gram panchayats, under
BharatNet.

A DigiGaon initiative will be launched to provide tele-medicine,


education and skills through digital technology

Financial Sector Foreign Investment Promotion Board to be abolished in 2017-18.

Propose to create an integrated public sector ‘oil major’ which will be


able to match the performance of international and domestic private
sector oil and gas companies

Fiscal Management For the first time, a consolidated Outcome Budget, covering all
Ministries and Departments, is being laid along with the other Budget
documents

fiscal deficit for 2017-18 is targeted at 3.2% of GDP and Government


remains committed to achieve 3% in the following year (2018-19)

The Revenue Deficit for next year is pegged at 1.9% , against 2%


mandated by the FRBM Act

Promoting digital No transaction above Rs. 3 lakh would be permitted in cash


economy
Aadhar Pay, a merchant version of Aadhar Enabled Payment System,
will be launched shortly

125 lakh people have adopted the BHIM app so far. The Government
will launch two new schemes to promote the usage of BHIM; these
are, Referral Bonus Scheme for individuals and a Cashback Scheme
for merchants

A Mission will be set up with a target of 2,500 crore digital


transactions for 2017-18 through UPI, USSD, Aadhar Pay, IMPS and
debit cards

Electoral Funding Maximum amount of cash donation, a political party can receive, will
be Rs. 2000/- from one person

Political parties will be entitled to receive donations by cheque or


digital mode from their donors.

Amendment to the Reserve Bank of India Act to enable the issuance


of electoral bonds

Personal Income- Existing rate of taxation for individual assesses between income of
Tax Rs. 2.5 lakhs to 5 lakhs reduced to 5% from the present rate of 10%
Whatsapp No 7023213423 http://iasselfstudy.com/ 83

Surcharge of 10% of tax payable on categories of individuals whose


annual taxable income is between Rs. 50 lakhs and Rs. 1 crore

Simple one-page form to be filed as Income Tax Return for the


category of individuals having taxable income upto Rs. 5 lakhs other
than business income

Public Service A Centralised Defence Travel System has been developed through
which travel tickets can be booked online by our soldiers and officers

To utilise the Head Post Offices as front offices for rendering passport
services

The Government e-market place which is now functional for


procurement of goods and services

Rupee Comes 19% from corporation tax


From
16% from Income Tax Direct Taxes

14% from Excise duty

10% from Service tax


Indirect Taxes

9% from Customs duty

10% Non-tax revenue

3% Non-debt capital receipts

19% from borrowing and other liabilities

Note: Direct tax collection is higher than indirect tax collection

Rupee Goes To 24% on States' shares of taxes and duties

18% on Interest payments

11% to Central Sector Scheme

10% Centrally Sponsored Scheme

10% on Subsidies

9% on Defence

5% on Finance commission Grants & other transfers

13% other expenditures

Note: As per recommendation of 14th Finance Commission, During


the award period of 14th Finance Commission (2015-2020), 42% of
the divisible pool of Union Taxes will be devolved to States.
Whatsapp No 7023213423 http://iasselfstudy.com/ 84

IAS Prelims 1991


The largest contribution to the central exchequer comes from
(a) Customs duties
(b) Income tax
(c) Central excise
(d) Estate duty

CDS-2015
Which one of the following is the major source of gross tax revenue (GTR) for the Government of
India?
(a) Income tax
(b) Corporation tax
(c) Customs duty
(d) Service tax

Fiscal Consolidation *
Meaning Fiscal Consolidation refers to the policies undertaken by Govt to reduce fiscal
deficits.

Focus Increase tax base and cut the non-productive expenditure


Whatsapp No 7023213423 http://iasselfstudy.com/ 85

Chapter-6

Money Market & Stock Market

Money
Function -It acts as a medium of exchange.

-Money is the most liquid of all assets in the sense that it is


universally acceptable and hence can be exchanged for other
commodities very easily.

Barter system Economic exchanges without the money are referred to as barter
exchanges. e.g. exchange of Milk with Rice

Opportunity cost It has an opportunity cost. If, instead of holding a certain cash
balance, you put the money in a fixed deposits you can earn interest
on that money.

Motive for holding People desire to hold money balance broadly from two motives.
money
The transaction motive i.e. to carry out transactions.

The Speculative Motive i.e. to earn monetary returns in future.

Demand deposits is Apart from currency notes and coins, the balance in savings or
also considered current account (i.e. Demand deposits) is also considered money
money since cheques drawn on these accounts are used to settle
transactions.

Promise from the Every currency note bears on its face a promise from the Governor of
Governor of RBI RBI that if someone produces the note to RBI, or any other
commercial bank, RBI will be responsible for giving the person
purchasing power equal to the value printed on the note. The same is
also true of coins.

Fiat money and Currency notes and coins are called fiat money. They do not have
Legal tenders intrinsic value like a gold or silver coin.

They are also called legal tenders as they cannot be refused by any
citizen of the country for settlement of any kind of transaction.

Cheques drawn on savings or current accounts, however, can be


refused by anyone as a mode of payment. Hence, demand deposits
are not legal tenders.
Whatsapp No 7023213423 http://iasselfstudy.com/ 86

SCRA-2013
Which one among the following best explains ‘Money’?
(a) It is a medium for producing capital goods
(b) It is a means for redistributing wealth among people
(c) It is a common denominator for measuring value
(d) It is a mechanism for resource allocation

Time Value of Money


Rupee today is more Money has time value as a Rupee today will have greater
valuable purchasing power than after a year.

Impact of Inflation A rising price level may erode the purchasing power of money.

Components of Reserve Money/ Monetary base of RBI (M0) *


Currency in Circulation + Bankers' Deposits with RBI + Other Deposits with RBI

Component of Money Supply (RBI) or Broad Money (M3) *


Currency with the Public
Demand Deposits with Banks
Time Deposits with Banks
Other Deposits with RBI

Narrow Money (M1) *


Currency with the Public + Demand Deposits with Banks + Other Deposits with RBI

SCRA-2012
Following are some components of money supply in India :
1. Currency with the public
2. Aggregate demand deposits with 'banks
3. Aggregate time deposits with banks
4. 'Other' deposits with the Reserve Bank of India
Which of the aforesaid items are components of narrow money (M1) in India?
(a) I , 2 and 3
(b) 2 and 4 only
(c) 1, 2 and 4
(d) 1 and 4 only
Whatsapp No 7023213423 http://iasselfstudy.com/ 87

IAS Prelims 2002


Consider the following:
1. Currency with the public
2. Demand deposits with banks
3. Time deposits with banks
Which of these are included in Broad Money (M3) in India?
(a) 1 and 2
(b) 1 and 3
(c) 2 and 3
(d) 1, 2 and 3

IAS Prelims 1997


The sum of which of the following constitutes Broad Money in India ?
I. Currency with the Public
II. Demand deposits with banks
III. Time deposits with banks
IV. Other deposits with RBI
Choose the correct answer using the codes given below :
(a) I and II
(b) I, II and III
(c) I, II, III and IV
(d) I, II and IV

Money in circulation **
Coins in 50 paise, 1, 2, 5 and 10 rupee
circulation

Notes in 5, 10, 20, 50, 100, 500 and 2000


circulation

Issuing Upto Re. 1 note and coins are issued by Govt of India (Ministry of finance).
authority
Rest are issued by RBI

Indian coinage As per Indian coinage act,


act
-Rupee coin 1 and above can be used to pay for any sum.

-50 paisa coin can be used to pay for any sum not exceeding 10 rupees.

Plastic notes Reserve Bank is considering issue of Rs. 10 banknotes in plastic.

Reasons for -for ensuring availability of clean notes


issue of plastic
-for strengthening the security features of bank notes and
note
-since the volumes involved and costs incurred in the printing, transport,
storage and removal of unfit/soiled notes is very high, the Reserve Bank is
evaluating ways to extend the life of bank notes – particularly in lower
denominations.
Whatsapp No 7023213423 http://iasselfstudy.com/ 88

CDS-2011
Consider-the following statements:
1. In India the minimum denomination coin acceptable for transaction is 50 paise.
2. Coins below 50 paise is not a legal tender for payment.
Which of the statements given above is/ are correct? I
(a) 1 only
(b) 2 only
(c) Both 1 and 2
(d) Neither 1 nor 2

Demonetization (8th Nov, 2016) ***


Reason The incidence of fake Indian currency notes in higher denomination has
increased.

For ordinary persons, the fake notes look similar to genuine notes, even
though no security feature has been copied.

The fake notes are used for antinational and illegal activities.

High denomination notes have been misused by terrorists and for


hoarding black money.

India remains a cash based economy hence the circulation of Fake Indian
Currency Notes continues to be a menace.

In order to contain the rising incidence of fake notes and black money, the
scheme to withdraw legal tender character of the old Bank Notes in the
denominations of 500 and 1000 (Specified Bank Notes) was introduced.

Exchange Upto 30-12-2016


facility

Ordinance The Specified Bank Notes (Cessation of Liabilities) Ordinance 2016


promulgated by the President of India dated December 30, 2016.

In terms of this ordinance, with effect from December 31, 2016, the
Specified Bank Notes shall cease to be the liabilities of the Reserve Bank
of India and shall cease to have the guarantee of the Central Government.

Grace A grace period has been provided during which the Specified Bank Notes
period can be deposited at five RBI Offices (Mumbai, New Delhi, Chennai,
Kolkata, and Nagpur)

Holding of A person can hold maximum 10 notes or


SBN after
25 notes for the purpose of study/ research/ numismatics.
the expiry
of grace Otherwise, Fine upto Rs.10,000 or 5 times the face value of the SBNs,
period whichever is higher.
Whatsapp No 7023213423 http://iasselfstudy.com/ 89

Sequence of Govt. initiatives for financial inclusion **


Opening of Jan Dhan accounts,

Giving a statutory basis for Aadhar,

Implementation of Directs Benefits Transfer,

Introduction of RuPay Cards,

Voluntary Disclosure Scheme for unaccounted money and

Demonetization of 500 and 1000 Rs. notes

Yet, nearly 95 per cent of India’s personal consumption expenditure transactions are cash-
based.

Lucky Grahak Yojana and Digi-Dhan Vyapar Yojana **


Launched by NITI Aayog

implementing National Payment Corporation of India (NPCI)


agency

Purpose The primary aim of these schemes is to incentivize digital


transactions so that electronic payments are adopted by all sections
of the society, especially the poor and the middle class.

to give cash awards to consumers and merchants who utilize digital


payment instruments for personal consumption expenditures

Period of scheme The scheme will become operational with the first draw on
25th December, 2016 (as a Christmas gift to the nation) leading up
to a Mega Draw on Babasaheb Ambedkar Jayanti on 14th April 2017

Lucky Grahak Yojana Daily reward of Rs 1000 to be given to 15,000 lucky Consumers for
a period of 100 days.
for Consumers
Weekly prizes worth Rs 1 lakh, Rs 10,000 and Rs. 5000

Eligible transactions transactions within the range of Rs 50 to Rs 3000 through UPI,


USSD, AEPS and RuPay Cards

Exclusion Private Credit Cards and Digital Wallets.

Digi-Dhan Vyapar Prizes for Merchants for all digital transactions conducted at
Yojana Merchant establishments
for Merchants
Weekly prizes worth Rs. 50,000, Rs 5,000 and Rs. 2,500
Whatsapp No 7023213423 http://iasselfstudy.com/ 90

Mega Draw on 14th a) 3 Mega Prizes for consumers worth Rs 1 cr, 50 lakh, 25 lakh for
of April digital transactions between 8thNovember, 2016 to 13th April,
– Ambedkar Jayanti 2017 to be announced on 14th April, 2017

b) 3 Mega Prizes for merchants worth Rs 50 lakhs, 25 lakh,


12 lakh for digital transactions between 8th November, 2016 to
13th April, 2017 to be announced on 14th

DigiDhan Mela To make Digital payments and a less cash society a mass
movement in India, the DigiDhan Mela will take place in 100
different cities across the country over the next 100 days until
March.

First 'Digi Dhan' mela (fair) organised in Gurugram, Haryana

Estimated estimated expenditure on the first phase of the scheme (up to


expenditure of Govt 14th April 2017) is likely to be 340 Crores

Bitcoin
About bitcoin Bitcoin is a web based crypto-currency.

Peer-to-peer It is the first decentralized peer-to-peer payment network that is


payment powered by its users with no central authority or middlemen.
network

Digital money Bitcoin is pretty much like cash for the Internet i.e. a completely
digital money

Triple entry Bitcoin can also be seen as the most prominent triple entry book
book keeping keeping system in existence
system

From a user Bitcoin is nothing more than a mobile app or computer program that
perspective provides a personal Bitcoin wallet and allows a user to send and
receive bitcoins with them.

Digital The authenticity of each transaction is protected by digital signatures.


signatures

24*7 use It is possible to send and receive bitcoins anywhere in the world at
any time.

No bank holidays. No borders. No bureaucracy


Whatsapp No 7023213423 http://iasselfstudy.com/ 91

ES-2014
The term 'bitcoin', frequently in the news, is mentioned in the context of :
(a) hologram stickers
(b) technology for improving the security features of paper currency
(c) peer to peer digital currency
(d) Near Field Communication Technology

IAS Prelims 2016


With reference to ‘Bitcoins’, sometimes seen in the news, which of the following statements is/are
correct?
1. Bitcoins are tracked by the Central Banks of the countries.
2. Anyone with a Bitcoin address can send and receive Bitcoins from anyone else with a Bitcoin
address.
3. Online payments can be sent without either side knowing the identity of the other.
Select the correct answer using the code given below.
(a) 1 and 2 only
(b) 2 and 3 only
(c) 3 only
(d) 1, 2 and 3

CDS 2017
Which of the following statements about Bitcoin is / are correct?
1. It is a decentralized virtual currency.
2 . It is generated through complex computer software systems.
3. The Reserve Bank of India recognized it as a legal tender in January 201 6.
Select the correct answer using the code given below.
(a) 1 only
(b) 1 and 2 only
(c) 2 and 3 only
(d) 1, 2 and 3

Money laundering
Converting the black money into white money

HAWALA
Hawala is an illegal method of remittance of money.

Hawala does not involve physical movement of cash

IAS Prelims 1996


Hawala transactions relate to payments
(a) received in rupees against overseas currencies and vice versa without going through the official
channels
(b) received for sale/transfer of shares without going through the established stock exchanges
(c) received as commission for services rendered to overseas investors/buyers/ sellers in assisting
them to get over the red tape and/or in getting preferential treatment
(d) made to political parties or to individuals for meeting election expenses
Whatsapp No 7023213423 http://iasselfstudy.com/ 92

Money market v/s Capital/Security market *


Money Market Capital/Security Market

Controlled by RBI. Controlled by SEBI.

Sources which meet short term Sources which meet medium and long term
requirements of money are the requirements of money are the constituents of the
constituents of the money market capital market. It has two type Primary market
and secondary market

Primary market v/s Secondary market *


Primary market Secondary market

It is a market of new issue. Since securities It provides market ability for the securities
are new hence they create capital formation coming for sale in stock market. There is no
profit for company. When price of securities
is increased, capitalization of company
increased.

Bull and Bear Market *


Bull market A bull market is one where prices are rising

Investor A bull investor has a very optimistic view of the market. He


Optimistic aggressively buys and sells stocks quickly.

Bear market A bear market is one where prices are falling.

Investor A bear investor is pessimistic about the market and may make more
Pessimistic conservative stock choices.

IAS PRELIMS 2010 & CDS-2012


In the parlance of financial investments, the term 'bear' denotes
(a.) An investor who feels that the price of a particular security is going to fall
(b.) An investor who expects the price of particular shares to rise
(c.) A shareholder or a bondholder who has' an interest in a company, financial or otherwise
(d.) Any lender, whether by making a loan or buying a bond
Whatsapp No 7023213423 http://iasselfstudy.com/ 93

IPO- Initial Public Offering *


Is a stock market launch of securities of a company to Public

Bonus Issue *
free shares given to current shareholders of the company,

Underwriting
Definition The arrangement wherein underwriter undertakes the risk associated
with the issue of new securities to public.

Who is underwriter Underwriters may be individual, Banks or Financial institutions

Example XYZ ltd issued a IPO of 2,00,000 shares.

XYZ limited entered into underwriting agreement with SBI for


underwriting of shares.

Offer from public came for 1,50,000 shares, now SBI will have to
purchase unsubscribed 50,000 shares.

The Securities and Exchange Board of India (SEBI) or Market Watchdog *


Establishment SEBI was enacted on April 12, 1992 in accordance with the
provisions of the Securities and Exchange Board of India Act, 1992.

Basic functions of - to protect the interests of investors in securities market


the SEBI
- to promote the development of securities market and

- to regulate the securities market

HQ Mumbai.

Chairman Ajay Tyagi

IAS Prelims 1995


To prevent recurrence of scams in Indian Capital Market, the Government of India has assigned
regulatory powers to
(a) SEBI
(b) RBI
(c) SBI
(d) ICICI
Whatsapp No 7023213423 http://iasselfstudy.com/ 94

Merger of Forward Markets Commission (FMC) with SEBI


Regulating act SEBI was enacted on April 12, 1992 in accordance with the
provisions of the Securities and Exchange Board of India Act, 1992
for regulating the securities markets.

Forward Markets Commission, on the other hand, had been


regulating commodities markets since 1953. It is overseen by
the Department of Economic Affair (Ministry of Finance).

Reason for merger The Government opted for merger of the two regulators since both
commodity derivatives and security derivatives have similar
economic purposes of hedging, efficient price discovery and risk
management and have close resemblance in so far as trade practices
and mechanisms are concerned.

FMC's merger with SEBI is aimed at streamlining the regulations and


curb wild speculations in the commodities market, while facilitating
further growth.

IAS Prelims 2010


In India, which of the following is regulated by the Forward Market Commission?
(a.) Currency Futures Trading
(b.) Commodities Futures Trading
(c.) Equity Futures Trading
(d.) Both Commodities Futures and Financial Futures trading

Masala Bonds
Nature Masala bonds are rupee denominated bonds issued to overseas buyers

First issue IFC issued the first Masala bond listed on the London Stock Exchange. The
bonds worth over Rs 1000 crore were issued to raise funds for infrastructure
projects.

Minimum minimum maturity period for such bonds is 3 years


maturity
period

NDA-2-2016
As per the RBI guidelines, which one of the following is the minimum tenure of masala bonds that an
Indian company can issue offshore?
(a) 5 years
(b) 4 years
(c) 3 years
(d) 2 years
Whatsapp No 7023213423 http://iasselfstudy.com/ 95

IAS Prelims 2016


With reference to `IFC Masala Bonds’, sometimes seen in the news, which of the statements given
below is/are correct?
1. The International Finance Corporation, which offers these bonds, is an arm of the World Bank.
2. They are the rupee-denominated bonds and are a source of debt financing for the public and private
sector.
Select the correct answer using the code given below.
(a) 1 only
(b) 2 only
(c) Both 1 and 2
(d) Neither 1 nor 2

Major stock exchanges of world *


Country Stock Exchange Indices

India Bombay Stock Exchange. SENSEX (30 companies)

India National Stock Exchange of NIFTY (50 companies)


India.

Japan Tokyo stock exchange NIKKEI

US New York Stock Exchange NYSE Composite, Dow Zones, S&P 500
(NYSE)

US NASDAQ NASDAQ Composite, Dow Zones, S&P 500

UK London stock exchange FTSE 100

China Shanghai stock exchange SSE Composite/SHCOMP

China Hong Kong stock market Hang Seng

China Shenzhen stock exchange SZSE component

Singapore Singapore stock market Straits Times STI

Pakistan Karachi Stock Exchange KSE 100

Brazil Sao Paulo stock exchange IBOVESPA

Korea Korea Stock exchange KOSPI

Germany German stock exchange DAX

France French stock market CAC 40

Russia Moscow Stock Exchange RTS and MICEX

South Johannesburg Stock Exchange FTSE/JSE


Africa
Whatsapp No 7023213423 http://iasselfstudy.com/ 96

IAS Prelims 2009


Which of the following pairs given is/are not correctly matched?
Country Stock Exchange
1. Japan– NIKKEI
2. Singapore– SHCOMP
3. UK – FTSE
4. USA – NASDAQ

CDS-2012
Which of the following statements is/ are correct?
1. NIFTY is based upon 50 firms in India.
2. NIFTY is governed and regulated by the Reserve Bank of India.
3. NIFTY does not trade in mutual funds.
Select the correct answer using the code given below
Code':
(a) 1 only (b) 2 (c) 3 only (d) 1 and 3

CDS-2009
Which one of the following statements is not correct ?
(a) The National Association of Securities Dealers Automated Quotations known as NASDAQ, is an
American stock exchange
(b) Nikkei is the stock market index for the Tokyo Stock Exchange
(c) S & P CNX Nifty is the index for 50 large companies on the Bombay Stock Exchange
(d) Hang Seng Indexes record daily changes of the largest companies of the Hong Kong stock market

IAS Prelims 1998


Which of the following pairs are correctly matched ?
I. Dow Jones : New York
II. Hang Seng : Seoul
III. FTSE 100 : London
Select the correct answer using the codes given below:
(a) I, II and III (b) II and III (c) I and II (d) I and III

IAS Prelims 2000


A rise in ‘SENSEX’ means
(a) a rise in prices of shares of all companies registered with Bombay Stock Exchang
(b) a rise in prices of shares of all companies registered with National Stock Exchange
(c) an overall rise in prices of shares of group of companies registered with Bombay Stock Exchange
(d) a rise in prices of shares of all companies belonging to a group of companies registered with Bombay
Stock Exchange

Participatory notes
Participatory Notes commonly known as P-Notes or PNs are instruments issued by registered
foreign institutional investors (FII) to overseas investors, who wish to invest in the Indian stock
markets without registering themselves with the market regulator, SEBI

IAS Prelims 2007


Participatory Notes (PNs) are associated with which one of the following ?
(a) Consolidated Fund of India (b) Foreign Institutional Investors
(c) United Nations Development Programme (d) Kyoto Protocol
Whatsapp No 7023213423 http://iasselfstudy.com/ 97

Chapter 7

Micro Economy

Adam Smith is regarded as the founding father of modern economics.

Micro and Macro economics


Micro economics Micro means small.

Micro economics is the study of particular markets, and segments of the


economy.

Macro economics Macro means large.

Macroeconomics is the study of the whole economy.

It looks at ‘aggregate’ variables, such as aggregate demand, national


output and inflation.

Central Problems of an Economy


What is produced and Whether to produce more agricultural goods or industrial
in what quantities? products and services.

How are these goods How much of which of the resources to use. Whether to use more
produced? labour or more machines.

For whom are these How should the produce of the economy be distributed among
goods produced? the individuals in the economy?

Thus, every economy faces the problem of allocating the scarce resources to the production of
different possible goods and services and of distributing the produced goods and services
among the individuals within the economy.
Whatsapp No 7023213423 http://iasselfstudy.com/ 98

How does a firm decide how much to produce?


Maximum Profit A firm is a profit maximizer. So, the amount that a firm produces
and sells in the market is that which maximizes its profit.

Equilibrium Where market supply equals market demand

equilibrium price at which equilibrium is reached

equilibrium quantity quantity bought and sold at equilibrium price

Price change Whenever market supply is not equal to market demand, there will
be a tendency for the price to change

price will rise If demand increases and supply remains the same

price will fall If supply increases and demand remains the same

CDS-2014
Which of the following statements is / are true ?
1. If increase in demand and supply are of equal magnitude, the price will remain unchanged, but the
equilibrium quantity will increase.
2. If increase in demand is of greater magnitude than increase in supply, both equilibrium price and
equilibrium quantity will increase.
3. If increase in supply is of greater magnitude than increase in demand, equilibrium price will fall but
equilibrium quantity will increase.
Select the correct answer using the code given below :
(a) 1 only
(b) 1 and 2 only
(c) 2 and 3 only
(d) 1, 2 and 3

Production possibility set


Resources of an economy as a whole are always limited.

The collection of all possible combinations of the goods and services that can be produced from
available amount of resources is called the production possibility set of the economy.
Whatsapp No 7023213423 http://iasselfstudy.com/ 99

Type of Economy **
The Centrally Where the government or the central authority plans all the important
Planned Economy activities in the economy.

The Where all the central problems are solved with the help of price
Market/capitalist mechanism.
Economy
The basis of price mechanism is that every commodity/service has a
price which is determined with the help of supply and demand.

If the buyers demand more of a certain good, the price of that good
will rise.

This will send a signal to the producer of that good that the society
wants more of that good and the producers of that good, are likely to
increase their production.

In this way, prices of goods and services send important information


to all the individuals across the market and help achieve coordination
in a market system.

Mixed economies In reality, all economies are mixed economies where some important
decisions are taken by the government and the economic activities are
by and large conducted through the market.

CDS-2009
Mixed economy means an economy where
(a) both agriculture and industry are equally promoted by the State
(b) there is co-existence of public sector along with private sector
(c) there is importance of small scale industries along with heavy industries
(d) economy is controlled by military as well as civilian rulers

ES-2011
The concept of 'Mixed Economy' relates to
(a) the coexistence of rural sector and urban sector
(b) the coexistence of public sector and private sector
(c) the coexistence of small-scale sector and large-scale sector
(d) the coexistence of service sector and manufacture sector
Whatsapp No 7023213423 http://iasselfstudy.com/ 100

Law of Demand *
a consumer’s demand for a good is inversely related to the price of the good.

e.g. if price is high, demand will be low. Demand curve can be drawn as follows:

Price

Quantity

Elasticity of Demand
Demand for food does not change much even if food prices go up.

On the other hand, demand for luxuries can be very responsive to price changes.

In general, demand for a necessity is likely to be price inelastic while demand for a luxury good
is likely to be price elastic.
Whatsapp No 7023213423 http://iasselfstudy.com/ 101

Substitutes and Complements *


Substitutes Goods like tea and coffee are not consumed together. They are
substitutes for each other.

Since tea is a substitute for coffee, if the price of coffee increases, the
consumers can shift to tea.

On the other hand, if the price of coffee decreases, the consumption of


tea is likely to go down.

Thus demand for a good usually moves in the direction of the price of
its substitutes.

Complements Goods which are consumed together are called complementary goods.
E.g. tea and sugar, pen and ink.

Since tea and sugar are used together, an increase in the price of
sugar is likely to decrease the demand for tea and

a decrease in the price of sugar is likely to increase the demand for


tea.

Hence, the demand for a good moves in the opposite direction of the
price of its complementary goods.

CDS-2015
When two goods are interchangeable, they are
(a) Perfect substitutes
(b) Perfect complements
(c) Giffen goods
(d) Veblen goods

Cartel
A cartel is an agreement between competing firm to control price or exclude entry of a new
competitor in a market.

Break even point *


is the point at which cost and revenue is equal i.e. there is no loss or profit.

Sales above the BEP will generate profit to the firm.

Sales below BEP will lead to loss to the firm.


Whatsapp No 7023213423 http://iasselfstudy.com/ 102

Shut down point *


When firm is incurring losses but can cover its variable cost from its revenue then it should
continue the production as fixed cost will always be incurred whether to continue the production
or not.

When revenue received cannot even cover the variable costs of production. Then At this point,
firm should shut down the production.

Marginal cost *
Means additional cost incurred on additional unit of production. It is dependent on the variable
cost only as fixed cost remains constant on every additional production.

Output total cost marginal cost


1 60 0
2 80 20
3 105 15

SCRA-2013
In the short run marginal cost
(a) Equal marginal variable cost
(b) is independent of fixed cost
(c) equals marginal variable cost and is independent of fixed cost
(d) equals the first derivative of the total cost

The law of diminishing marginal product


It says that if we keep on increasing the input, with other inputs fixed, eventually a point will
be reached after which the resulting addition to output (i.e., marginal product of that input)
will start falling.

Monopoly & Oligopoly


Monopoly When there is a single producer of a particular commodity; no other
commodity works as a substitute for this commodity

Oligopoly When market of a particular commodity consists of more than one seller but
the number of sellers is few

Duopoly The special case of oligopoly where there are exactly two sellers
Whatsapp No 7023213423 http://iasselfstudy.com/ 103

Economic wealth of a country does not depend on mere possession of


resources
In fact the resource rich Africa and Latin America have some of the poorest countries in the
world, whereas many prosperous countries have scarcely any natural wealth.

the point is how these resources are used in generating a flow of production.

Final Good
An item that is meant for final use and will not pass through any more stages of production or
transformations is called a final good.

Intermediate goods
Which are used as raw material or inputs for production of other commodities. These are not
final goods.

Why are we measure final goods only?


Value of the final goods already includes the value of the intermediate goods that have entered
into their production as inputs. Counting them separately will lead to the error of double
counting.

Gross investment
Part of our final output that comprises of capital goods constitutes gross investment of an
economy

Net Investment = Gross investment – Depreciation

Four factors of production


Joint efforts Production of goods and services is a result of joint
efforts of four factors of production.

1. Land (i.e. natural resources) remuneration for which is called rent

2. Labour remuneration for which is called wages

3. Capital remuneration for which is called interest

4. Entrepreneurship remuneration of which is profit


Whatsapp No 7023213423 http://iasselfstudy.com/ 104

Chapter -8

International Trade & Agreements

Foreign exchange market


The market in which national currencies are traded for one another.

Exchange rate
The price of one currency in terms of the other is known as the exchange rate. E.g.
1US$=INR68

The real exchange rate


Meaning is the relative price of foreign goods in terms of domestic goods.

It measures the international competitiveness of a country in


international trade.

If real exchange rate the two countries are said to be in purchasing power parity. This
is equal to one means that goods cost the same in two countries

If the real exchange this means that goods abroad have become more expensive than
rises above one goods at home.

Exchange rate regime *


Fixed exchange rate When exchange rate is determined by Govt (e.g. China)

Floating/Flexible When exchange rate is determined by demand and supply in market


(e.g. US)

Managed When demand and supply force in market are independent and Govt.
also give direction.(e.g. India)
Whatsapp No 7023213423 http://iasselfstudy.com/ 105

CDS-2015
Under flexible exchange rate system, the exchange rate is determined
(a) predominantly mechanism by market
(b) by the Central Bank
(c) as a weighted index of a group of Currencies
(d) by the World Trade Organization

Purchasing Power Parity (PPP) theory *


According to the theory, as long as there are no barriers to trade like tariffs (taxes on trade)
and quotas (quantitative limits on imports), exchange rates should eventually adjust so that
the same product costs the same whether measured in rupees in India, or dollars in the US,
yen in Japan and so on, except for differences in transportation.

According to the PPP theory, differences in the domestic inflation and foreign inflation are a
major cause of adjustment in exchange rates.

If one country has higher inflation than another, its exchange rate should be depreciating.

Open economy v/s Closed economy *


Open economy Closed economy

Which has economic relations with the rest Which has no economic relations with the
of the world. rest of the world.

Most countries of the world are open Example of closed economy is difficult to
economy. find in present day world. (e.g. North Korea)

In an open economy, the demand for In a closed economy, there are three
domestic goods is equal to the domestic sources of demand for domestic goods –
demand for goods (consumption, investment Consumption, government spending, and
and government spending) plus exports domestic investment.
minus imports i.e. In an open economy,
exports constitute an additional source of
demand for domestic goods and services.

An essential difference between a closed economy and an open economy is that while in a
closed economy saving and investment must always be equal, in an open economy, they can
differ.

IAS Prelims 2000


In an open economy, the national income (Y) of the economy is: (C, I, G, X, M stand for Consumption,
Investment, Govt. Expenditure, total exports and total imports respectively.)
(a) Y = C + I + G + X
(b) Y = C + I + G X+ M
(c) Y = C + I + G + (X -M)
(d) Y = C + I+G + X -M
Whatsapp No 7023213423 http://iasselfstudy.com/ 106

IAS Prelims 2011


A “closed economy’’ is an economy in which ?
(a) The money supply is fully controlled.
(b) Deficit financing takes place.
(c) Only exports take place.
(d) Neither exports nor imports take place.

India’s Top Export Destinations- Ranks *


1. USA
2. UAE
3. Hong Kong
4. China
5. UK

IAS Prelims 1992 & 2002


India has the maximum volume of foreign trade with
(a) USA
(b) Japan
(c) Germany
(d) UAE

NDA-2015
Which one of the following continents accounts for the maximum share in exports from India?
(a) Asia
(b) Europe
(c) Africa
(d) North America

India’s Top Import Sources- Ranks *


1. China
2. USA
3. Saudi Arabia
4. UAE
5. Switzerland
Whatsapp No 7023213423 http://iasselfstudy.com/ 107

India’s Top Export Items- Ranks *


1. Petroleum Products
2. Pearls, prec. & semi-prec. Stones
3. Gold, other prec. metal jewellery

India’s Top Import Items- Ranks *


1. Petroleum: crude
2. Gold
3. Pearl, prec., semi-prec. Stones

ES-2011
In India, which item of imports is the highest in terms of value?
(a) Gold, gems and jewellery
(b) IT imports
(c) Petroleum and petroleum products
(d) Machinery and equipment

Composition of India’s External Debts **


1. Multilateral
2. Bilateral
3. IMF
4. Export Credit
5. Commercial Borrowings
6. NRI Deposits
7. Rupee Debt
8. Total Long Term Debt (1 to 7)
9. Short Term Debt
10. Total External Debt (8+9)

Status report on India’s External Debts is prepared by Department of Economic Affair


(MOF)

Credit rating agencies *


India’s sovereign debt is usually rated by 6 Sovereign Credit Rating Agencies (SCRAs) viz.

Moody’s Investor Services,


Standard and Poor’s (S&P),
Dominion Bond Rating Service (DBRS),
Fitch Ratings,
Japanese Credit Rating Agency (JCRA) and
Rating and Investment Information (R&I).
Whatsapp No 7023213423 http://iasselfstudy.com/ 108

Dumping *
Dumping is a kind of predatory pricing, especially in the context of international trade.

It occurs when manufacturers export a product to another country at a price either below the
price charged in its home market or below its cost of production

Anti dumping *
Anti dumping is a measure to rectify the situation arising out of the dumping of goods.

Thus, the purpose of anti dumping duty is to rectify the trade distortive effect of dumping and
re-establish fair trade.

Arbitrage
simultaneous buying and selling of securities, currency, or commodities in different markets in
order to take advantage of prices difference for the same asset.

World Largest Economy-Gross domestic product 2015 (World Bank report)*


Ranking Economy GDP (millions of US dollars)

1 United States 18,036,648

2 China 11,007,720

3 Japan 4,123,257

4 Germany 3,363,446

5 United Kingdom 2,858,003

6 France 2,418,836

7 India 2,095,398
Whatsapp No 7023213423 http://iasselfstudy.com/ 109

Chabahar port development agreement with Iran **


Location of located in the Sistan-Baluchistan Province on Iran's south-eastern coast.
Chahbahar port
It lies outside the Persian Gulf and is easily accessible from India's
western coast

Benefit to India It will provide India an alternative and reliable access route into
Afghanistan utilizing India’s earlier investment in Zaranj-Delaram road
built in Afghanistan, and

a reliable and more direct sea-road access route into Central Asian
Region, and

Alternative to North South Corridor (Access to Russia and North Baltic


countries)

India’s Port Ports on the Western Coast of India like Kandla (550 NM) and Mumbai /
JNPT (786 NM) will get the maximum benefit from this agreement.

CDS 2017
India signed an agreement in 2016 to develop a strategic port in one of its neighbouring countries.
What is the name of the port?
(a) Chabahar
(b) Gwadar
(c) Hambantota
(d) Mongla
Whatsapp No 7023213423 http://iasselfstudy.com/ 110

The International North–South Transport Corridor (INSTC) *


About INSTC INSTC is a multi-modal transportation established in 12 SEP 2000 in
St. Petersburg, by Iran, Russia and India for the purpose of
promoting transportation cooperation among the Member States.

Detail of corridor This corridor connects India Ocean and Persian Gulf to the Caspian
Sea via IRAN, then is connected to St. Petersburg and North
European via Russian Federation.

Founder Members Iran, Russia and India


(3)

Other members Azerbaijan, Armenia, Kazakhstan, Kyrgyz Republic (Kyrgyzstan),


(10) Tajikistan, Turkey, Ukraine, Belarus, Oman, Syria,

Observer Bulgaria
Whatsapp No 7023213423 http://iasselfstudy.com/ 111

China Pakistan Economic Corridor (CPEC) **


About CPEC It will link the Gwadar city in South Western Pakistan to China’s
North Western region Xinjiang through a vast network of highways
and railways.

Benefit to China CPEC will cut short the trade route for China’s oil imports and also
open up new strategic gateway for China to tap into African, West
Asian, South Asian and European market

India’s objection Because it crosses through the Pakistan occupied part of Kashmir.

The Golden Crescent (also called the silk route of drugs)


Location The Golden Crescent is a mountainous area of Iran, Afghanistan and
Pakistan where opium has been grown for hundreds of years

Why in news It is the Route through which drugs are making their way into Punjab
Whatsapp No 7023213423 http://iasselfstudy.com/ 112

BBIN (Bangladesh-Bhutan-India-Nepal) Motor Vehicle Agreement *


Why BBIN As SAARC Motor Vehicle Agreement could not be signed due to
reservations of Pakistan.

So it was considered appropriate that a sub-regional Motor Vehicle


Agreement among Bangladesh, Bhutan, India and Nepal (BBIN) may
be pursued.

Benefit BBIN countries will be benefited by mutual cross border movement of


passenger and goods for overall economic development of the region.

Cost sharing Each Party will bear its own costs arising from implementation of this
agreement.
Whatsapp No 7023213423 http://iasselfstudy.com/ 113

OBOR (One Belt, One Road initiative)


About OBOR It refers to the New Silk Road Economic Belt, which will link China with
Europe through Central and Western Asia, and the 21st Century
Maritime Silk Road, which will connect China with Southeast Asian
countries, Africa and Europe.

Started by China

Roadmap Neither the belt nor the road follows any clear line geographically
speaking; they serve more as a roadmap for how China wants to further
integrate itself into the world economy and strengthen its influence in
these regions

IAS Prelims 2016


‘Belt and Road Initiative’ is sometimes mentioned in the news in the context of the affairs of (a) African
Union (b) Brazil (c) European Union (d) China

China-Market Economy Status *


History When China joined the WTO on December 11, 2001, it was written into the
agreement that member states could treat China as a non-market economy for
15 years.

This allowed them to impose heavy anti-dumping duties on the basis that
China's low prices did not reflect market reality.

Present Now after 15 years China are demanding that they should be recognized as a
situation 'market economy' but The US, EU and Japan are defending their economies
against dumping

India’s India is also “non-committal” on according MES to China.


position
The intention will be to ensure India’s manufacturing sector is not hit by
unfairly priced Chinese goods.
Whatsapp No 7023213423 http://iasselfstudy.com/ 114

India-Myanmar-Thailand Trilateral Highway (IMT) *


Purpose to improve connectivity with South East Asia by road

Project detail The Trilateral Highway starts from Moreh (Manipur) in India up to Mae
Sot in Thailand through Myanmar.

Extension of Government of India is exploring the possibility of extending the India-


Highway Myanmar-Thailand (IMT) highway to Cambodia, Lao PDR (Laos) and
Vietnam.
Whatsapp No 7023213423 http://iasselfstudy.com/ 115

Kaladan Multi Modal Transit Transport Project *


Purpose It will provide an alternate access route to the North-Eastern region
of India and contribute towards the region's economic development.

Project detail The Kaladan Multi Modal Transit Transport (through Sea, Road &
Inland waterways) Project was jointly identified by the India and
Myanmar to create a multi-modal mode of transport for shipment of
cargo from the eastern ports of India to Myanmar as well as to the
North-Eastern part of India through Myanmar.

This project, which will connect Sittwe Port in Myanmar to the India-
Myanmar border, is expected to contribute to the economic
development of the North-Eastern States of India, by opening up the
sea route for the products.

It also provides a strategic link to the North-East, thereby reducing


pressure on the Siliguri Corridor.

Being a key connectivity project, it will promote economic,


commercial and strategic links between India and Myanmar.

India's grant Since the project is of political and strategic significance, it was
decided to execute it through India's grant assistance to Myanmar.

Administered by is being administered by the Ministry of External Affairs (MEA)


Whatsapp No 7023213423 http://iasselfstudy.com/ 116

TAPI Gas Pipeline Project *


Countries Turkmenistan-Afghanistan-Pakistan-India
involved

Entry in India Pipeline will enter India at Fazilka, Punjab

Benefit 1814 km long TAPI pipeline project is envisaged to supply 38 MMSCMD


of natural gas to India

ES-2013
TAPI gas pipeline is meant to go through :
(a) Turkey-Azerbaijan-Pakistan -Iran
(b) Turkmenistan-Afghanistan-Pakistan-lndia
(c) Tajikistan-Afghanistan-Pakistan-lran
(d ) Turkmenistan- Azerbaijan-Pakistan-lndia

ES-2016
‘TAPI’ gas pipeline, sometimes mentioned in the news, is planned to be laid in:
(a) Turkey-Afghanistan-Pakistan-India
(b) Turkmenistan-Ajarbaijan-Pakistan-Iran
(c) Turkey-Ajarbaijan-Pakistan- Iran
(d) Turkmenistan- Afghanistan -Pakistan- India
Whatsapp No 7023213423 http://iasselfstudy.com/ 117

India’s decision to join Ashgabat Agreement *


Objective of to facilitate the transport of goods between Central Asia and the Persian
the agreement Gulf. It seeks to create an international transport and transit corridor

Members Oman, Iran, Turkmenistan and Uzbekistan as founding members.

Kazakhstan has also joined this arrangement subsequently

Depository State (Turkmenistan).

Pakistan also decided to join the Ashgabat Agreement

Benefit to Accession to the Agreement would enable India to utilise this existing
India transport and transit corridor to facilitate trade and commercial
interaction with the Eurasian region.

Further, this would synchronise with our efforts to implement the


International North South Transport Corridor (INSTC) for enhanced
connectivity.

India’s Status India would become party to the Agreement upon consent of the
founding members.
Whatsapp No 7023213423 http://iasselfstudy.com/ 118

NAFTA
The North American Free Trade Agreement (NAFTA) is a comprehensive trade agreement
that sets the rules of trade and investment between Canada, the United States, and
Mexico.

CDS-2016
The members of NAFTA are,
(a) USA, Canada and Mexico
(b) USA, Canada and India
(c) USA, Canada and Japan
(d) USA, UK and India

The Bay of Bengal Initiative for Multi-Sectoral Technical and Economic


Cooperation (BIMSTEC) *
About BIMSTEC BIMSTEC is a regional organization comprising 7 Member States lying
in the littoral and adjacent areas of the Bay of Bengal constituting a
contiguous regional unity.

BIMSTEC is a sector-driven cooperative organization

Establishment in June 1997 through the Bangkok Declaration.

Member States 5 from South Asia-Bangladesh, Bhutan, India, Nepal & Sri Lanka 2
(7) from Southeast Asia-Myanmar and Thailand

Founding Initially, it was 'BIST-EC' (Bangladesh, India, Sri Lanka and Thailand
member (BIST- Economic Cooperation).
EC)

'BIMST-EC Following the inclusion of Myanmar the Group was renamed 'BIMST-
EC' (Bangladesh, India, Myanmar, Sri Lanka and Thailand Economic
Cooperation).

Present status With the admission of Nepal and Bhutan in 2004, the name of the
grouping was changed to 'Bay of Bengal Initiative for Multi-Sectoral
Technical and Economic Cooperation' (BIMSTEC).

Advantages It constitutes a bridge between South and South East Asia.

It has also established a platform for intra-regional cooperation


between SAARC and ASEAN members.

BIMSTEC region The BIMSTEC region is home to around 1.5 billion people which
constitute around 22% of the global population with a combined gross
domestic product (GDP) of 2.7 trillion economy.
Whatsapp No 7023213423 http://iasselfstudy.com/ 119

In the last five years, BIMSTEC Member States have been able to
sustain an average 6.5% economic growth trajectory despite global
financial meltdown.

CDS-2014
Consider the following statements on Bay of Bengal Initiative for Multi sectoral Technical and
Economic Cooperation (BIMSTEC)
1. BIMSTEC has seven members Bangladesh, Bhutan, India, Myanmar, Nepal, Sri Lanka and
Thailand.
2. BIMSTEC provides a link between South Asia and South East Asia by way of economic
cooperation and linkages in identified areas of cooperation.
3. BIMSTEC was rechristened as BISTEC in the year 2014.
Select the correct answer using the code given below:
(a) 1 only
(b) 2 only
(c) 1 and 2 only
(d) 1, 2 and 3

CAPF-2010
Consider the following statements regarding the Bay of Bengal Initiative for Multi Sectoral Technical
and Economic Cooperation (BIMSTEC) :
1. It is an international organization involving a group of countries in South Asia and South East
Asia.
2. All countries with whom India shares international border are members of the organisation:
3. Bangladesh, India, Sri Lanka and Thailand were the original members of the organisation.
Which of the statements given above is/are correct ?
(a) 1, 2 and 3
(b) 1 and 3 only
(c) 1 only
(d) 2 and 3 only

SCRA-2013
Which one among the following countries is not a member of the Bay of Bangal Initiative for Multi-
Sectoral Technical and Economic Cooperation (BIMSTEC) ?
(a) Bangladesh (b) Malaysia (c) Nepal (d) Myanmar
Whatsapp No 7023213423 http://iasselfstudy.com/ 120

The Logistics Exchange Memorandum of Agreement (LEMOA)


Signed between Ministry of defence of India and the Department of defence of US.

Purpose LEMOA provides a framework for reciprocal provision of logistic


support, supplies and services for activities such as joint exercises,
training or humanitarian assistance and disaster relief.

Exclusion It does not provide for the establishment of any bases or basing
arrangements.

Exchange rate of Foreign Currency relating to import and export of goods


*
Central board of Excise & Custom (CBEC) determines the exchange rate of conversion of
Foreign Currency into Indian currency or vice versa relating to import and export of goods.

Exchange rate wef 03-02-2017

Sl.No. Foreign Currency Rate of exchange of one unit of foreign


currency equivalent to Indian rupees

(For Imported (For Export Goods)


Goods)

1. Australian Dollar 52.50 50.70

2. Bahrain Dinar 185.50 173.10

3. Canadian Dollar 52.70 51.10

4. Danish Kroner 10.00 9.60

5. EURO 74.20 71.70

6. Hong Kong Dollar 8.80 8.60

7. Kuwait Dinar 229.05 214.35

8. New Zealand Dollar 50.25 48.50

9. Norwegian Kroner 8.35 8.05

10. Pound Sterling 87.05 84.20

11. Singapore Dollar 48.60 47.15

12. South African Rand 5.20 4.85


Whatsapp No 7023213423 http://iasselfstudy.com/ 121

13. Saudi Arabian Riyal 18.60 17.45

14. Swedish Kroner 7.85 7.60

15. Swiss Franc 69.25 67.05

16. UAE Dirham 19.00 17.80

17. US Dollar 68.40 66.70

18. Chinese Yuan


10.00 9.65

19. Qatari Riyal


19.15 18.10

20. Japanese Yen 0.60 0.58

21. Kenya Shilling 0.67 0.62

CDS 2017
'Rand/ZAR' is the currency of
(a) Burundi
(b) Libya
(c) Sudan
(d) South Africa

Bilateral Investment Treaty (BIT) to boost investment *


Purpose of To promote and protect investments from either country.
treaty
The Treaty encourages each country to create favourable conditions
for investors of the other country.

Benefit BITs help project India as a preferred foreign direct investment (FDI)
destination

India’s BIT with The first BIT was signed by India on March 14, 1994.
other countries
Since then, the Government of India has signed BITs with 83
countries
Whatsapp No 7023213423 http://iasselfstudy.com/ 122

Foreign Trade Policy 2015-2020 *


Announced on 1st April, 2015

Focus supporting both manufacturing and services exports

Introduction of Merchandise Exports from India Scheme (MEIS) and


two new schemes
Services Exports from India Scheme (SEIS)

MEIS MEIS aims to incentivize export of merchandise which is


produced/manufactured in India.

SEIS The Scheme provided rewards to service providers of notified


services who are providing service from India.

Classification of Economies by World Bank *


Four categories Low-income economies (Afghanistan, Nepal)

Lower middle-income economies (India, Pakistan, Sri Lanka,


Bangladesh, Bhutan, Myanmar )

Upper middle-income economies (China, Maldives)

High-income economies

India’ position India is now a 'lower-middle-income' economy for World Bank

World Bank has dropped the use of developing nation tag for India
Whatsapp No 7023213423 http://iasselfstudy.com/ 123

Chapter 9

International Institution

G-7 **
Established with 6 developed countries (France, Germany, Japan, Italy, USA, UK).

Later on Canada was included and Lastly Russia.

Russia was suspended in 2014.

CISF-LDC-2012
Six industrialized nations had an association before formation of G-8 Summit. The original members
were U.S., U.K, France, Germany, Italy and Japan. Subsequently two nations joined. The two countries
are
(a) India and China
(b) Brazil and South Mrica
(c) Russia and Canada
(d) New Zealand and Mexico

ES-2016
Which of the following is not a member of a group of countries known as G7?
(a) Australia
(b) Canada
(c) France
(d) Japan

IAS Prelims 2009


A present group of nations known as G8 started first as G7. Which one among the following was not one
of them?
A. Canada
B. Italy
C. Japan
D. Russia
Whatsapp No 7023213423 http://iasselfstudy.com/ 124

BRICS ***
History The acronym BRIC was first used in 2001 by Goldman Sachs

Nature An association of 5 major emerging economies

Founding Member Brazil, Russia, India, China. Earlier known as BRIC.


(4)
South Africa joined later in 2010 then called as BRICS

First BRIC summit held in Yekaterinburg, Russia, on 16 June 2009

6th BRICS summit at Fortaleza, Brazil

Announcement of The New Development Bank and Contingent


Reserve Arrangement.

7th BRICS summit held in July 2015 in Ufa, Russia

8th BRICS Summit held in October 2016 in Goa, India

Theme was "Building Responsive, Inclusive and Collective Solutions.”

9th BRICS Summit China will host the 9thBRICS Summit in 2017

CDS-2015
Which of the following are members of BRICS?
(a) Bhutan, Russia, India, China and Sri Lanka
(b) Brazil, Russia, India, China and South Africa
(c) Brazil, Russia, Indonesia, China and Singapore
(d) Bangladesh, Republic of Korea, Indonesia, Canada and Sri Lanka

CDS-2016
BRICS leaders signed the agreement to establish a New Development Bank at the summit held In : .
(a) New Delhi, India (2012)
(b) Durban, South Africa (2013)
(c) Fortaleza, Brazil (2014)
(d) Ufa, Russia (2015)

ES-2015
In the group of countries known as BRICS,
S stands for:
(a)Singapore (b) Spain © South Africa (d) South Korea

IAS Prelims 2015


The ‘Fortaleza Declaration’ recently in the news, is related to the affairs of:
a) ASEAN b) BRICS c) OECD d) WTO

IAS Prelims 2014


With reference to a grouping of countries known as BRICS, consider the following statements:
1. The First Summit of BRICS was held in Rio de Janeiro in 2009.
2. South Africa was the last to join the BRICS grouping.
Which of the statements given above is / are correct?
A. 1 only
Whatsapp No 7023213423 http://iasselfstudy.com/ 125

B. 2 only
C. Both 1 and 2
D. Neither 1 nor 2

CDS 2017
BRICS Summit, 2016 was held in
(a) Brazil (b) China (c) India (d) South Africa

The New Development Bank **


Genesis The idea for creation of the New Development Bank was first mooted in
the Fourth BRICS Summit at New Delhi on March 29, 2012.

On July 15, 2014 at the sixth summit in Fortaleza, Brazil the BRICS
countries signed the Articles for the New Development Bank with an
Authorized Capital of USD 100 billion

Share of each The initial subscribed capital of the NDB is USD 50 billion divided equally
member among the BRICS countries

Purpose mobilizing resources for infrastructure and sustainable development


projects in BRICS and other emerging economies and developing
countries, complementing the existing efforts of multilateral and regional
financial institutions for global growth and development

HQ Shanghai, China

President Mr. K.V. Kamath (India) was elected the first President of the Bank

IAS Prelims 2016


Consider the following statements :
1. New Development Bank has been set up by APEC.
2. The headquarters of New Development Bank is in Shanghai.
Which of the statements given above is/are correct?
(a) 1 only
(b) 2 only
(c) Both 1 and 2
(d) Neither 1 nor 2

CDS-2015
Which of the following statements with regard to New Development Bank BRICS, formerly referred to
as the BRICS Development Bank, is/are correct?
1. The Headquarters of the Bank is situated at Moscow, Russia.
2. K. V. Kamath is the first President of the Bank.
Select the correct answer using the code given below.
(a) 1 only
(b) 2 only
(e) Both 1 and 2
(d) Neither 1 nor 2
Whatsapp No 7023213423 http://iasselfstudy.com/ 126

BRICS Contingent Reserve Agreement (CRA) *


Genesis Announced in the sixth BRICs summit in Fortaleza, Brazil

Purpose to provide short-term liquidity support to the members through currency


swaps to help mitigating Balance of Payment crisis situation

Share of each The Arrangement totals $100 billion:


member
China's share at $41 billion,

South Africa's share at $5 billion,

Brazil, India's and Russia's shares at $18 billion each.

South Asian Association for Regional Cooperation (SAARC) ***


Whatsapp No 7023213423 http://iasselfstudy.com/ 127

Established on 8th December 1985 Dhaka, Bangladesh (First Summit)

Founding Bangladesh, Bhutan, India, Maldives, Nepal, Pakistan and SriLanka.


Members
Afganistan joined later at 14th SAARC summit in 2007

About SAARC It is a organization of 8 countries that are primarily located in South Asia.

Afghanistan is a Central Asian country

Secretariat/HQ Kathmandu, Nepal

Postponement Pakistan in Islamabad was scheduled to host the 19th annual SAARC
of 19th summit summit.

After the terrorist attacks on the Indian army base in Uri, Kashmir, India
decided that It would boycott the summit.

Bangladesh, Sri Lanka, Afghanistan, Maldives and Bhutan followed suit,


also boycotting the summit

Nepal is only nation that have not pulled out of SAARC Summit 2016

Observer states (i) Australia

(ii) China

(iii) European Union

(iv) Iran

(v) Japan

(vi) Republic of Korea

(vii) Mauritius

(viii) Myanmar

(ix) USA

Secretary The Secretariat is headed by the Secretary General, who is appointed by


General the Council of Ministers from Member States in alphabatical order for a
three year term.

The Secretary General is assisted by eight Directors on deputation from


the Member States.

South Asian The SAFTA Agreement was signed on 6 January 2004 during Twelfth
Free Trade SAARC Summit held in Islamabad, Pakistan and came into force from 1st
Area (SAFTA) January 2006 and the Trade Liberalization Programme commenced from
1st July 2006.
Whatsapp No 7023213423 http://iasselfstudy.com/ 128

Following the Agreement coming into force the SAFTA Ministerial Council
(SMC) has been established comprising the Commerce Ministers of the
Member States.

To assist the SMC, a SAFTA Committee of Experts (SCOE) has been


formed. SCOE is expected to submit its report to SMC every six months.

The SAFTA Agreement states that the “the SMC shall meet at least once
every year or more often as and when considered necessary by the
Contracting States.

Each Contracting State shall chair the SMC for a period of one year on
rotational basis in alphabetical order.”

Currency Swap India has Currency Swap Arrangement for SAARC Member Countries
Arrangement
Under the facility, RBI offers swaps of varying sizes to each SAARC
Member countries depending on their two months import requirement
and not exceeding US$ 2 billion in total, in USD, Euro or INR to meet any
balance of payments and liquidity mismatches

CDS-2014
Consider the following statements on SAFTA :
1. SAFTA is a trade liberalisation programme among the South Eastern countries of Asia.
2. According to SAFTA, the Ministerial Council shall meet at least once every year or more often as and
when considered necessary by the Contracting States.
Select the correct answer using the code given below:
(a) 1 only
(b) 2 only
(c) Both 1 and 2
(d) Neither 1 nor 2

CDS-2015
Which of the following is not true for SAFTA (South Asian Free Trade Area) ?
(a) It is a step towards a South Asian customs union and common market.
(b) The agreement came into effect in 2006.
(c) The SAFTA is a trade liberalization regime.
(d) The SAFTA agreement takes precedence over any other agreement a member country may have
with states outside SAFTA.

IAS Prelims 2006


Consider the following statements :
1. The agreement on South Asian Free Trade Area (SAFTA) came into effect from 1st December, 2005.
2. As per SAFTA agreement terms, India, Pakistan and Sri Lanka have to decrease their custom duties
to the level of 0 to 5 percent by the year 2013.
Which of the statements given above is/are correct ?
(a) 1 only (b) 2 only
(c) Both 1 and 2 (d) Neither 1 nor 2
Whatsapp No 7023213423 http://iasselfstudy.com/ 129

IAS Prelims 1992


Which one of the following statements regarding SAARC countries is correct?
(a) No SAARC country other than India has a common border with another member country, Sri Lanka
and Maldives being islands
(b) All the countries with which India has border are members of SAARC, Sri Lanka and Maldives being
islands
(c) Except Sri Lanka and Maldives, all the SAARC countries have common border with China
(d) Except Sri Lanka and Maldives no other SAARC country has islands as part of its territory

CDS-2014
Consider the following statements about SAARC:
1. The SAARC Secretariat is located at Kathmandu.
2. The Secretariat is headed by the Secretary General, who is appointed by the Council of Ministers
from Member States in alphabetical order for a three year term.
3. The Secretary General is assisted by eight Directors on deputation from the Member States.
Select the correct answer using the code given below :
(a) 1 only
(b) 2 and 3 only
(c) 1, 2 and 3
(d) 1 and 3 only

CISF-LDC-2011
In which city is the secretariat of South Asian Association for Regional Cooperation (SAARC) located?
(a) New Delhi
(b) Islamabad
(c) Kathmandu
(d) Dhaka

CISF-LDC-2013
India is a regular/permanent member of :
1. Association of South-East Asian Nations
2. Shanghai Cooperation Organization
3. South Asian Association for Regional Cooperation
Select the correct answer using the code given below :
(a) I and 2 only
(b) 3 only
(c) 1 and 3 only
(d) I, 2 and 3

ES-2010
The permanent headquarters of SAARC secretariat is located at :
(a) Kathmandu
(b) Dhaka
(c) New Delhi
(d) Islamabad

ES-2016
Which one of the following is not a member of SAARC ?
(a) Myanmar
(b) Maldives
(c) Bhutan
(d) Afganistan
Whatsapp No 7023213423 http://iasselfstudy.com/ 130

NDA-2015
The SAARC Secretariat is located at:
(a) New Delhi
(b) Colombo
(c) Kathmandu
(d) Karachi

CDS 2017
Which one of the following statements is not correct in respect of the South Asian Association for
Regional
Cooperation (SAARC)?
(a) Its Headquarters is located in Kathmandu.
(b) China is the only country with an Observer status in SAARC.
(c) The First SAARC Summit was held in Dhaka.
(d) The Eighteenth SAARC Summit was held in Nepal.

The South Asia Subregional Economic Cooperation (SASEC) Program *


Established In 2001

About It brings together Bangladesh, Bhutan, India, the Maldives, Nepal, and Sri
SASEC Lanka in a project-based partnership to promote regional prosperity by
improving cross-border connectivity, boosting trade among member
countries, and strengthening regional economic cooperation.

Pakistan and Afganistan not a part of this group

Role of ADB The Asian Development Bank (ADB) serves as Secretariat to the SASEC
Program.

ADB supports SASEC countries in strengthening regional ties for growth and
promoting cooperation; and provides financial and technical support to
improve connectivity, strengthen institutions and trade links, and expand
human capital
Whatsapp No 7023213423 http://iasselfstudy.com/ 131

ASEAN-Association of Southeast Asian Nations **

Established on 8 August 1967 in Bangkok (Thailand)

HQ Jakarta (Indonesia)

Aims and Purposes To accelerate the economic growth, social progress, and cultural
development,

To promote regional peace and stability etc

Members 10

Founding members Indonesia, Malaysia, Philippines, Singapore and Thailand


(5)

Other members as Brunei Darussalam, Viet Nam, Lao PDR (Laos), Myanmar and
per joining date Cambodia
sequence

Observer state Papua New Guinea

India’s membership India is in South Asia. It is not a member of ASEAN

Agreement India has ASEAN–India Free Trade Area (AIFTA) agreement

Financial assistance India has provided Financial assistance to ASEAN countries in


following forms:

ASEAN-India Cooperation Fund,


Whatsapp No 7023213423 http://iasselfstudy.com/ 132

ASEAN-India Science and Technology Development Fund,

ASEAN-India Green Fund

Motto of ASEAN “One Vision, One Identity, One Community”.

IAS Prelims 2009


Consider the following pairs:
Organisation Location of Headquarters
1. Asian Development Bank Tokyo
2. Asia Pacific Economic Cooperation Singapore
3. Association of South East Asian Nations Bangkok
Which of the above pairs is/are correctly matched?
A. 1 only
B. 2 only
C. Both 1 and 2
D. Neither 1 nor 2

IAS Prelims 2009


Consider the following countries:
1. Brunei Darussalam.
2. East Timor
3. Laos
Which of the above is/are member/members of ASEAN?
A. 1 only
B. 2 and 3 only
C. 1 and 3 only
D. 1, 2 and 3

IAS Prelims 2006


Which one of the following countries is not a member of ASEAN?
(a) Vietnam
(b) Brunel Darussalam
(c) Bangladesh
(d) Myanmar

IAS Prelims 2005


Which one of the following is not an ASEAN member?
(a) Cambodia
(b) China
(c) Laos
(d) Philippines

CDS-2009
Which one of the following statements regarding the Association of South East Asian Nations (ASEAN) is
not correct ?
(a) ASEAN was established in the year 1967 in Bangkok by the five original Member Countries
(b) Singapore is one of the original members of ASEAN
(c) India is a sectoral dialogue partner of ASEAN
(d) There are ten members of ASEAN

CAPF-2011
ASEAN (Association of South East Asian Nations) Regional Block consists of a number of countries. Which
one of the following groups of countries is a part of this block?
(a) India, Pakistan, Sri Lanka, Bangladesh, Thailand, Myanmar
Whatsapp No 7023213423 http://iasselfstudy.com/ 133

(b) Brunei Darussalam, Indonesia, Malaysia, Singapore, Thailand, Vietnam


(c) Indonesia, Malaysia, New Zealand, Australia, Philippines, China
(d) Indonesia, Maldives, Bhutan, Nepal, China, Myanmar

CISF-LDC-2011
Which one of the following is not a member of ASEAN ?
(a) Brunei Darussalam
(b) Cambodia
(c) Vietnam
(d) India

ES-2010
Which one of the following is not a member of ASEAN ?
(a) Cambodia
(b) Laos
(c) Singapore
(d) South Korea

ES-2011
With reference to ASEAN, consider the following statements
1. ASEAN was initially formed by the governments of five South Asian countries.
2. It was formed in August 1967 through the Bangkok Declaration,
Which of the statements given above is/are correct?
(a) 1 only
(b) 2 only
(c) Both 1 and 2
(d) Neither 1 nor 2

ES-2013
Which one of the following is not a member of ASEAN ?
(a) Brunei Darussalam
(b) Cambodia
(c) India
(d) Vietnam

ES-2016
Which one of the following countries is not a member of ASEAN ?
(a) Indonesia
(b) South Korea
(c) Thailand
(d) Vietnam

NDA-2014
Which one of the following is not an ASEAN member?
(a) Cambodia
(b) Laos
(c) Myanmar
(d) Taiwan

CDS-2010
Which of the following statements about ASEAN is not correct?
(a) India is a founder member of ASEAN
(b) Its objectives were primarily to accelerate 'economic and social progress and cultural development'
© It is committed primarily to promote regional peace and stability
(d) It encourage negotiation over conflict in the region
Whatsapp No 7023213423 http://iasselfstudy.com/ 134

Regional Comprehensive Economic Partnership (RCEP) *


Mega-regional largely driven by ASEAN
economic agreement

negotiated between 10 ASEAN (Association of South-East Asian Nations) governments


and their six FTA (Free Trade Area) partners: Australia, China, India,
Japan, New Zealand and South Korea

IAS Prelims 2016


The term ‘Regional Comprehensive Economic Partnership’ often appears in the news in the context of
the affairs of a group of countries known as
(a) G20
(b) ASEAN
(c) SCO
(d) SAARC

Asia-Pacific Economic Cooperation (APEC) **


About APEC APEC is a regional economic forum established in 1989 to leverage the
growing interdependence of the Asia-Pacific

HQ Singapore

Member Australia; Brunei Darussalam; Canada; Chile; China; Hong Kong-China;


countries (21) Indonesia; Japan; Republic of Korea; Malaysia; Mexico; New Zealand;
Papua New Guinea; Peru; Philippines; Russia; Singapore; Chinese Taipei;
Thailand; USA; Viet Nam

Observers Association of Southeast Asian Nations Secretariat,

Pacific Economic Cooperation Council and

Pacific Islands Forum Secretariat

India’s India is not a member


membership

ES-2014
India is a member of :
1. Asia-Pacific Economic Cooperation (APEC)
2. Asian Development Bank (ADB)
3. Organization for Economic Cooperation and Development (OECD)
4. World Trade Organization (WTO)
Select the correct answer using the code given below :
(a) 1 and 3 only
(b) 2 and 4 only
(c) 1, 3 and 4 only
(d) 1, 2, 3 and 4
Whatsapp No 7023213423 http://iasselfstudy.com/ 135

ES-2010
India is not a member of :
(a) ADB
(b) APEC
(c) Antarctic Treaty
(d) Colombo Plan

Asia Pacific Trade Agreement (APTA) *


About APTA APTA (formerly known as the Bangkok Agreement) is a oldest preferential
trade agreement among developing countries in Asia-Pacific.

It is based on Margin of Preference.

It was signed in 1975.

It is an initiative of the United Nations Economic and Social Commission for


Asia and the Pacific (UNESCAP)

Members (6) Bangladesh, China, India, Lao PDR, Republic of Korea (South Korea), and
Sri Lanka.

Mongolia is soon to become the 7th member of APTA.

Secretariat ESCAP functions as the Secretariat to the Agreement.

Features of Open to all developing member countries, APTA is a truly region-wide trade
APTA agreement spanning East and South Asia, with potential to expand to other
sub-regions, including Central Asia and the Pacific.

APTA is the first plurilateral agreement among the developing countries in


the region to adopt common operational procedures for certification and
verification of the origin of goods and it has the longest effective
implementation period amongst the trade agreements in the entire Asia-
Pacific.

Notably, APTA is the only operational trade agreement linking China and
India, two of the fastest growing markets in the world, and other major
markets such as the Republic of Korea.
Whatsapp No 7023213423 http://iasselfstudy.com/ 136
Whatsapp No 7023213423 http://iasselfstudy.com/ 137

Asian Development bank *


Genesis The Asian Development Bank was conceived in the early 1960s as a
financial institution that would be Asian in character and foster
economic growth and cooperation in one of the poorest regions in the
world

Established in 1966

Function ADB assists its members, and partners, by providing loans, technical
assistance, grants, and equity investments to promote social and
economic development

HQ Manila, Philippines

India’s India is a founder member


Membership

Shareholding Japan has largest shareholding and voting power

Asian Infrastructure Investment Bank (AIIB) *


Purpose AIIB will complement and cooperate with the existing multilateral
development banks to jointly address the daunting infrastructure needs
in Asia

HQ Beijing, China

India’s India is a founder member


Membership

Shareholding China has largest shareholding and voting power

CDS-2015
Which of the following statements with regard to the proposed Asian Infrastructure Investment Bank
is/are correct ?
1. India is one of the founding members of the Bank.
2. The Bank is to be headquartered in Shanghai.
Select the correct answer using the code given below:
(a) 1 only
(b) 2 only
(c) Both 1 and 2
(d) Neither 1 nor 2
Whatsapp No 7023213423 http://iasselfstudy.com/ 138

The Colombo Plan for Cooperative Economic and Social Development in Asia
and the Pacific
History Conceived at the Commonwealth Conference on Foreign Affairs held in
Colombo, Ceylon (now Sri Lanka) in January 1950 and was launched on 1
July 1951 as a cooperative venture for the economic and social
advancement of the peoples of South and Southeast Asia

It was a group of seven Commonwealth nations – Australia, Britain,


Canada, Ceylon, India, New Zealand and Pakistan

Came into 1977


present form

Present 27, including non-Commonwealth countries


member
Afghanistan, Australia, Bangladesh, Bhutan, Brunei Darussalam, Fiji, India,
Indonesia, Iran, Japan, South Korea, Lao PDR, Malaysia, Maldives,
Mongolia, Myanmar, Nepal, New Zealand, Pakistan, Papua New Guinea,
Philippines, Saudi Arabia, Singapore, Sri Lanka, Thailand, America, Vietnam

IAS Prelims 2008


India is a member of which of the following ?
1. Asian Development Bank
2. Asia Pacific Economic Cooperation
3. Colombo Plan
4. Organization for Economic Cooperation and Development (OECD)
Select the correct answer using the code given below:
(a) 1 and 3 only (b) 2 and 4 only
(c) 1, 2 and 3 only (d) 1, 2, 3 and 4

UN Bretton Woods conference 1944


Held in Bretton Woods, New Hampshire, United States

Purpose to regulate the international monetary and financial order after the
conclusion of World War II.

Result of To create the International Monetary Fund (IMF) whose purpose was to
conference promote stability of exchange rates and financial flows and

to create IBRD which is today part of World Bank group whose purpose was
to speed reconstruction after the Second World War and to foster economic
development, especially through lending to build infrastructure

IAS Prelims 2008


How is the United Nations Monetary and financial Conference wherein the agreements were signed to
set up IBRD, GATT and IMF, commonly known?
(a) Bandung Conference (b) Bretton Woods Conference
(c) Versailles Conference (d) Yalta Conference
Whatsapp No 7023213423 http://iasselfstudy.com/ 139

International Monetary Fund **


Genesis The IMF, also known as the Fund, was conceived at a UN conference in
Bretton Woods, New Hampshire, United States, in July 1944.

The 44 countries at that conference sought to build a framework for


economic cooperation to avoid a repetition of the competitive
devaluations that had contributed to the Great Depression of the 1930s.

About IMF The International Monetary Fund (IMF) Created in 1945 is an


organization of 189 countries, working to foster global monetary
cooperation, secure financial stability, facilitate international trade,
promote high employment and sustainable economic growth, and reduce
poverty around the world.

IMF The IMF's primary purpose is to ensure the stability of the international
responsibilities monetary system—the system of exchange rates and international
payments that enables countries (and their citizens) to transact with
each other

It does so in three ways:

keeping track of the global economy and the economies of member


countries; (Surveillance)

lending to member countries with balance of payments difficulties; and


(Lending)

giving practical help/training to members countries. (Capacity


Development)

HQ Washington DC (US)

Key Publications World Economic outlook report, Global financial stability report, Fiscal
monitor, Regional Economic Reports,

India’s India is a founder member of the IMF.


membership
India is represented at the IMF by an Executive Director. He also
represents Bangladesh, Sri Lanka and Bhutan

Managing On July 5, 2011, Christine Lagarde (France) became the eleventh


Director Managing Director of the IMF, and the first woman to hold that position.

On February 19, 2016, the IMF Executive Board selected her to serve as
IMF Managing Director for a second five-year term starting on July 5,
2016

Board of The Board of Governors is the highest decision-making body of the IMF
Governors
Whatsapp No 7023213423 http://iasselfstudy.com/ 140

Meeting of IMF The Boards of Governors of the IMF and the World Bank Group normally
and World Bank meet once a year, during the IMF-World Bank Spring and Annual
Meetings, to discuss the work of their respective institutions.

Ministerial The IMF Board of Governors is advised by two ministerial committees,


Committees the International Monetary and Financial Committee (IMFC) and
the Development Committee.

IMFC The IMFC has 24 members, drawn from the pool of 187 governors.

The Committee discusses matters of common concern affecting the


global economy and also advises the IMF on the direction its work.

There is no formal voting at the IMFC, which operates by consensus.

World Bank, participate as observers in the IMFC’s meetings.

IAS Prelims 2016


With reference to the International Monetary and Financial Committee (IMFC), consider the following
statements :
1. IMFC discusses matters of concern affecting the global economy, and advises the International
Monetary Fund (IMF) on the direction of its work.
2. The World Bank participates as observer in IMFC’s meetings.
Which of the statements given above is/are correct?
(a) 1 only
(b) 2 only
(c) Both 1 and 2
(d) Neither 1 nor 2

CDS-2014
Which of the following statements about International Monetary Fund (IMP) are correct?
1. The IMF is a United Nations specialized agency.
2. The IMF was founded at the Bretton Woods Conference in 1944 to secure international monetary
cooperation.
3. The objective of the IMF is to stabilize currency exchange rates, and to expand international liquidity
(access to hard currencies).
Select the correct answer using the code given below :
(a) 1, 2 and 3
(b) 2 and 3 only
(c) 1 and 2 only
(d) 1 and 3 only

IAS Prelims 1991


The main watchdog of international trade is
(a) IMF
(b) World Bank
(c) GATT (WTO)
(d) IFC

IAS Prelims 1994


The main function of the I.M.F. is to
(a) arrange international deposits from banks
(b) help to solve balance of payments problems of member countries
Whatsapp No 7023213423 http://iasselfstudy.com/ 141

(c) act as a private sector lending arm of the World Bank


(d) finance investment to developing countries

IAS Prelims 1999


Match List I with List II and select the correct answer using the codes given below the Lists :
List I List II
I. WTO (A) Provides loans to address short term balance of payment problems
II. IDA (B) Multilateral trade negotiation body
III. IMF (C) Sanction of soft loans
IV. IBRD (D) Facilitating lendings and borrowings for reconstruction and development
Codes:
(a) IB, IIC, IIID, IVA
(b) IB, IIC, IIIA, IVD
(c) IC, IIB, IIID, IVA
(d) IC, IIB, IIIA, IVD

IAS Prelims 2016


‘Global Financial Stability Report’ is prepared by the
(a) European Central Bank
(b) International Monetary Fund
(c) International Bank for Reconstruction and Development
(d) Organization for Economic Cooperation and Development

IAS Prelims 2014


Which of the following organizations brings out the publication known as ‘World Economic Outlook’?
A. The International Monetary Fund
B. The United Nations Development Programme
C. The World Economic Forum
D. The World Bank

IAS Prelims 2011


Regarding the international monetary fund, which one of the following statements is correct ?
(a) It can grant loans to any country.
(b) It can grant loans to only developed countries.
(c) It grants loans to only member countries.
(d) It can grant loans to the central bank of a country.

CDS-2011
India is represented at the IMF by an Executive Director. He also represents :
1. Bangladesh
2. Sri Lanka
3. Bhutan
4. Pakistan
Select the correct answer using the code given below :
(a) 1 only
(b) 3 and 4 only
(c) 1, 2 and 3 only
(d) 1, 2, 3 and 4
Whatsapp No 7023213423 http://iasselfstudy.com/ 142

World Bank Group **


Established in 1944 to help Europe recover from the devastation of World War II.

HQ Washington DC (US)

About World The World Bank is a vital source of financial and technical assistance to
Bank developing countries around the world

Five Institutions, World bank group consist of five organizations.


One Group
International Bank for Reconstruction and Development (IBRD) (India
Founder Member)

International Development Association (IDA) (India Founder Member)

International Finance Corporation (IFC) (India Founder Member)

Multilateral Investment Guarantee Agency (MIGA) (India became


Member in 1994)

International Centre for Settlement of Investment Disputes (ICSID)

(India is not a member of ICSID)

Together, IBRD and IDA make up the World Bank

IBRD It was created in 1944 to help Europe rebuild after World War II.

lends to governments of middle-income and creditworthy low-income


countries.

IBRD is the original World Bank institution.

It works closely with the rest of the World Bank Group to help
developing countries reduce poverty, promote economic growth, and
build prosperity.

IDA IDA provides interest-free loans — called credits — and grants to


governments of the poorest countries.

IFC IFC is the largest global development institution focused exclusively on


the private sector in developing countries.

helps developing countries achieve sustainable growth by financing


investment, mobilizing capital in international financial markets, and
providing advisory services to businesses and governments.

MIGA mission is to promote foreign direct investment (FDI) into developing


countries to support economic growth, reduce poverty, and improve
people's lives.
Whatsapp No 7023213423 http://iasselfstudy.com/ 143

MIGA fulfills this mandate by offering political risk insurance


(guarantees) to investors and lenders

ICSID It provides international facilities for conciliation and arbitration of


investment disputes.

Two ambitious The World Bank Group has set two goals for the world to achieve by
goals 2030:

End extreme poverty by decreasing the percentage of people living on


less than $1.90 a day to no more than 3%

Promote shared prosperity by fostering the income growth of the


bottom 40% for every country

President Jim Yong Kim (South Korea-America)

Key Publications World Development Report, World development indicators, Doing


Business, Global Economic Prospects, International Debt Statistics,
Poverty and shared prosperity, Global Financial Development Report,
South Asia Economic Focus Fall 2016:Investment Reality Check

IAS Prelims 2016


India’s ranking in the ‘Ease of Doing Business Index’ is sometimes seen in the news. Which of the
following has declared that ranking?
(a) Organization for Economic Cooperation and Development (OECD)
(b) World Economic Forum
(c) World Bank
(d) World Trade Organization (WTO)

CDS-2014
Consider the following statements relating to the World Bank :
1. The World Bank was established in 1946, which is headquartered in New York.
2. The World Bank Group has set for itself the goal to end extreme poverty from the World by 2030.
3. The World Bank is a vital source of financial and technical assistance to developing countries around
the world. It is not a bank in the ordinary sense but a unique partnership to reduce poverty and
support development.
4. The World Bank Group comprises five institutions managed by their member countries in order to
promote shared prosperity by fostering the income growth of the bottom 40% for every country.
Which of the statements given above are correct?
(a) 1, 2 and 3
(b) 2, 3 and 4
(c) 1, 3 and 4
(d) 2 and 4 only

SCRA-2013
Which one among the following agencies is known as ‘soft credit window’ of the World Bank?
(a) The International Finance Corporation
(b)The International Bank for Reconstruction and Development
(c) The International Development Association
(d)The Multilateral Investment Guarantee Agency
Whatsapp No 7023213423 http://iasselfstudy.com/ 144

IAS Prelims 2010


The International Development Association, a lending agency, is administered by the
(a.) International Bank for Reconstruction and Development
(b.) International Flllld for Agricultural Development
(c.) United Nations Development Programme
(d.) United Nations Industrial Development Organization

IAS Prelims 2010


Which of the following is not related to United Nations?
(a) Multilateral Investment Guarantee Agency
(b) International Finance Corporation
© International Centre for Settlement of lnvestment Disputes
(d.) Bank for International Settlements

IAS Prelims 2002


“World Development Report” is an annual publication of
(a) United Nations Development Programme
(b) International Bank of Reconstruction and Development
(c) World Trade Organisation
(d) International Monetary Fund

IAS Prelims 1995


Which of the following constitute the World Bank ?
I. International Bank for Reconstruction and Development
II. International Finance Corporation
III. International Development Association
IV. International Monetary Fund.
Choose the correct answer from the codes given below :
Codes :
(a) I, II and III
(b) I and II
(c) III and IV
(d) I, II, III and I

IAS Prelims 2015

Which one of the following issues the “Global Economic Prospects” report periodically?
a) The Asian Development Bank
b) The European Bank for Reconstruction and Development
c) The US Federal Reserve Bank
d) The World Bank
Whatsapp No 7023213423 http://iasselfstudy.com/ 145

World Trade Organization **


Created by Uruguay Round negotiations (1986-94)

Established 1 January 1995 replacing General Agreement on Tariffs and Trade


(GATT, 1948).

About WTO The WTO is a rules-based, member-driven organization — all decisions


are made by the member governments, and the rules are the outcome
of negotiations among members.

WTO deals with the global rules of trade between nations.

Its main function is to ensure that trade flows as smoothly, predictably


and freely as possible

Function • Administering WTO trade agreements


• Forum for trade negotiations
• Handling trade disputes
• Monitoring national trade policies
• Technical assistance and training for developing countries
• Cooperation with other international organizations

WTO agreements The WTO agreements cover goods, services and intellectual property

These agreements are negotiated and signed by the bulk of the world’s
trading nations and ratified in their parliaments.

These documents provide the legal ground rules for international


commerce.

The goal is to help producers of goods and services, exporters, and


importers conduct their business.

The WTO is currently the host to new negotiations, under the ‘Doha
Development Agenda’ launched in 2001.

Fundamental The WTO agreements are lengthy and complex because they are legal
principles of WTO texts covering a wide range of activities.
Agreements
But a number of simple, fundamental principles run throughout all of
these documents.

These principles are the foundation of the multilateral trading system.

Non-discrimination-A country should not discriminate between its


trading partners and it should not discriminate between its own and
foreign products, services or nationals.

More open-Lowering trade barriers


Whatsapp No 7023213423 http://iasselfstudy.com/ 146

Predictable and transparent-Foreign companies, investors and


governments should be confident that trade barriers should not be
raised arbitrarily.

More competitive-Discouraging ‘unfair’ practices, such as export


subsidies and dumping products at below cost to gain market share;

More beneficial for less developed countries-Giving them more


time to adjust, greater flexibility and special privileges; Over 75% of
WTO members are developing countries.

Protect the environment-The WTO’s agreements permit members to


take measures to protect not only the environment but also public
health, animal health and plant health.

However, these measures must be applied in the same way to both


national and foreign businesses.

In other words, members must not use environmental protection


measures as a means of disguising protectionist policies.

HQ Geneva, Switzerland

Director-General Roberto Azevêdo (Brazil)

India’s India is a founder members of WTO


membership

Afghanistan Recently, Afghanistan became 164th WTO member

Key Publications World trade report, World trade statistical review

Most-favoured- Under the WTO agreements, countries cannot normally discriminate


nation (MFN): between their trading partners.
treating other
Grant someone a special favour (such as lower customs duty), you
people equally
have to do the same for all other WTO members.

MFN doesn’t mean preferential treatment. MFN clauses promote non-


discrimination among countries.

However, a country may enter into a free trade agreement or customs


union granting more favourable treatment to the participating states
than to the other WTO members if it observes certain conditions
stipulated in the relevant provisions of the WTO agreements, to ensure
the complementarity of the FTA with the WTO system

India MFN Status India has already granted MFN status to Pakistan in 1996. Pakistan has
yet to grant the most favoured nation (MFN) status to India
Whatsapp No 7023213423 http://iasselfstudy.com/ 147

Subsidies In WTO terminology, subsidies in general are identified by “boxes”


which are given the colours of traffic lights: green (permitted), amber
(slow down — i.e. be reduced), red (forbidden).

World Trade The WTO has launched a new World Trade Outlook Indicator (WTOI)
Outlook Indicator designed to provide “real time” information on trends in global trade

Trade Facilitation WTO members concluded negotiations at the 2013 Bali Ministerial
Agreement (TFA) Conference on the landmark Trade Facilitation Agreement (TFA), which
entered into force on 22 February 2017 following its ratification by
two-thirds of the WTO membership

TRIPS Agreement Trade-Related Aspect of Intellectual Property Rights (TRIPS) is a most


comprehensive multilateral agreement on intellectual property which
came into effect on 1st January 1995.

CDS 2017
The Most Favoured Nation (MFN) Clause under WTO regime is based on the principle of
(a) non-discrimination between nations
(b) discrimination between nations
(c) differential treatment between locals and foreigners
(d) uniform tariff across commodities

NDA-2011
TRIPS (Trade Related aspects of Intellectual Property Rights) agreement is administered by :
(a) United Nations Conference on Trade and Development (UNCTAD)
(b) United Nations Organization (UNO)
(c) World Trade Organization (WTO)
(d) World Bank (WB)

CDS 2017
TRIPS Agreement pertains to
(a) international tariff regime
(b) intellectual property protection
(c) international practices on trade facilitation
(d) international taxation of property

ES-2015
'Doha Development Agenda' sometimes appears in news with reference to :
(a) Organisation for Economic Cooperation and Development
(b) United Conference on Trade and Development
(c) United Nations Development Programme
(d) World Trade Organisation

CDS-2012
Which one among the following is not a clause of World Trade Organization?
(a) Most favoured treatment
(b) Lowering trade barriers with Negotiations
© Providing financial support to the countries having deficit balance of payments
(d) Discouraging unfair trade practices such as antidumping and export subsidies
Whatsapp No 7023213423 http://iasselfstudy.com/ 148

CDS-2014
Consider the following statements relating to the World Trade Organization (WTO) :
1. The WTO deals with the global rules of trade between nations.
2. The goal of the WTO is to help producers of goods and services, exporters, and importers conduct
their business.
3. The WTO, which is a successor body of the General Agreement on 'Tariffs and Trade, came into
being following the ,'Uruguay Round of Negotiations.
4. The WTO distances itself in framing of rules on trade In intellectual property rights.
Which of the statements given above are correct?
(a) 1, 2 and 3
(b) 2, 3 and 4
(c) 1, 2 and 4
(d) 1 and 3 only

CDS-2015
Which of the following statements is not true?
(a) The General Agreement on Tariffs and Trade (GAIT) had regulated global trade since 1947.
(b) GATT was replaced by the World Trade Organisation (WTO) in 1995.
© The Most Favoured Nation principle under GATT' provided that preferential trading agreements
reached with one country should be extended to other countries.
(d) The WTO has been able to cover in its agreements the agriculture and textile sectors which are the
principal concerns for the Least Developed Countries (LDCs).

CDS-2009
In which of the following years was General Agreement on Tariffs and Trade (GATT) absorbed into the
World Trade Organization (WTO)?
(a) 1991
(b) 1995
(c) 2000
(d) 2005

CISF-LDC-2012
The World Trade- Organization (WTO) is an international organization which sets the rules for Global
Trade. It was set up in the year 1995 as the successor to the
(a) General Agreement on Trade and Tariffs
(b) International Monetary Fund
(c) World Bank
(d) United Nations Conference on Trade and Development

CISF-LDC-2012
Which one among the following does not fall under WTO's mission ?
(a) Assist developing countries in its trade activities
(b) Set rules for trade policy
(c) Regulate international financial transactions and markets
(d) Protect small and weaker countries from larger, more powerful ones

ES-2009
"Uruguay Round" is mentioned in the context of the establishment of which one of the following?
(a) World Bank
(b) IMF
(c) World Economic Forum
(d) WTO
Whatsapp No 7023213423 http://iasselfstudy.com/ 149

ES-2016
Consider the following statements:
1. Headquarters of World Trade Organization (WTO) are in Doha
2. World Intellectual Property Organization is a specialized agency of WTO
Which of the statements given above is/are correct
(a) 1 only
(b) 2 only
(c) Both 1 and 2
(d) Neither 1 nor 2

NDA-2011
Which among the following statements is/are true with regard to WTO membership ?
1. All WTO members automatically receive the 'most favoured nation' status
2. Over 75% of WTO members are from developing countries . WTO membership allows them access
to developed markets at the lower tariff
Select the correct answer using the code given below :
(a) 1 only
(b) 2 only
(c) Both 1 and 2
(d) Neither 1 nor 2

IAS Prelims 2001


The earlier name of WTO was
(a) UNCTAD
(b) GATT
(c) UNIDO
(d) OECD

IAS Prelims 1993


Most Favoured Nations (MFN) clause under GATT implies
(a) most favour to some countries
(b) most favour to all countries
(c) no favour to any country
(d) no favour to some countries

IAS Prelims 1996


Which of the following were the aims behind the setting up of the World Trade Organization (WTO) ?
I. Promotion of free trade and resource flows across countries.
II. Protection of intellectual property rights.
III. Managing balanced trade between different countries.
IV. Promotion of trade between the former East Block countries and the western world.
Select the correct answer by using the codes given below :
Codes :
(a) I, II, III and IV
(b) I and II
(c) II and III
(d) I and IV

IAS Prelims 2016


In the context of which of the following do you sometimes find the terms `amber box, blue box and
green box’ in the news?
(a) WTO affairs
(b) SAARC affairs
(c) UNFCCC affairs
(d) India-EU negotiations on FTA
Whatsapp No 7023213423 http://iasselfstudy.com/ 150

CDS exam
Consider the following statements about World Trade Organization :
1. It oversees financial institutions and regulations that act at the international level
2. It was set up as the successor to the General Agreement on Trade and Tariff (GATT)
Which of the statements given above is/are correct?
(a) 1 only
(b) 2 only
(c) Both 1 and 2
(d) Neither 1 nor 2

European Union *
About EU The European Union is a unique economic and political union
between 28 European countries. The EU was created in the
aftermath of the Second World War

History In 1957, Belgium, France, Italy, Luxembourg, the Netherlands


and West Germany signed the Treaty of Rome, which created
the European Economic Community (EEC)

The European Union was formally established when the Maastricht


Treaty came into force on 1 November 1993.

Countries Austria, Belgium, Bulgaria, Croatia, Republic of Cyprus, Czech


Republic, Denmark, Estonia, Finland, France, Germany, Greece,
Hungary, Ireland, Italy, Latvia, Lithuania, Luxembourg, Malta,
Netherlands, Poland, Portugal, Romania, Slovakia, Slovenia,
Spain, Sweden and United Kingdom

Capital Brussels (Belgium)

Currency Euro

Working of EU The EU is based on the rule of law: everything it does is founded


on treaties, voluntarily and democratically agreed by its member
countries.

Parliament The EU is also governed by the principle of representative


democracy, with citizens directly represented at Union level in
the European Parliament and

Member States represented in the European Council and


the Council of the EU

Nobel Peace Prize In 2012, the EU was awarded the Nobel Peace Prize for advancing
the causes of peace, reconciliation, democracy and human rights
in Europe.
Whatsapp No 7023213423 http://iasselfstudy.com/ 151

Brexit (Britain-exit) The United Kingdom's withdrawal from the European Union is
widely known as Brexit

Reason of Greece Excessive borrowing from IMF after the global financial crisis in
default of IMF loan 2008

Troika Troika means a group of three international organisations that were


lending money to Greece bailout

European Commission

European Central Bank

International Monetary Fund

13th EU-India Summit Held on 30 March 2016 in Brussels (Belgium)

Outcome of summit EU-India Agenda for Action-2020 as a common roadmap to jointly


guide and strengthen the India-EU Strategic Partnership in the
next five years

CDS 2017
BREXIT refers to the Great Britain leaving which one of the following?
(a) International Monetary Fund
(b) Commonwealth
(c) World Trade Organization
(d) European Union

IAS Prelims 2009


Consider the following countries:
1. Switzerland
2. Malta
3. Bulgaria
Which of the above are members of European Union?
A. 1 and 2 only
B. 2 and 3 only
C. 1 and 3 only
D. 1, 2 and 3

CDS-2011
Which among the following statements about European Union (EU) are correct?
I, The EU is the world's largest economy.
2. The EU has its own flag, anthem and currency.
3. The EU's combined armed forces are the second largest in the world.
4. The EU has its own Constitution.
Select the correct answer using the code given below:
(a) 1, 2 and 3
(b) 1 and 4
(c) 2 and 3 only
(d) 3 and 4
Whatsapp No 7023213423 http://iasselfstudy.com/ 152

CDS-2012
The basis of European Union began with the signing of
(a) Maastricht Treaty
(b) Treaty of Paris
(c) Treaty' of Rome
(d) Treaty of Lisbon

CDS-2016
Which one of the following nations has faced severe economic crisis in the year 2015
resulting in default in repayment of IMF loan?
(a) China
(b) Greece
(c) Ireland
(d) Belgium

ES-2011
In the recent past, one of the European countries suffered from a huge debt crisis often referred to as
'Sovereign Debt Crisis', Which one of the following is the country in question?
(a) Greece
(b) Italy
(c) Hungary
(d) Poland

NDA-2011
Which one among the following led to the Greece economic crisis 20I0 ?
(a) Excessive borrowing from IMF
(b) Sudden depreciation of Euro
(c) Unrestricted spending and cheap loans
(d) Outflow of foreign capital due to political unrest

India–Africa Forum Summit *


Held in every three years rotational basis

First summit, in New Delhi


2008

Second summit, in Addis Ababa, Ethiopia


2011

Third summit, in New Delhi (54 African countries participated)


2015

Reason of delay Ebola outbreak in few western African countries


Whatsapp No 7023213423 http://iasselfstudy.com/ 153

The Shanghai Cooperation Organization **


About SCO SCO is a permanent intergovernmental international organisation,
created on 15 June 2001 in Shanghai (China) by the Kazakhstan,
China, Kyrgyz Republic, Russia, Tajikistan, and the Uzbekistan

History Earlier it was known as Shanghai Five mechanism. All the above
countries except Uzbekistan were members

Decision-making The Heads of State Council (HSC) is the supreme decision-making


body body in the SCO. It meets once a year and adopts decisions and
guidelines on all important matters of the organisation

Secretariat or HQ Beijing (China)

Observer states SCO has six observer states:

India,

Pakistan,

Afghanistan,

Belarus,

Iran,

Mongolia,

dialogue partners SCO has six dialogue partners:

Azerbaijan,

Armenia,

Cambodia,

Nepal,

Turkey,

Sri Lanka.

Presidency of the Every year, presidency of the SCO rotates. The chairing nation
SCO completes its year of presidency with a summit.

India’s At the 15th SCO summit in July 2015 held in Ufa (Russia), it was
membership decided to admit India and Pakistan as member states in SCO.

India signed a memorandum of obligations at the 16th SCO summit


held in Tashkent (Uzbekistan) in June 2016.

India is expected to join the SCO at the 17th SCO summit to be held
in Astana (Kazakhstan) in June 2017.
Whatsapp No 7023213423 http://iasselfstudy.com/ 154

IAS Prelims 2007


Which one of the followings not a member of the Shanghai Cooperation Organisation ?
(a) Russia (b) Kazakhstan
(c)Ukraine (d) Uzbekistan

IAS Prelims 2007


Consider the following statements:
1. China has the observer’s status at the South Asian Association for Regional Cooperation.
2. India has the observer’s status at the Shanghai Cooperation Organisation.
Which of the statements given above is/are correct?
(a) 1 only (b) 2 only
(c) Both 1 and 2 (d) neither 1 nor 2
Whatsapp No 7023213423 http://iasselfstudy.com/ 155

United Nations (UN) **


Founded In 1945

Members 193 countries

Function The mission and work of the United Nations are guided by the
purposes and principles contained in its founding Charter.

Due to the powers vested in its Charter and its unique international
character, the United Nations can take action on the issues
confronting humanity in the 21st century, such as peace and security,
climate change, sustainable development, human rights,
disarmament, terrorism, humanitarian and health emergencies,
gender equality, governance, food production, and more.

Main organs of the General Assembly,


UN
Security Council,

Economic and Social Council,

Trusteeship Council, (suspended operation on 1 November 1994)

International Court of Justice, and

UN Secretariat

All were established in 1945 when the UN was founded

General Assembly The General Assembly is the main deliberative, policymaking and
representative organ of the UN.

All 193 Member States of the UN are represented in the General


Assembly, making it the only UN body with universal representation.

Each year, in September, the full UN membership meets in the


General Assembly Hall in New York for the annual General Assembly
session, and general debate, which many heads of state attend and
address.

Decisions on important questions, such as those on peace and


security, admission of new members and budgetary matters, require a
two-thirds majority of the General Assembly.

Decisions on other questions are by simple majority.

The General Assembly, each year, elects a GA President to serve a


one-year term of office.

Security Council The Security Council has primary responsibility, under the UN Charter,
for the maintenance of international peace and security.
Whatsapp No 7023213423 http://iasselfstudy.com/ 156

It has 15 Members (5 permanent and 10 non-permanent members).

Each Member has one vote.

Under the Charter, all Member States are obligated to comply with
Council decisions.

The Security Council has a Presidency, which rotates, and changes,


every month

Permanent US, UK, France, China and Russia


Members (P5)

Veto Power of (P5) As per UN charter, every permanent member of UN security council
can exercise VETO power and disapprove any decision

Selection of non- Each year the General Assembly elects five non-permanent members
permanent (out of 10 in total) for a two-year term.
members
In the election of non-permanent members, due regard shall be paid
to the contribution of Members to the maintenance of international
peace and security and to the other purposes of the Organization, and
also to equitable geographical distribution.

A retiring member of the Security Council shall not be eligible for


immediate re-election

General Assembly has decided that “the ten non-permanent members


of the Security Council shall be elected according to the following
pattern:

(a) Five from African and Asian States;

(b) One from Eastern European States;

(c) Two from Latin American States;

(d) Two from Western European and other States.”

Economic and It is the United Nations’ central platform for reflection, debate, and
Social Council innovative thinking on sustainable development.

It is the principal body for coordination, policy review, policy dialogue


and recommendations on economic, social and environmental issues,
as well as implementation of internationally agreed development goals

International The International Court of Justice is the principal judicial organ of the
Court of Justice United Nations. Its seat is at the Peace Palace in the Hague
(Netherlands).

It is the only one of the six principal organs of the United Nations not
located in New York (United States of America).
Whatsapp No 7023213423 http://iasselfstudy.com/ 157

The Court’s role is to settle, in accordance with international law, legal


disputes submitted to it by States and to give advisory opinions on
legal questions referred to it by authorized United Nations organs and
specialized agencies.

Secretariat The Secretariat comprises the Secretary-General and tens of


thousands of international UN staff members who carry out the day-
to-day work of the UN

How to become Memberships is open to all peace-loving States that accept the
member of UN obligations contained in the United Nations Charter.

States are admitted to membership in the United Nations by decision


of the General Assembly upon the recommendation of the Security
Council.

Secretary-General The Secretary-General is chief administrative officer of the


Organization, appointed by the General Assembly on the
recommendation of the Security Council for a five-year, and may
serve up to two terms

Present Secretary-General is Ban Ki –moon (South Korea)

Official languages There are six official languages of the UN.


of the UN
These are Arabic, Chinese, English, French, Russian and Spanish.

UN Specialized UN specializes agencies are autonomous organizations working with


Agencies the UN. All were brought into relationship with the UN through
negotiated agreements. They are:

World Bank

IMF

World Health organization (WHO)

UNESCO

International Labour organization (ILO)

World Intellectual property organization (WIPO)

Food and Agriculture organization (FAO)

International Fund for Agricultural Development (IFAD)

HQ New York, USA

India’s India is a founding member of the United Nations


membership
Whatsapp No 7023213423 http://iasselfstudy.com/ 158

CDS-2011
Consider the following statements
1. The five permanent members of the Security Council are the only countries recognized as nuclear-
weapon States under the Nuclear Non-Proliferation Treaty.
2. The term of non-permanent members of the Council is five years.
Which of the statements given above is/are correct?
(a) 1 only
(b) 2 only
(c) Both 1 and 2
(d) Neither 1 nor 2

CDS-2011
Consider the following statements regarding India's advocacy for a permanent seat in the United
Nations Security Council :
I. India is the largest democracy in the world.
2. India is among the top five largest growing economies in the world.
3. India has been the largest contributor to the United Nations Peacekeeping Forces.
4. India is one of the top ten contributors of the United Nations Budget.
Which of the statements given above is/ are correct?
(a) I, 2, 3 and 4
(b) I and 2 only
(c) 2 only
(d) I, 3 and 4 only

CDS-2011
Which one among, the following statements about United Nations organs is correct ?
(a) Decisions of the General Assembly are binding on all members
(b) The term of the non-permanent members of the Security Council is for three years
(c) International Court of Justice has 20 Judges elected for a period of five years
(d) The Trusteeship Council has been suspended since 1 November 1994

CDS-2012
Which one among the following statements about Veto Power with regard to United Nations is correct?
(a) One veto can stall a Security Council resolution
(b) It is a kind of positive vote power
(c) The Secretary General uses this power when not satisfied with any decision of the Council
(d) All the members of the Security Council possess the veto power

CDS-2013
Consider the following statements about the UNO:
1. The creation of the UNO was a reflection of the ongoing quest for international peace during the
Second World War
2. The Charter of the UN explicitly mentions two vital objectives of the organization : (i) the quest for
international peace and (ii) all round welfare of the people of the world
3. The UN has resolved all the major global disputes including the Palestine dispute
Which of the statements given above is/are correct ?
(a) 1 and 2
(b) 1 and 3
(c) 2 and 3
(d) 1 only

ES-2009
The Presidency of the Security Council of United Nations is alternated every
(a) Month
(b) Six months.
(c) Year
(d) Two years
Whatsapp No 7023213423 http://iasselfstudy.com/ 159

ES-2012
With reference to the Security Council of United Nations, which of the following statements is/arc
correct?
1 It has 15 members.
2. The non-permanent members are elected by the General Assembly for a five-year period.
3. Retiring members are not eligible for immediate re-election.
Select the correct answer using the codes given. below :
(a) I only
(b) 2 and 3 only
(c) I and 3 only
(d) I, 2 and 3

ES-2015
How many are the non-permanent members of United Nations Security-Council?
(a) Five
(b) Ten
(c) Fifteen
(d) Twenty

IAS Prelims 2009


The Security Council of UN consists of 5 permanent members, and the remaining 10 members are elected
by the General Assembly for a term of
A. 1 year
B. 2 years
C. 3 years
D. 5 years

IAS Prelims 2001


Consider the following organizations :
I. International Bank for Reconstruction and Development
II. International Finance Corporation
III. International Fund for Agricultural Development
IV. International Monetary Fund
Which of these are agencies of the United Nations ?
(a) I and II
(b) II and III
(c) III and IV
(d) I, II, III and IV

IAS Prelims 1994


Which one of the following statements regarding the “veto” power in the Security Council is correct
according to the United Nations Charter ?
(a) The decisions of the Security Council on all nonprocedural matters must be made by an affirmative
vote of nine members, including the concurring, votes of the permanent members of the Council
(b) Every permanent member of the Security Council can prevent any decision from being accepted, by
vetoing it
(c) The term veto was used in Article 27 of the United Nations Charter to enable any permanent member
of the Security Council to prevent any resolution from being passed by the majority
(d) Any member of the Security Council can prevent any resolution from being passed by voting against
it

IAS Prelims 1998


What are the official languages of the U.N.O. ?
(a) English, French and Russian
(b) English, French, German and Russian
(c) English, French, Russian, Chinese and Hindi
(d) English, French, Chinese, Russian, Arabian and Spanish
Whatsapp No 7023213423 http://iasselfstudy.com/ 160

United Nations Sustainable Development Goals (SDGs) **


When adopted On 70th anniversary of UN in 2015

To be achieved By 2030

Replaced MDGs The new 17 goals replaced the 8 Millennium Development Goals
adopted in 2000 which expired at the end of 2015

Goals & Target There are 17 Sustainable Development Goals and 169 targets

Effective from The new Goals and targets will come into effect on 1 January 2016.

17 SDG Goal 1. End poverty in all its forms everywhere

Goal 2. End hunger, achieve food security and improved nutrition and
promote sustainable agriculture

Goal 3. Ensure healthy lives and promote well-being for all at all ages

Goal 4. Ensure inclusive and equitable quality education and promote


lifelong learning opportunities for all

Goal 5. Achieve gender equality and empower all women and girls

Goal 6. Ensure availability and sustainable management of water and


sanitation for all

Goal 7. Ensure access to affordable, reliable, sustainable and modern


energy for all

Goal 8. Promote sustained, inclusive and sustainable economic growth,


full and productive employment and decent work for all

Goal 9. Build resilient infrastructure, promote inclusive and sustainable


industrialization and foster innovation

Goal 10. Reduce inequality within and among countries

Goal 11. Make cities and human settlements inclusive, safe, resilient
and sustainable

Goal 12. Ensure sustainable consumption and production patterns

Goal 13. Take urgent action to combat climate change and its impacts

Goal 14. Conserve and sustainably use the oceans, seas and marine
resources for sustainable development

Goal 15. Protect, restore and promote sustainable use of terrestrial


ecosystems, sustainably manage forests, combat desertification, and
halt and reverse land degradation and halt biodiversity loss
Whatsapp No 7023213423 http://iasselfstudy.com/ 161

Goal 16. Promote peaceful and inclusive societies for sustainable


development, provide access to justice for all and build effective,
accountable and inclusive institutions at all levels

Goal 17. Strengthen the means of implementation and revitalize the


global partnership for sustainable development

Responsible At UN United Nations Development Programme (UNDP)


agency for
In India Development Monitoring and Evaluation Office
implementation
of the SDGs (DMEO), an attached office under NITI Aayog

IAS Prelims -2016


Consider the following statements:
1. The Sustainable Development Goals were first proposed in 1972 by a global think tank called
the 'Club of Rome',
2. The Sustainable Development Goals have to be achieved by 2030.
Which of the statements given above is / are correct?
(a) 1 only
(b) 2 only
(c) Both 1 and 2
(d) Neither 1 nor 2

Human Development index under Human Development Report **


Issued by United Nations Development Programme (UNDP)

created by Indian economist Amartya Sen and Pakistani economist Mahbub ul Haq
in 1990

A composite index measuring average achievement in three basic


dimensions of human development

1. Education

(i) Mean years of schooling: Average number of years of education


received by people ages 25 and older,

(ii) Expected years of schooling: Number of years of schooling that a


child of school entrance age can expect

2. Gross national income per capita in purchasing power parity terms

3. Life expectancy at birth

Classification of Countries have been divided in 4 categories- If HDI index is:


countries
0.800 or greater - Very high human development

0.700 to 0.799- High human development


Whatsapp No 7023213423 http://iasselfstudy.com/ 162

0.550 to 0.699 - Medium human development

Less than 0.550- Low human development

Theme of Work for Human Development


Human
development
report 2015

Ranking of India HIGH HUMAN DEVELOPMENT


and its
Sri Lanka-73
neighbour
China-90

Maldives-104

MEDIUM HUMAN DEVELOPMENT

India-130

Bhutan-132

Bangladesh-142

LOW HUMAN DEVELOPMENT

Nepal-145

Pakistan-147

Myanmar-148

CDS-2016
Human Development Report for each year at global level is published by
(a) WTO
(b) World Bank
(c) UNDP
(d) IMF

IAS Prelims 1997


Human Development Index comprises literacy rates, life expectancy at birth and
(a) Gross Domestic Product per head in US dollars
(b) Gross Domestic Product per head at real purchasing power
(c) Gross National Product in US dollars
(d) National Income per head in US dollars

IAS Prelims 2003


As per the Human Development Index given by UNDP, which one of the following sequences of South
Asian countries is correct, in the order of higher to lower development?
(a) India—Sri Lanka—Pakistan—Maldives
(b) Maldives—Sri Lanka—India—Pakistan
(c) Sri Lanka—Maldives—India—Pakistan
(d) Maldives—India—Pakistan—Sri Lanka
Whatsapp No 7023213423 http://iasselfstudy.com/ 163

Publications/Reports of World Renowned Institutions ***


International Monetary World Economic Outlook, Global Financial Stability Report, Fiscal
Fund (IMF) Monitor, Regional Economic Reports

World Bank World Development Report, World development indicators, Doing


Business, Global Economic Prospects, International Debt
Statistics, Poverty and shared prosperity, Global Financial
Development Report

World Trade World trade report, World trade statistical review


Organization (WTO)

United Nations (UN) Sustainable Development Goals report, World Economic Situation
and Prospects, World Youth Report, World Statistics Pocketbook

United Nations Children's State of the World’s children


Fund (UNICEF)

United Nations World Investment Report, Trade and Development report


conference on trade and
development (UNCTAD)

United Nations Human Development Report


Development
Programme (UNDP)

United Nations Office on World Drug Report, World wildlife crime report
Drugs and
Crime (UNODC)

United Nations State of the World’s Population


Population Fund

United Nations Office for Sendai Framework for Disaster Risk Reduction 2015-2030
Disaster Risk Reduction
(UNISDR)

Food and Agriculture The State of Food and Agriculture, The State of World Fisheries
Organization of the and Aquaculture
United Nations (FAO)

World Economic Forum Global Competitiveness Report, Global Competitiveness Index,


Global Risks Report, Global Gender Gap Report, Human Capital
Report, Global Information Technology Report, The Inclusive
Growth and Development Report

International labour World Employment Social Outlook, Global Wage Report, World
organisation Social Protection Report, Global Employment Trends for Youth,
World Report on Child Labour
Whatsapp No 7023213423 http://iasselfstudy.com/ 164

Asian Development Bank Asian Development Outlook, Asian Development Review, Asia
Bond Monitor

European Central Bank Economic Bulletin, Financial Stability Review, Convergence


Report, Macroeconomic projections

IAS Prelims 2014


Which of the following organizations brings out the publication known as ‘World Economic Outlook’?
(a) The International Monetary Fund
(b) The United Nations Development Programme
(c) The World Economic Forum
(d) The World Bank

IAS Prelims 2015


Which one of the following issues the “Global Economic Prospects” report periodically?
(a) The Asian Development Bank
(b) The European Bank for Reconstruction and Development
(c) The US Federal Reserve Bank
(d) The World Bank

G-4 **
Countries India, Brazil, Japan, Germany

Purpose formed for securing permanent seat in United Nations Security


Council

Benefit They support each other bid for permanent membership

ES-2016
A group of countries called G4 is often mentioned in the news as contenders for a permanent
membership in the United Nations Security Council. Which of the following is not one of them?
(a) Brazil
(b) Germany
(c) Japan
(d) South Africa
Whatsapp No 7023213423 http://iasselfstudy.com/ 165

The Mekong-Ganga Cooperation (MGC) *


launched in 2000 at Vientiane, Lao PDR (Laos)

About MGC Both the Ganga and the Mekong are civilizational rivers, and the MGC
initiative aims to facilitate closer contacts among the people inhabiting
these two major river basins.

Purpose for cooperation in tourism, culture, education, as well as transport and


communications

Countries (6) India and five ASEAN countries: Cambodia, Lao PDR (Laos), Myanmar,
Thailand and Vietnam

IAS Prelims 2015


In the Mekong-Ganga Cooperation, an initiative of six countries, which of the following is/are not a
participant/ participants?
1. Bangladesh
2. Cambodia
3. China
4. Myanmar
5. Thailand
Select the correct answer using the code given below.
(a) 1 only
(b) 2, 3 and 4
(c) 1 and 3
(d) 1, 2 and 5
Whatsapp No 7023213423 http://iasselfstudy.com/ 166

International Organization for Standardization (ISO)


About ISO ISO is an independent, non-governmental international organization
with a membership of 163 national standards bodies

HQ Geneva, Switzerland

Popular standards ISO 9000 Quality management

ISO 14000 Environmental management

ISO 26000 Social responsibility

ISO 22000 Food safety management

ISO 27001 Information security management

ISO 45001 Occupational health and safety

India’s membership India is member

ES-2015
The International Organization for Standardization (ISO) is located in:
(a) Geneva
(b) London
(c) Rome
(d) Vienna

IAS Prelims 2005


Consider the following statements:
1. The Headquarters of the International Organisation for Standardization are located in Rome.
2. ISO 9000 is related to the quality management system and standards.
3. ISO 14000 relates to environmental management system standards.
Which of the statements is/are correct?
(a) 1, 2 and 3
(b) 3 only
(c) 2 and 3
(d) None

Organisation for Economic Co-operation and Development (OECD) *


History Created in 1960 by 18 European countries plus the United States
and Canada for economic development.

Mission The mission of the OECD is to promote policies that will improve
the economic and social well-being of people around the world

HQ Paris, France

Member countries 35 (India is not a member)


Whatsapp No 7023213423 http://iasselfstudy.com/ 167

ES-2011
The Headquarters of which one of the following is located in Paris?
(a) NATO
(b) OECD
(c) European Union
(d) UNIDO

Trans-Pacific Partnership (TPP) Agreement *


Purpose It would create a free-trade zone among 12 nations around the Pacific,
making it the world’s largest

Member countries United States, Australia, Brunei, Canada, Chile, Japan, Malaysia,
Mexico, New Zealand, Peru, Singapore, and Vietnam

On 23 January 2017, US President Donald Trump signed an executive


order to withdraw the United States from the agreement

GDP of member These 12 countries account for 40% of the world’s GDP
countries

IAS Prelims 2016


With reference to the ‘Trans-Pacific Partnership’, consider the following statements:
1. It is an agreement among all the Pacific Rim countries except China and Russia.
2. It is a strategic alliance for the purpose of maritime security ONLY
Which of the statements given above is/are correct?
(a) 1 only
(b) 2 only
(c) Both 1 and 2
(d) Neither 1 nor 2

CDS 2017
Which of the following statements about the Trans-Pacific Partnership (TPP) is / are correct?
1. The TPP was signed by 12 Pacific Rim nations in the year 20I5.
2. The TPP is likely to be a game-changer in global trade as member countries account for about 40
percent of global GDP.
3. India is a founder member of TPP.
Select the correct answer using the code given below.
(a) I, 2 and 3
(b) I and 2 only
(c) 2 and 3 only
(d) 1 only
Whatsapp No 7023213423 http://iasselfstudy.com/ 168

The Comprehensive Nuclear-Test-Ban Treaty (CTBT) *


About treaty It bans all nuclear explosions - everywhere, by everyone.

It curbs the development of new nuclear weapons and the improvement


of existing nuclear weapon designs.
Negotiated at The Treaty was negotiated at the Conference on Disarmament in Geneva
and adopted by the United Nations General Assembly.

It was opened for signature on 24 September 1996 Since then, the


Treaty has reached near-universality.

182 countries have signed the Treaty. 154 countries have ratified the
Treaty.
Why has the The Treaty’s entry into force depends on 44 specific States that must have
Treaty not signed and ratified the Treaty.
entered into
These States had nuclear facilities at the time the Treaty was negotiated
force yet?
and adopted.

When the Treaty enters into force it provides a legally binding norm
against nuclear testing

Status of 44 36 of these States have signed and ratified the Treaty.


specific States
5 States have signed but not ratified the Treaty: China, Egypt, Iran,
Israel, and the United States. (The United States and China are the only remaining
NPT Nuclear Weapon States that have not ratified the CTBT)

India, North Korea and Pakistan even have not signed the Treaty

Why ratification The ratification symbolizes the official sanction of a treaty to make it legally
required binding for the government of a country.

This process involves the treaty’s adoption by the legislature of a country


such as the parliament

Why India not It is discriminatory in nature as it gives advantage to the nuclear weapon
signed CTBT holders.

CTBT bans all nuclear explosions but is silent on nuclear disarmament.

India wants the world nuclear weapon free.

CDS-2009
Consider the following statements about Comprehensive Test Ban Treaty (CTBT) :
1. The treaty comes into force only if and when all nuclear capable countries sign it.
2. Iraq and India have not signed the treaty.
Which of the statements given above is/ are correct?
(a) 1 only
(b) 2 only
(c) Both 1 and 2
(d) Neither 1 nor 2
Whatsapp No 7023213423 http://iasselfstudy.com/ 169

Nuclear Non-proliferation Treaty (NPT) *


When came March 1970
into force

Purpose The treaty covers three mutually reinforcing pillars—disarmament,


nonproliferation, and peaceful uses of nuclear energy—and is the basis for
international cooperation on stopping the spread of nuclear weapons.

No of countries 190
subscribed

Classification 190 states-parties are classified in two categories:


of countries
Nuclear-weapon states (NWS)—US, Russia, China, France, and UK,

Non-nuclear-weapon states (NNWS).

Obligation Countries with nuclear weapons will move towards disarmament;

countries without nuclear weapons will not acquire them; and

all countries can access peaceful nuclear technology.

Countries With its near-universal membership, the NPT has the widest adherence of
outside the any arms control agreement, with only North Korea, South Sudan, India,
treaty Israel, and Pakistan remaining outside the treaty

Review The Treaty allows for the Parties to gather every five years to review its
Process operation

IAEA Treaty establishes a safeguards system under the responsibility of


the International Atomic Energy Agency (IAEA).

Safeguards are used to verify compliance with the Treaty through


inspections conducted by the IAEA.

The Treaty promotes cooperation in the field of peaceful nuclear technology


and equal access to this technology for all States parties, while safeguards
prevent the diversion of fissile material for weapons use.

Why India not NPT came into force in 1970 and India first tested nuclear weapon in 1974.
signed NPT It does not regard India as Nuclear weapon states.

Joining NPT as a NNWS would require India unilaterally giving up its nuclear
weapons hence India opposed the NPT because It is discriminatory against
the NNWS

IAS Prelims 1991


India is opposed to the Nuclear Non-proliferation Treaty (NPT) because
(a) It is discriminatory against the nonnuclear powers
(b) It forbids India to use nuclear energy for peaceful purposes
(c) It aims at making South Asia a nuclear free zone
(d) It allows Pakistan to develop its own nuclear capability
Whatsapp No 7023213423 http://iasselfstudy.com/ 170

IAS Prelims 2015


Consider the following countries
1. China
2. France
3. India
4. Israel
5. Pakistan
Which among the above are Nuclear Weapons States as recognized by the Treaty on the
Nonproliferation of Nuclear Weapons, commonly known as Nuclear Non-Proliferation Treaty (NPT)?
a) 1 and 2 only
b) 1, 3, 4 and 5 only
c) 2, 4 and 5 only
d) 1, 2, 3, 4 and 5

The Nuclear Suppliers Group (NSG) *


About NSG NSG is a group of nuclear supplier countries that seeks to contribute to
the non-proliferation of nuclear weapons through the implementation of
two sets of Guidelines for nuclear exports and nuclear-related exports.

Reason for NSG was created following the explosion in 1974 of a nuclear device by a
Creation non-nuclear-weapon State (actually India), which demonstrated that
nuclear technology transferred for peaceful purposes could be misused.

Non- whereby a supplier, authorizes a transfer only when satisfied that the
Proliferation transfer would not contribute to the proliferation of nuclear weapons.
Principle

Working of NSG NSG works on the basis of consensus.

Overall responsibility for activities lies within NSG Plenary meetings that
are held once a year.

Participating Argentina, Australia, Austria, Belarus, Belgium, Brazil, Bulgaria, Canada,


Governments China, Croatia, Cyprus, Czech Republic, Denmark, Estonia, Finland,
(48) France, Germany, Greece, Hungary, Iceland, Ireland, Italy, Japan,
Kazakhstan, South Korea, Latvia, Lithuania, Luxembourg, Malta, Mexico,
Netherlands, New Zealand, Norway, Poland, Portugal, Romania, Russian
Federation, Serbia, Slovakia, Slovenia, South Africa, Spain, Sweden,
Switzerland, Turkey, Ukraine, United Kingdom, And United States

India’s Position China has opposed India’s bid to get NSG membership on the ground that
it has not signed the NPT.

It stated that if India is admitted, Pakistan should also be admitted


simultaneously.

Benefits on Membership of the NSG would enable India to have enhanced and
joining uninterrupted access to nuclear technology, fuel and materials required
for its expanding civil nuclear programme..
Whatsapp No 7023213423 http://iasselfstudy.com/ 171

CAPF-2010
Consider the following statements :
1. Nuclear power is the fourth largest source of electricity in India after thermal, hydro and renewable
sources.
2. India is a non-signatory of the Nuclear Non Proliferation Treaty.
3. India is a member of Nuclear Suppliers Group.
Which of the statements given above is/ are correct?
(a) 1, 2 and 3
(b) 1 and 2 only
(c) I only
(d) 2 and 3 only

IAS Prelims 2008


Consider the following statements:
1. The Nuclear Suppliers Group has 24 countries as the members.
2. India is a member of the Nuclear Suppliers Group.
Which of the statements given above is/are correct ?
(a) 1 Only (b) 2 Only
(c) Both 1 and 2 (d) Neither 1 nor 2

Bank for International Settlements (BIS)


About BIS Established on 17 May 1930, BIS is the world's oldest international
financial organization.

The BIS has 60 member central banks, representing countries from


around the world that together make up about 95% of world GDP

Mission The mission of the BIS is to serve central banks in their pursuit of
monetary and financial stability, to foster international cooperation
in those areas and to act as a bank for central banks

HQ Basel, Switzerland

Capital and The Bank's capital is held by central banks only.


membership
60 central banks and monetary authorities are currently members of
the BIS and have rights of voting and representation at General
Meetings

RBI membership RBI is member

Raghuram Rajan (Ex Rajan was the first Indian central bank Governor to become the vice-
RBI Governor) chairman of BIS.
Whatsapp No 7023213423 http://iasselfstudy.com/ 172

The Heart of Asia-Istanbul Process (HoA) *


Founded HoA was founded on November 2nd, 2011 in Istanbul, Turkey

About HoA It provides a platform for sincere and results-oriented regional


cooperation by placing Afghanistan at its center, in recognition of the
fact that a secure and stable Afghanistan is vital to the prosperity of
the Heart of Asia region

Participating Afghanistan, Azerbaijan, China, India, Iran, Kazakhstan, Kyrgyz


countries (14) Republic, Pakistan, Russia, Saudi Arabia, Tajikistan, Turkey,
Turkmenistan, United Arab Emirates

Three main pillars The Heart of Asia-Istanbul Process has three main pillars, which work
together to build regional consensus on how we can achieve our shared
goals. They include:

1. Political Consultations
2. Confidence Building Measures (CBMs)
3. Cooperation with Regional Organizations

Heart of Asia – Kabul, Afghanistan


Istanbul Process
Desk

The India Development Foundation of Overseas Indians (IDF-OI)*


Set up by Government of India in 2008 as a not-for-profit Trust

Purpose It facilitates philanthropic contributions by Overseas Indians into social


and development projects in India.

The Trust is exempt from provisions of the Foreign Contributions


Regulation Act (FCRA), 2010 of the Ministry of Home Affairs which
enables IDF-OI to receive foreign contributions

Administered by Board of Trustees, chaired by Smt. Sushma Swaraj, Minister of


External Affairs

Charges/Fee IDF-OI does not recover any administrative cost from contributions
received from Overseas Indians.
Whatsapp No 7023213423 http://iasselfstudy.com/ 173

International Atomic Energy Agency (IAEA) *


About IAEA It was set up as the world's "Atoms for Peace" organization in 1957
within the United Nations family.

The IAEA is the world's centre for cooperation in the nuclear field

Function It works with its Member States and multiple partners worldwide to
promote the safe, secure and peaceful use of nuclear technologies.

Hence contribute to international peace and security and the United


Nations' Sustainable Development Goals

Reporting In terms of its Statute, IAEA reports annually to the UN General


Assembly and,

when appropriate, to the UN Security Council regarding States' non-


compliance with safeguards obligations, as well as on matters relating
to international peace and security.

HQ Vienna International Centre in Vienna, Austria

India’s membership India is member

CDS-2009
Consider the following statements about IAEA:
l. It was set up as the world's Atoms for Peace organization in 1957.
2. The IAEA Secretariat is headquartered at the Vienna International Centre in Vienna, Austria.
3. In terms of its Statute, the IAEA reports annually to the UN General Assembly.
Which of the statements given above is/are correct?
(a) 1, 2 and 3
(b) 1 only
(c) 2 and 3 only
(d) 3 only

Quadrilateral Coordination Group (QCG) for Afghan Peace


Formed by Afghanistan, China, Pakistan and the United States for Afghan Peace and
reconciliation process

Quadrilateral Cooperation and Coordination Mechanism (QCCM) to counter


terrorism
The armed forces of Afghanistan, China, Pakistan and Tajikistan have formed a Quadrilateral
Cooperation and Coordination Mechanism (QCCM) to counter terrorism.
Whatsapp No 7023213423 http://iasselfstudy.com/ 174

The Indian Ocean Rim Association (IORA) *


launched at First Ministerial Meeting in Mauritius on 6-7 March 1997.

This meeting adopted the IORA Charter

Priority Areas To promote the sustained growth and balanced development of the
region and of the Member States, and to create common ground for
regional economic co-operation, six priority areas were identified:

(i) Maritime Safety & Security,


(ii) Trade & Investment Facilitation,
(iii) Fisheries Management,
(iv) Disaster Risk Management,
(V) Academic, Science & Technology,
(vi) Tourism & Cultural Exchanges

Member States Australia, Bangladesh, Comoros, India, Indonesia, Iran, Kenya,


(21) Madagascar, Malaysia, Mauritius, Mozambique, Oman, Seychelles,
Singapore, Somalai, South Africa, Sri Lanka, Tanzania, Thailand, UAE
and Yemen

Dialogue China
Partners (7)
Egypt

France

Japan

United Kingdom

United States

Germany

IAS Prelims 2015


With reference to ‘Indian Ocean Rim Association for Regional Cooperation (IOR-ARC)’, Consider the
following statements:
1. It was established very recently in response to incidents of piracy and accidents of oil spills
2. It is an alliance meant for maritime security only
Which of the following statements given above is/are correct?
a) 1 only
b) 2 only
c) Both 1 and 2
d) Neither 1 nor 2
Whatsapp No 7023213423 http://iasselfstudy.com/ 175

Group of Twenty (G20) *


History The G20 started in 1999 as a meeting of Finance Ministers and Central
Bank Governors in the aftermath of the Asian financial crisis.

In 2008, the first G20 Leaders’ Summit was held in Washington D.C.
(United States of America), and the group played a key role in
responding to the global financial crisis.

About G20 The Group of Twenty (G20) is the central forum for international
cooperation on financial and economic issues

members Argentina, Australia, Brazil, Canada, China, France, Germany, India,


Indonesia, Italy, Japan, Republic of Korea, Mexico, Russia, Saudi Arabia,
South Africa, Turkey, the United Kingdom, the United States and the
European Union

Meetings G20 leaders meet annually.

In addition, Finance Ministers and Central Bank Governors meet


regularly during the year to discuss ways to strengthen the global
economy, reform international financial institutions, improve financial
regulation and implement the key economic reforms that are needed in
each member economy.

G20 Summit In 2017, the G20 Summit will be held in Hamburg by the German
Presidency

W20 The Turkish Presidency in 2015 proposed establishing the Women-20


(W20) as a G20 engagement group to focus on promoting gender
inclusiveness and gender equality.

The mission of the W20 is to promote global gender-inclusive economic


growth

CDS-2013
Which one among the following regarding G-20 is not correct ?
A A group of developed countries
B. An integral part of the United Nations
C. Outside the World Bank and IMF
D. An offshoot of G-7

ES-2016
Which of the following is/are among the main objective/objectives of G20 group of countries?
1. Discussing key issues in the global economy
2. Promoting international financial stability
Select the correct answer using the code given below
(a) 1 only
(b) 2 only
(c) Both 1 and 2
(d) Neither 1 nor 2
Whatsapp No 7023213423 http://iasselfstudy.com/ 176

United Nations Economic and Social Commission for Asia and the Pacific
(ESCAPE) *
About ESCAPE ESCAP is the regional development arm of the United Nations for
the Asia-Pacific region established in 1947

HQ Bangkok, Thailand

India’s membership INDIA is a member

CDS-2016
The headquarters of 'Economic and Social Commission for Asia and the Pacific' is located at
(a) Singapore
(b) Manila
(c) Bangkok
(d) Hong Kong

International Solar Alliance (ISA) *


launched at UN Climate Change Conference in Paris

launched by India and France

Purpose ISA will be dedicated to promotion of solar energy for making solar
energy a valuable source of affordable and reliable green and clean
energy in 121 member countries located in between Tropic of
Cancer and Tropic of Capricorn

HQ Gurgaon (ISA is India’s first international and inter-governmental


organization headquartered in India)

IAS Prelims-2016
Consider the following statements:
1. The International Solar Alliance was launched at the United Nations Climate Change Conference
in 2015.
2. The Alliance includes all the member countries of the United Nations.
Which of the statements given above is/are correct?
(a) 1 only (b) 2 only (c) Both 1 and 2 (d) Neither 1 nor 2

ES-2016
Recently, the Interim Secretariat of the International Solar Alliance was inaugurated at :
(a) Gurgaon
(b) Panipat
(c) Vadodara
(d) Visakhapatnam
Whatsapp No 7023213423 http://iasselfstudy.com/ 177

India-Brazil-South Africa Dialogue Forum *


About IBSA IBSA is a trilateral developmental initiative amongst the governments
of India, Brazil and South Africa to promote, inter-alia, South-South
cooperation and exchange

Establishment IBSA was formalised by the Brasilia Declaration of 6 June 2003, which
mentions India, Brazil and South Africa's democratic credentials, their
condition as developing nations and their capacity of acting on a
global scale as the main reasons for the three countries to come
together

Hague Code of Conduct [HCOC] **


About HCOC The HCOC is the only multilateral transparency and confidence
building instrument concerning the spread of ballistic missiles capable
of carrying weapons of mass destruction.

Voluntary The HCoC is a voluntary, legally non-binding.

By subscribing to the HCoC, members voluntarily commit themselves


politically to provide pre-launch notifications (PLNs) on ballistic missile
and space-launch vehicle launches (SLVs) and test flights.

Members 138 countries

India’s status India has joined the HCoC in 2016

Benefit India joining HCoC and Missile Technology Control Regime (MTCR)
would strengthen its case at the Nuclear Suppliers Group (NSG).

Is the HCoC Yes


endorsed by the
United Nations
?
Whatsapp No 7023213423 http://iasselfstudy.com/ 178

The Missile Technology Control Regime (MTCR) **


About MTCR MTCR is an informal political understanding among states that seek to
limit the proliferation of missiles and missile technology

Background The regime was formed in 1987 by the G-7 industrialized countries
(Canada, France, Germany, Italy, Japan, the UK, and the United
States).

There are currently 35 countries that are members of the MTCR

India’s status India has joined in 2016

Benefit MTCR membership is expected to ease the access to high technology


for India’s space programme

Relationship While there is no formal linkage, the activities of the MTCR are
between the consistent with the UN’s non-proliferation and export control efforts
MTCR and the
UN

CDS 2017
India became a member of which one of the following in 2016?
(a) Non-Proliferation Treaty
(b) Missile Technology Control Regime
(c) Nuclear Suppliers Group
(d) Wassenaar Arrangement

Forum for India–Pacific Islands Cooperation (FIPIC)


When Launched During Prime Minister, Narendra Modi's visit to Suva, Fiji in
November 2014 (First Summit).

14 island Cook Islands, Fiji, Kiribati, Marshall Islands, Micronesia, Nauru,


countries Niue, Palau, Papua New Guinea, Samoa, Solomon Islands, Tonga,
Tuvalu, and Vanuatu.

Second summit Jaipur

CDS-2016
The first summit of the Forum for India-Pacific Islands Cooperation (FIPIC) was held in :
(a) Jaipur
(b) Suva
(c) New Delhi
(d) Port Moresby
Whatsapp No 7023213423 http://iasselfstudy.com/ 179

The Organization of the Petroleum Exporting Countries (OPEC) *


About OPEC OPEC is a permanent intergovernmental organization of 13 oil-
exporting developing nations that coordinates and unifies the
petroleum policies of its Member Countries

Created at the Baghdad Conference in 1960

Founding Iran, Iraq, Kuwait, Saudi Arabia and Venezuela.


Members (5)

Other members Qatar, Libya, United Arab Emirates, Algeria, Nigeria, Ecuador,
(8) Angola, and Gabon

Purpose to secure fair and stable prices for petroleum producers;

an efficient, economic and regular supply of petroleum to consuming


nations; and

a fair return on capital to those investing in the industry.

HQ Vienna, Austria

ES-2009
Which one of the following is a member of OPEC?
(a) Vietnam
(b) Philippines
(c) Laos
(d) none of the above

IAS Prelims 2009


Other than Venezuela, which one among the following from South America is a member of OPEC ?
A. Argentina
B. Brazil
C. Ecuador
D. Bolivia

Gulf Cooperation council


Member Bahrain, Kuwait, Oman, Qatar, Saudi Arabia, and the United Arab
Emirates

HQ Riyadh, Saudi Arabia

IAS Prelims 2016


Which of the following is not a member of ‘Gulf Cooperation Council’?
(a) Iran (b) Saudi Arabia (c) Oman (d) Kuwait
Whatsapp No 7023213423 http://iasselfstudy.com/ 180

Eurasian Economic Union *


About EEU It has international legal personality and is established by
the Treaty on the Eurasian Economic Union

Member-States Russia, Armenia, Belarus, Kazakhstan and Kyrgyz Republic


(5)

East Asia Summit (EAS)


Established in 2005

About EAS It is an initiative of ASEAN.

It is a unique Leaders-led forum of 18 countries of the Asia-Pacific


region formed to further

the objectives of regional peace, security and prosperity.

Member 10 ASEAN Member States (i.e. Brunei Darussalam, Cambodia,


countries Indonesia, Lao PDR, Malaysia, Myanmar, Singapore, Thailand,
Philippines and Vietnam), Australia, China, India, Japan, New
Zealand, Republic of Korea, Russia and the USA

India’s India is a founding member


membership

IAS Prelims 2015


India is a member of which of the following?
1. Asia-Pacific economic corporation.
2. Association of South-East Asian Nations.
3. East Asia Summit
Select the correct answer using the code given below.
a) 1 and 2 only
b) 3 only
c) 1, 2, and 3
d) India is a member of none of them
Whatsapp No 7023213423 http://iasselfstudy.com/ 181

The Financial Action Task Force (FATF) *


Establishment FATF is an inter-governmental body established in 1989 by the
Ministers of its Member jurisdictions

Objectives to set standards and promote effective implementation of legal,


regulatory and operational measures for combating money laundering,
terrorist financing and other related threats to the integrity of the
international financial system.

Secretariat Secretariat is located at the OECD Headquarters in Paris

India’s Indian is a member


membership

The Australia Group (AG)


About AG AG is an informal forum of countries which, through the harmonization
of export controls, seeks to ensure that exports do not contribute to
the development of chemical or biological weapons

India’s Position India is not a participant

Wassenaar Arrangement
About The Wassenaar Arrangement has been established in order to
Wassenaar contribute to regional and international security and stability, by
promoting transparency and greater responsibility in transfers of
conventional arms and dual-use goods and technologies, thus
preventing destabilizing accumulations.

Aim to prevent the acquisition of these items by terrorists.

India’s Position India is not a participant


Whatsapp No 7023213423 http://iasselfstudy.com/ 182

Chapter 10

Govt schemes related to Economy

JAM number trinity **


Jan Dhan account, Aadhaar and Mobile –to effectively target public resources to those who
need it most.

Atal Mission for Rejuvenation and Urban Transformation (AMRUT)


Launched in June 2015

Purpose Providing basic services (e.g. water supply, sewerage, urban transport)
to households and build amenities in cities which will improve the quality
of life for all, especially the poor and the disadvantaged is a national
priority

Target 500 cities

Stand-Up India Scheme launched on 5th April, 2016


About it Stand-Up India Scheme facilitates bank loans between Rs 10 lakh and Rs
1 Crore to at least one Scheduled Caste (SC) or Scheduled Tribe (ST)
borrower and at least one woman borrower per bank branch for setting up
a greenfield (i.e. new) enterprise in manufacturing, services or the trading
sector.

In case of non-individual enterprises at least 51% of the shareholding and


controlling stake should be held by either an SC/ST or woman
entrepreneur.

Refinance through Small Industries Development Bank of India (SIDBI) with an


initial amount of Rs. 10,000 crore.

Operated by The scheme would be operated by all the branches of Scheduled


Commercial Banks.
Whatsapp No 7023213423 http://iasselfstudy.com/ 183

IAS Prelims 2016


With reference to ‘stand up India scheme’, which of the following statement is/are correct?
1. Its purpose is to promote entrepreneurship among SC/ST and women entrepreneurs.
2. It provides for refinance through SIDBI.
Select the correct answer using the code given below.
(a) 1 only
(b) 2 only
(c) Both 1 and 2
(d) Neither 1 nor 2

Pradhan Mantri Jan-Dhan Yojana (PMJDY) *


Launched in August, 2014

National mission PMJDY is National Mission for Financial Inclusion to ensure access to
financial services

Zero balance Account can be opened in any bank branch or Business


account Correspondent (Bank Mitr) outlet with Zero balance.

Features Accidental insurance cover of Rs. 1 lac

life cover of Rs. 30,000/-

an overdraft facility upto Rs. 5000/- after satisfactory operation of


the account for 6 months. Overdraft facility is available in only one
account per household, preferably lady of the household

Beneficiaries of Government Schemes will get Direct Benefit Transfer


in these accounts.

IAS Prelims 2015


Pradhan Mantri Jan Dhan Yojana has been launched for
a) providing housing loan to poor people at cheaper interest rates
b) Promoting women’s Self Help Groups in backward areas
c) promoting financial inclusion in the country
d) providing financial help to marginalised communities

NDA-2016
Which of the following are the features of Pradhan Mantri Jan Dhan Yojana ?
1. Ensuring financial inclusion of the poor
2. Enhancing financial literacy
3.Provision for accidental insurance to account holders
4. Allowing bank accounts with zero Balance
Select the correct answer using the code given below:
(a) 1 and 4 only
(b) 1, 2 and 3 only
(c) 2 and 4·only
(d) 1,2, 3 and 4
Whatsapp No 7023213423 http://iasselfstudy.com/ 184

Shyama Prasad Mukherji Rurban Mission to drive economic, social and


infrastructure development in rural areas
Launched in September 2015

Aim development of rural growth clusters by provisioning of economic


activities, developing skills & local entrepreneurship and providing
infrastructure amenities.

The Rurban Mission will thus develop a cluster of Smart Villages.

Who will identify The State Governments would identify the clusters in accordance with
the clusters the Framework for Implementation prepared by the Ministry of Rural
Development

The clusters will be geographically contiguous Gram Panchayats with


a population of about 25000 to 50000 in plain and coastal areas and
a population of 5000 to 15000 in desert, hilly or tribal areas.

Target to create 300 Rurban growth clusters over the next 3 years, across
the country. In the first phase, 100 clusters will be taken up

Price Stabilisation Fund *


About PSF PSF as a Central Sector Scheme, with a corpus of Rs.900 crores, to
support market interventions for price control of perishable agri-
horticultural commodities.

Initial use Initially the fund is proposed to be used for onion and potato only.

Use of Fund PSF will be used to advance interest free loan to State
Governments and Central agencies to support their working capital
and other expenses on procurement and distribution interventions
for such commodities.

Procurement of these commodities will be undertaken directly from


farmers or farmers’ organizations at farm gate/mandi and made
available at a more reasonable price to the consumers.
Whatsapp No 7023213423 http://iasselfstudy.com/ 185

National Agriculture Market (NAM) *


Launched in April 2016

About NAM NAM is a pan-India electronic trading portal which networks the existing
APMC mandis to create a unified national market for agricultural
commodities.

The NAM Portal provides a single window service for all APMC related
information and services.

Functioning The NAM Portal provides a single window service for all APMC related
information and services e.g. commodity arrivals & prices, buy & sell
trade offers etc.

While material flow (agriculture produce) continue to happen through


mandis, an online market reduces transaction costs and information
asymmetry.

Why NAM Agricultural marketing is regulated by the States’ Agricultural Produce


Marketing Regulation (APMR) Acts.

State is divided into several market areas, each of which is administered


by a separate Agricultural Produce Marketing Committee (APMC) which
imposes its own marketing regulation (including fees).

This fragmentation of markets, even within the State, hinders free flow
of agri commodities from one market area to another and multiple
handling of agri-produce and multiple levels of mandi charges ends up
escalating the prices for the consumers without commensurate benefit
to the farmer.

NAM addresses these challenges by creating a unified market through


online trading platform, both, at State and National level and promotes
uniformity, streamlining of procedures across the integrated markets,
removes information asymmetry between buyers and sellers and
promotes real time price discovery, based on actual demand and
supply, promotes transparency in auction process, and access to a
nationwide market for the farmer, with prices commensurate with
quality of his produce and online payment and availability of better
quality produce and at more reasonable prices to the consumer.

IAS Prelims 2015


In India, markets in agricultural products are regulated under the
a) Essential Commodities Act, 1955
b) Agricultural Produce Market Committee Act enacted by States
c) Agricultural Produce (Grading and Marking) Act, 1937
d) Food Products Order, 1956 and Meat and Food Products Order, 1973
Whatsapp No 7023213423 http://iasselfstudy.com/ 186

Agricultural Marketing and Farmer Friendly Reforms Index **


Launched by NITI Aayog in Oct 2016

Range 0 to 100

“0” implies no reforms and “100” implies complete reforms in the selected
areas.

States and UTs have been ranked in terms of the score of the index.

First rank Maharashtra achieved first rank in implementation of various reforms.

Gujarat ranks second then Rajasthan and Madhya Pradesh.

Sovereign Gold Bonds Scheme **


Product name Sovereign Gold Bond 2016-17 – Series IV

Issuance To be issued by Reserve Bank India on behalf of the Government of


India.

Eligibility The Bonds will be restricted for sale to resident Indian entities including
individuals, HUFs, Trusts, Universities and Charitable Institutions.

Denomination The Bonds will be denominated in multiples of gram(s) of gold with a


basic unit of 1 gram.

Tenor The tenor of the Bond will be for a period of 8 years with exit option
from 5th year to be exercised on the interest payment dates.

Minimum size Minimum permissible investment will be 1 grams of gold.

Maximum limit The maximum amount subscribed by an entity will not be more than
500 grams per person per fiscal year (April-March).

A self-declaration to this effect will be obtained.

Joint holder In case of joint holding, the investment limit of 500 grams will be
applied to the first applicant only.

Issue price Price of Bond will be fixed in Indian Rupees on the basis of simple
average of closing price of gold of 999 purity published by the India
Bullion and Jewellers Association Limited for the week (Monday to
Friday) preceding the subscription period.
Whatsapp No 7023213423 http://iasselfstudy.com/ 187

The issue price of the Gold Bonds will be Rs. 50 per gram less than the
nominal value.

Payment Payment for the Bonds will be through cash payment (upto a maximum
option of Rs. 20,000) or demand draft or cheque or electronic banking.

Issuance form The Gold Bonds will be issued as Government of India Stocks under GS
Act, 2006.

The investors will be issued a Holding Certificate for the same.

The Bonds are eligible for conversion into demat form.

Redemption The redemption price will be in Indian Rupees based on previous


price week’s (Monday-Friday) simple average of closing price of gold of 999
purity published by IBJA.

Sales channel Bonds will be sold through banks, Stock Holding Corporation of India
Limited (SHCIL), designated post offices as may be notified and
recognised stock exchanges viz., National Stock Exchange of India
Limited and Bombay Stock Exchange, either directly or through agents.

Interest rate The investors will be compensated at a fixed rate of 2.50% per
annum payable semi-annually on the nominal value.

Collateral Bonds can be used as collateral for loans.

The loan-to-value (LTV) ratio is to be set equal to ordinary gold loan


mandated by the Reserve Bank from time to time.

Tax treatment The interest on Gold Bonds shall be taxable as per the provision of
Income Tax Act, 1961 (43 of 1961).

The capital gains tax arising on redemption of SGB to an individual


has been exempted.

Tradability Bonds will be tradable on stock exchanges within a fortnight of the


issuance on a date as notified by the RBI.

Benefit The scheme will help in reducing the demand for physical gold by
shifting a part of the estimated 300 tons of physical bars and coins
purchased every year for Investment into gold bonds.

Since most of the demand for gold in India is met through imports, this
scheme will, ultimately help in maintaining the country's Current
Account Deficit within sustainable limits.
Whatsapp No 7023213423 http://iasselfstudy.com/ 188

Gold Monetization Schemes *


Objective to reduce the country's reliance on the import of gold to meet
domestic demand.

Benefit scheme will help in mobilizing the large amount of gold lying as an
idle asset with households, trusts and various institutions in India
and put this gold into productive use.

Mobilized gold will also supplement RBI’s gold reserves and will
help in reducing the government's borrowing cost

IAS Prelims 2016


What is/are the purpose/purposes of Government’s ‘Sovereign Gold Bond Scheme’ and ‘Gold
Monetization Scheme’?
1. To bring the idle gold lying with Indian households into the economy
2. To promote FDI in the gold and jewellery sector
3. To reduce India’s dependence on gold imports
Select the correct answer using the code given below.
(a) 1 only
(b) 2 and 3 only
(c) 1 and 3 only
(d) 1, 2 and 3

The Pradhan Mantri Gram Sadak Yojana (PMGSY) *


Share of Govt 100% Centrally Sponsored Scheme

Objective to provide Connectivity, by way of an All-weather Road to the


eligible unconnected Habitations in the rural areas with a population
of 500 persons and above in Plain areas.

In respect of the Hill States (North-East, Sikkim, Himachal Pradesh,


Jammu & Kashmir and Uttarakhand), the Desert Areas ,the Tribal
areas and Selected Tribal and Backward Districts the objective
would be to connect eligible unconnected Habitations with a
population of 250 persons and above.

Target preponed The Government has brought forward the target date by three years
from 2022 to 2019 to achieve complete rural connectivity through
all-weather roads under Pradhan Mantri Gram Sadak Yojana, PMGSY

Only rural areas The PMGSY shall cover only the rural areas. Urban roads are
excluded from the purview of this Programme
Whatsapp No 7023213423 http://iasselfstudy.com/ 189

ATAL PENSION YOJANA (APY)


Launched in May, 2015

Applicability all citizen of India aged between 18-40 years

Focus all citizens in the unorganized sector

Guaranteed there is guaranteed minimum monthly pension for the subscribers


pension ranging between Rs. 1000 and Rs. 5000 per month

Govt. Govt will co-contribute 50% of the subscriber’s contribution or Rs.


Contribution 1000 per annum, whichever is lower for a period of 5 years, i.e., from
Financial Year 2015-16 to 2019-20, who join the NPS between the
period 1st June, 2015 and 31st December, 2015 and who are not
members of any statutory social security scheme and who are not
income tax payers.

Monthly pension Subscribers would receive the fixed minimum pension of Rs.
1000/2000/3000/4000/5000 per month, at the age of 60 years,
depending on their contributions.

In case of death Monthly pension would be available to his spouse and on the death of
of the Subscriber both, the pension corpus would be returned to the nominee of the
subscriber

Administration The scheme is administered by the Pension Fund Regulatory and


Development Authority (PFRDA) through (NPS) National Pension
System architecture

Whether bank Yes, APY is open to all saving bank account holders. A person first
account must open saving bank account
mandatory

IAS Prelims 2016


Regarding ‘Atal Pension Yojana’, which of the following statements is/are correct?
1. It is a minimum guaranteed pension scheme mainly targeted at unorganized sector workers.
2. Only one member of a family can join the scheme.
3. Same amount of pension is guaranteed for the spouse for life after subscriber’s death.
Select the correct answer using the code given below.
(a) 1 only
(b) 2 and 3 only
(c) 1 and 3 only
(d) 1, 2 and 3
Whatsapp No 7023213423 http://iasselfstudy.com/ 190

Pradhan Mantri Jeevan Jyoti Bima Yojana (PMJJBY) *


Launched in May, 2015

About scheme is a one year life insurance scheme, renewable from year to year,
offering coverage for death due to any reason and is available to
people in the age group of 18 to 50 years ( life cover upto age
55) having a savings bank account who give their consent to join
and enable auto-debit

Risk Coverage life cover of Rs. 2 lakhs is available for a one year period
stretching from 1st June to 31st May at a premium of Rs.330/-
per annum per member and is renewable every year.

Pradhan Mantri Suraksha Bima Yojana (PMSBY) *


Launched in May, 2015

Eligibility people in age group 18 to 70 years with bank account.

Premium Rs.12 per annum

Payment Mode premium will be directly auto-debited by the bank

Risk Coverage For accidental death and full disability - Rs.2 Lakh and for partial
disability – Rs.1 Lakh

 .

CORE OF THE CORE SCHEME **


1 National Social Assistance Progamme

2 Mahatma Gandhi National Rural Employment Guarantee Programme

3 Umbrella Scheme for Development of Schedule Castes

4 Umbrella Programme for Development of Scheduled Tribes

5 Umbrella Programme for Development of Minorities

6 Umbrella Programme for Development of Other Vulnerable Groups


Whatsapp No 7023213423 http://iasselfstudy.com/ 191

Saansad Adarsh Gram Yojana (SAANJHI)


Launched in October, 2014 on the birth anniversary of Jayaprakash Narayan

Goal to develop three Adarsh Grams by March 2019, of which one would be
achieved by 2016.

Thereafter, five such Adarsh Grams (one per year) will be selected and
developed by 2024.

Identification of The MP will identify one Gram Panchayat to be taken up immediately,


Gram and two others to be taken up a little later.

Lok Sabha MP has to choose a Gram Panchayat from within his/her


constituency and

Rajya Sabha MP a Gram Panchayat from the rural area of a district of


his/her choice in the State from which he/she is elected.

Nominated MPs may choose a Gram Panchayat from the rural area of
any district in the country.

CAPF-2015
Which one of the following statements with regard to the Saansad Adarsh Gram Yojana is NOT
correct ?
(a) It was launched in the year 2014 on the birth anniversary of Lal Bahadur Shastri
(b) Members of the Parliament will adopt villages to create model villages
© District Magistrates will act as coordinators
(d) A Rajya Sabha MP has to choose a Gram Panchayat in the State from which he or she was
elected
Whatsapp No 7023213423 http://iasselfstudy.com/ 192

Pradhan Mantri Awas Yojana (PMAY) | Housing for All 2022 Scheme ***
Launched in Nov 2016

National priority Housing for all has been proclaimed as a national priority for the
Government of India

About scheme The Government in the President's address announced - "By the time
the nation completes 75 years of its Independence (by 2022), every
family will have a pucca house with water connection, toilet facilities,
24x7 electricity supply and access”.

Components Housing for All implemented by Ministry of Rural Development


(Rural)

Housing for All Implemented by Ministry of Housing and Urban


(Urban) Poverty Alleviation.

Action plan To achieve the goal of Housing for All by 2022,

nearly 3 crore houses would be built in rural areas in the next 7 years
and about 2 crore houses in the urban areas.

PRADHAN MANTRI FASAL BIMA YOJANA


Launched in January 2016

Replacement It replaced existing two schemes viz. National Agricultural Insurance


Scheme (NAIS) as well as Modified NAIS (MNAIS) which had some
inherent drawbacks.

Farmers’ For rabi crops 1.5%


premium
For kharif crops 2%

For horticultural and 5%


commercial crops

Coverage Crop cycle- pre-sowing to post-harvest losses

IAS Prelims 2016


With referene to ‘Pradhan Mantri Fasal Bima Yojana’, consider the following statements:
1. Under this scheme, farmers will have to pay a uniform premium of two percent for any crop they
cultivate in any reason of the year.
2. This scheme covers post harvest losses arising out of cyclones and unseasonal rains.
Which of the statements given above is/are correct?
(a) 1 only (b) 2 only (c) Both 1 and 2 (d) Neither 1 nor 2
Whatsapp No 7023213423 http://iasselfstudy.com/ 193

Entire Country gets National Food Security Act coverage **


With the Kerala and Tamil Nadu rolling out the National Food Security Act (NFSA), now the
Act has been implemented in all the States and Union Territories.

As a result, now 81.34 crore persons will get wheat at Rs. 2/ kg and rice at Rs. 3/ kg.

Sagarmala Project
Launched in March, 2015

Aim To exploit the potential of India’s approximately 7,500 km long


coastline and 14,500km of potentially navigable waterways the
Sagarmala Programme aims to promote port-led development in the
country.

Objectives Port modernization & new port development, enhancing port


connectivity to hinterland, port led Industrialization and coastal
community development.

National As part of the programme, a National Perspective Plan (NPP) for the
Perspective Plan comprehensive development of the coastline and maritime sector
has been prepared

Sagarmala Incorporated under the Companies Act, 2013.


Development
The company has an initial Authorized Share Capital of Rs. 1,000
Company
Crore and a subscribed share capital of Rs. 90 Crore.

Objective of the to identify port-led development projects under the Sagarmala


company Programme and provide equity support for the project Special
Purpose Vehicles (SPVs) set up by the Ports / State / Central
Ministries and funding window and /or implement only those residual
projects which cannot be funded by any other means / mode.

Administrative Ministry of Shipping


ministry
Whatsapp No 7023213423 http://iasselfstudy.com/ 194

Pradhan Mantri Kaushal Vikas Yojana **


Investment Rs.12000 crore

Target to impart skilling to one crore people over the next four years (2016-2020)

Fresh PMKVY will impart fresh training to 60 lakh youths and certify skills of 40
training lakh persons acquired non-formally under the Recognition of Prior Learning
(RPL).

Swachh Bharat Mission **


Launched on October 2, 2014 (Birth Anniversary of Mahatma Gandhi)

Objective to achieve universal sanitation coverage, improve cleanliness and


eliminate open defecation in the country by October 2, 2019 as a
fitting tribute to the 150thBirth Anniversary of Mahatma Gandhi

Sub missions Swachh Bharat Mission (Gramin)-for rural areas under the Ministry of
Drinking Water and Sanitation (MDWS) and

Swachh Bharat Mission (Urban)- for urban areas under the Ministry of
Urban Development

Rastiya Swachta Fund

Open defecation First Sikkim


free states
Second Himachal Pradesh

Third Kerala
Whatsapp No 7023213423 http://iasselfstudy.com/ 195

National Apprenticeship Promotion Scheme (NAPS) **


Launched in August 2016 to promote apprenticeship

Investment Rs. 10,000 crore

Target 50 lakh apprentices to be trained by 2019-20

Components The scheme has the two components:

Govt will reimburse 25% of prescribed stipend subject to a


maximum of Rs. 1500/- per month per apprentice to all employers
who engage apprentices.

Govt will reimburse cost of basic training ( upto a limit of Rs. 7500/-
for a maximum of 500 hours/3 months) to Basic Training Providers
(BTPs) in respect of apprentices who come directly for apprenticeship
training without any formal training.

Ujwal DISCOM Assurance Yojana (UDAY)


Launched in Nov 2015

Purpose for operational and financial turnaround of State owned Power


Distribution Companies (DISCOMs).

Aim The Scheme aims to reduce the interest burden, reduce the cost of
power, reduce power losses in Distribution sector, and improve
operational efficiency of DISCOMs.

Optional scheme UDAY is optional for all States

IAS Prelims 2016


Which one of the following is a purpose of `UDAY’, a scheme of the Government?
(a) Providing technical and financial assistance to start-up entrepreneurs in the field of renewable
sources of energy
(b) Providing electricity to every household in the country by 2018
(c) Replacing the coal-based power plants with natural gas, nuclear, solar, wind and tidal power
plants over a period of time
(d) Providing for financial turnaround and revival of power distribution companies
Whatsapp No 7023213423 http://iasselfstudy.com/ 196

National LED programme - Unnat Jyoti by Affordable LEDs for All (UJALA)
Launched in January 2015

Purpose/target Replacement of 77 crore incandescent lamps with LED bulbs by March


2019.

UJALA is the largest non-subsidised LED programme in the world.

Implemented Energy Efficiency Services Limited (EESL) a joint venture PSU


through

Why LED LED bulbs consume half the energy as that of CFLs and one tenth as
that of incandescent bulbs.

Pradhan Mantri Ujjwala Yojana (PMUY)


Launched on 1st May, 2016 in Ballia, Uttar Pradesh

About scheme 5 Cr LPG connections will be provided to BPL families with a


support of Rs.1600 per connection in the next 3 years

Women’s Ensuring women’s empowerment, especially in rural India, the


empowerment connections will be issued in the name of women of the
households.

Identification of will be done through Socio Economic Caste Census Data.


the BPL families

Investment Rs. 8000 Cr


Whatsapp No 7023213423 http://iasselfstudy.com/ 197

The Pradhan Mantri Rojgar Protsahan Yojana (PMRPY) *


Launched in August 2016

Announced in Budget 2016-17

Purpose to incentivise employers for generation of new employment

Incentive Government of India will be paying the 8.33% EPS contribution of


the employer for the new employment

For the textile Government will also be paying the 3.67% EPF contribution
(apparel) sector

Period of contribution 3 years, provided workers continue in the employment by the


same employer.

Implemented through Ministry of Labour and Employment

Coverage workers earning wages upto Rs. 15,000/- per month

Interest Subvention Scheme *


About scheme The Central Government will provide interest subvention of 5% per
annum to all farmers for short term crop loan upto one year for loan
upto Rs. 3 lakhs borrowed by them during the year 2016-17.

Farmers will thus have to effectively pay only 4% as interest instead of


9%

Non-payment In case farmers do not repay the short term crop loan in time they
of loan would be eligible for interest subvention of 2% as against 5% available
above.
Whatsapp No 7023213423 http://iasselfstudy.com/ 198

Pradhan Mantri Garib Kalyan Deposit Scheme (PMGKDS), 2016 **


Who can declare any person can declare undisclosed income under the Scheme
between 17th December, 2016 to 31st March, 2017

Taxes and Penalty Tax @30% of the undisclosed income, surcharge @33% of tax and
penalty @10% of such income will be levied (i.e. effective tax total
50%)

Mandatory deposit Besides tax and penalty, 25% of the undisclosed income will be
deposited in Pradhan Mantri Garib Kalyan Deposit Scheme, 2016

Interest on The deposits will be interest free and have a lock-in period of four
deposit years

Proposed This amount is proposed to be utilised for the schemes of irrigation,


utilization of housing, toilets, infrastructure, primary education, primary health,
deposit livelihood, etc., so that there is justice and equality.

Interest subsidy scheme for housing ***


For urban areas Loans of upto Rs. 9 lakh taken in 2017, will receive interest
subvention of 4%.

Loans of upto Rs. 12 lakh taken in 2017, will receive interest


subvention of 3%.

For Rural areas Loans of upto Rs. 2 lakh taken in 2017, for new housing, or extension
of housing in rural areas, will receive an interest subvention of 3%
Whatsapp No 7023213423 http://iasselfstudy.com/ 199

Chapter 11

Miscellaneous Topics related to Economy

Corporate governance
Corporate Governance may be defined as a set of systems, processes and principles which
ensure that a company is governed in the best interest of all stakeholders.

Demographic dividend *
refers to a period – usually 20 to 30 years – when a greater proportion of people are young
and in the working age-group.

This cuts spending on dependents, spurring economic growth.

Because people of this age group are productive. India is counted as demographic dividend
country.

IAS Prelims 2013


To obtain full benefits of demographic dividend, what should India do?
a. Promoting skill development
b. Introducing more social security schemes
c. Reducing infant mortality rate
d. Privatization of higher education

IAS Prelims 2011


India is regarded as a country with “Demographic Dividend’’. This is due to ?
(a) Its high population in the age group below 15 years.
(b) Its high population in the age group of 15-64 years.
(c) Its high population in the age group above 65 years.
(d) Its high total population.

CDS-2011
The term 'demographic gap' signifies the difference
(a) in sex ratio
(b) in age
(c) in child/woman ratio
(d) between the birth and the death rate
Whatsapp No 7023213423 http://iasselfstudy.com/ 200

CDS-2014
'Population dividend' refers to
(a) total number of population
(b) youthful age structure of a population
(c) relatively high proportion of experienced aged people
(d) migration from richer region to poorer region

Capital gain *
When amount realised on sale of capital asset exceeds the purchase price.

Five year plans in India **


Five year plans were implemented in the Soviet union in 1920.

Plans Year Remarks/Objectives Target Actual


Growth Growth
rate

Ist 1951- Based on Harrod-Domar Model, Focus on 2.1% 3.6%


1956 Agriculture, price stability, power and transport

2nd 1956- Based on Mahalanobis model, Focus on public 4.5% 4.3%


1961 sector & heavy & basic industries, Industrial policy
1956

3rd 1961- to make India a 'self-reliant' and 'self-generating' 5.6% 2.8%


1966 economy, agriculture was given top priority to
support the exports and industry , Plan failed
because India-china war in 1962, Drought in 1965,
war with Pakistan in 1965-66

Three 1966- Due to miserable failure of the Third Plan the


Annual 67, government was forced to declare plan holidays,
Plans 1967- Green Revolution
68,
1968-69

4th 1969- Indira Gandhi-PM, Nationalization of banks, India- 5.7% 3.3%


1974 Pak war in 1971, Nuclear test in 1974, Gadgil
formula

5th 1974- Prepared by DP Dhar, High inflation, Removal of 4.4% 4.8%


1979 poverty (Garibi Hatao), Self Reliance, Emergency in
Whatsapp No 7023213423 http://iasselfstudy.com/ 201

1975. In 1978 Janata Party Govt-Morarji Desai-PM


rejected the 5th plan

Rolling 1978- Focus on employment. This plan was again rejected


plan 1980 by the Congress Government in 1980

6th 1980- focus on Increase in national income, 5.2% 5.7%


1985 modernization of technology, ensuring continuous
decrease in poverty and unemployment, Population
control

7th 1985- aimed at accelerating food grain production, 5% 6%


1990 increasing employment opportunities & raising
productivity with focus on ‘food, work &
productivity’. .

Annual 1990- Crisis of foreign exchange, Beginning of


plans 1992 privatization and liberalization in India.

8th 1992- Worsening balance of payment position, rising 5.6% 6.8%


1997 debts, widening budget deficit, recession in
industry, India became member of WTO in 1995,
high growth of agriculture and allied sector, and
manufacturing sector, growth in exports and
imports

9th 1997- Started on completion of 50 years of independence, 6.5% 5.4%


2002 focus on “Growth With Social Justice & Equality,
Focus on 7 basic minimum services

10th 2002- Employment generation. 5 crores Jobs creation 8% 7.6%


2007 target, States role in planning was increased with
greater involvement of Panchayati Raj Institutions

11th 2007- Objective: Faster & More Inclusive Growth 9% 8%


2012

12th 2012- Objective: Faster, Sustainable, and More Inclusive 9%


2017 Growth

IAS Prelims 2014


The main objective of the 12th Five-Year Plan is
A. inclusive growth and poverty reduction
B. inclusive and sustainable growth
C. sustainable and inclusive growth to reduce unemployment
D. Faster, sustainable and more inclusive growth.
Whatsapp No 7023213423 http://iasselfstudy.com/ 202

CDS exam
Consider the following statements about the achievements of growth rate in India's Five Year Plans:
1. GDP grew less than the target during the Eleventh Plan
2. GDP grew more than the target during the Tenth Plan
3. GDP grew less than the target during the Ninth Plan
Which of the statements given above is/are correct?
(a) l only
(b) I and 2
(c) 1 and 3
(d) 2 and 3

CDS-2011
Match List I with List II and select the correct answer using the code given below the Lists:
List I List II
(Five Year Plan) (Basic Strategy)
A. Eighth Five Year Plan 1. Export-led growth
B. Ninth Five Year Plan 2. Agricultural development led growth
C. Tenth Five Year Plan 3. Equity and Social Justice
D. Eleventh Five Year Plan 4. Faster and Inclusive growth
Code:
A B C D
(a) 4 3 I 2
(b) 4 1 3 2
(c) 2 1 3 4
(d) 2 3 I 4

CDS-2012
The 'Nehru-Mahalanobis strategy of development guided the planning practice in India from the
(a) First Five-Year Plan to the Sixth Five-year Plan'
(b) Fourth Five-Year Plan to the Eighth Five-Year' Plan
(c) Second Five-year Plan to the Seventh Five-Year Plan
(d)First Five-Year Plan to the Eighth Five-Year Plan

CDS-2015
In recent plans, certain words/phrases were used in the title of the plan along with 'growth'. They are
1. Inclusive
2. Faster
3. More inclusive
4. Sustainable
5. More sustainable
Which combination is true of the Twelfth Five Year Plan (2012 -17) ?
(a) 1,2 and 3 (b) 1,4 and 5 (c) 2,3 and 4 (d) 1,2 and 4

CDS-2015
Inclusion strategy does not focus on
(a) reduction of inequality
(b) reduction of poverty
(c) diversifying livelihood for tribal population
(d) getting poorer countries closer

CDS-2016
Which one of the Five Year Plans had a high priority to bring inflation under control and to achieve
stability in the economic situation ?
(a) Fourth Plan (1969-74) (b) Fifth Plan (1974-79)
(c) Sixth Plan (1980-85 ) (d) Seventh Plan (1985- 90) .
Whatsapp No 7023213423 http://iasselfstudy.com/ 203

CISF-LDC-2013
Which one of the following is the main objective of 12th Five Year Plan ?
(a) Sustainable development only
(b) Inclusive growth only
(c) Faster development with globalization
(d) Faster, sustainable and more inclusive growth

NDA-2015
The Second Five Year Plan that called for the establishment of Socialist pattern of society was
commonly referred to as the
(a) Harrod-Domar Plan
(b) Mahalanobis Plan
(c) Nehru Plan
(d) Peoples Plan

NDA-2015
The First Five Year Plan (1951-1956) was drafted by:
(a) P C Mahalanobis
(b) K N Raj
(c) J C Kumarappa
(d) Jawaharlal Nehru

NDA-2016
Which. of the following best 'explains 'the stated broad vision and aspirations of the Twelfth Five Year
Plan (2012 - 2017) ?
(a) Faster, Sustainable' and More Inclusive Growth
(b) Modernization of Industries and Strengthening Infrastructure
(c) Enhancing : Agricultural and Rural Incomes
(d) Checking Inflation and Strengthening non-economic variables like Nutritional Requirements, Health
'and Family Planning

NDA-2016
Which one of the following is not a feature of the Nehru-Mahalanobis model of development
strategy ?
(a) Development of capital goods industries
(b) Major involvement of the State in the economy
(c) Industrial deregulation and disinvestment in the public sector
(d) Enhancing the scope and importance of the public sector

SCRA-2014
The Government's priority in the first Five Year Plan of lndia was
(a) Industry
(b) Agriculture
(c) Technology
(d) Education

IAS Prelims 2010


In the context of India's Five Year Plans, a shift in the pattern of industrialization, with lower
emphasis on heavy industries and more on infrastructure begins in
(a.) Fourth Plan
(b.) Sixth Plan
(c.) Eighth Plan
(d.) Tenth Plan
Whatsapp No 7023213423 http://iasselfstudy.com/ 204

IAS Prelims 2009


During which Five Year Plan was the Emergency clamped, new elections took place and the Janata Party
was elected?
A. Third
B. Fourth
C. Fifth
D. Sixth

IAS Prelims 2009


Consider the following statements regarding Indian Planning:
1. The Second Five Year Plan emphasized on the establishment of heavy industries.
2. The Third Five Year Plan introduced the concept of import substitution as a strategy for
industrialization.
Which of the statements given above is/ are correct?
A. 1 only
B. 2 only
C. Both 1 and 2
D. Neither 1 nor 2

IAS Prelims 1992


The average growth achieved during the Seventh Five Year Plan in India (in terms of GDP at factor
cost) was
(a) 3.5
(b) 4.2
(c) 5.1
(d) 5.6

IAS Prelims 1995


Which one of the following Five Year Plans recognised human development as the core of all
developmental efforts ?
(a) The Third FiveYear Plan
(b) The Fifth FiveYear Plan
(c) The Sixth FiveYear Plan
(d) The Eighth Five Year Plan

IAS Prelims 1997


The Sixth and the Eighth Five Year Plans covered the period 1980 – 85 and 1992 – 97 respectively.
The Seventh Five Year Plan covered the period
(a) 1987 – 92
(b) 1986 – 91
(c) 1985 – 90
(d) 1988 – 94

CDS-2014
'Inclusive growth' is a phrase used in India's
1. 9th Plan
2. 10th Plan
3. 11th Plan
4. 12th Plan
Select the correct answer using the code given below.
(a) 1, 2 and 3
(b) 2 and 4
(c) 3 and 4
(d) 4 only
Whatsapp No 7023213423 http://iasselfstudy.com/ 205

CDS 2017
Consider the following statements about the Second Five -Year Plan :
1. It was drafted under the leadership of K. N. Raj.
2. It proposed that industries like electricity, railways, steel, machineries and communication could be
developed in the public sector .
3. The drafters found balancing industry and agriculture very difficult.
4. The drafters found balancing industry and agriculture really easy.
Which of the statements given above is / are correct?
(a) 1 only
(b) 1 and 2
(c) 2 and 3
(d) 3 and 4

CDS 2017
The Twelfth Five-Year Plan focused on inclusive growth. Which of the following were considered as
challenges for inclusiveness?
1. Poverty
2. Group inequality
3. Regional imbalance
4. Unemployment
Select the correct answer using the code given below.
(a) I, 3 and 4 only
(b) I, 2, 3 and 4
(c) I, 2 and 4 only
(d) 2 and 3 only

Major sector of economy


Primary Which makes direct use of natural resources. This includes
agriculture, forestry, fishing and mining

Secondary Manufacturing & construction

Tertiary Services

CDS-2013
Which of the following occupations are included under secondary sector as per the national income
accounts?
1. Manufacturing
2. Construction
3. Gas and water supply
4. Mining and quarrying
Select the correct answer using the code given below.
A 1,2,3 and 4
B. 1,2 and 4 only
C. 1,2 and 3 only
D. 3 and 4 only
Whatsapp No 7023213423 http://iasselfstudy.com/ 206

CDS-2013
Which one among the following is a tertiary activity ?
(a) Farming
(b) Weaving
(c) Manufacturing
(d) Trading

CAPF-2011
Which one of the following types of economic activities dominates in all rural settlements?
(a) Primary
(b) Secondary
(c) Tertiary
(d) Quaternary

Inflation ***
Meaning It is a sustained increase in the general price level of goods and
services.

purchasing power It reflects a reduction in the purchasing power per unit of money.

Benefit/Loss Inflation provide benefits to debtors and loss to creditors/bond-


holders.

Limited inflation is good for growth in developing country.

measured by Inflation is measured by CPI & WPI

Retail inflation measured in term of CPI

Wholesale inflation measured in term of WPI

CDS-2016
Which of the following will be the outcome if an economy is under the inflationary pressure?
1. Domestic currency heads for depreciation.
2. Exports become less competitive with imports getting costlier.
3. Cost of borrowing decreases.
4 . Bondholders get benefitted.
Select the correct answer using the code given below.
(a) 1 and 2
(b) 2 and 3
(c) 1 and 3 only
(d) I, 3 and 4

CAPF-2009
Consider the following statements :
1 When the inflation decreases, but still remains positive, it is deflation.
2 Deflation reduces the real value of money over time.
3 Historically not all episodes of deflation correspond with periods of poor economic growth.
Whatsapp No 7023213423 http://iasselfstudy.com/ 207

Which of the statements given above is/are correct ?


(a) 1, 2 and 3
(b) 1 and 3 only
(c) 3 only
(d) 2 only

CAPF-2013
The rate of inflation In India is measured generally in respect of movement of
(a) consumer price index
(b) wholesale price index
(c) cost of living index for agricultural labour
(d) money supply

IAS Prelims 2015


Which reference to inflation in India, which of the following statements is correct?
a) Controlling the inflation in India is the responsibility of the Government of India only
b) The Reserve Bank of India has no role in controlling the inflation
c) Decreased money circulation helps in controlling the inflation
d) Increased money circulation helps in controlling the inflation

IAS Prelims 2013


Consider the following statements:
1.Inflation benefits the debtors.
2.Inflation benefits the bond-holders.
Which of the statements given above is/are correct?
a. 1 only
b. 2 only
c. Both 1 and 2
d. Neither 1 nor 2

IAS Prelims 2013


A rise in general level of prices may be caused by
2. an increase in the money supply
3. a decrease in the aggregate level of output
4. an increase in the effective demand
Select the correct answer using the codes given below.
a. 1 only
b. 1 and 2 only
c. 2 and 3 only
d. 1, 2 and 3

Parallel economy
Economy controlled by the black money is called is called Parallel economy.

Parallel economy, based on the black money or unaccounted money, is a big menace to the
Indian economy.

It is also a cause of big loss in the tax-revenues for the government


Whatsapp No 7023213423 http://iasselfstudy.com/ 208

Deflation *
It is a persistent fall in the general price level of goods and services.

Deflation occurs when the inflation rate falls below 0% (a negative inflation rate).

SCRA-2010
Deflation means
(a) negative inflation rate
(b) decline in inflation rate
(c) depreciation of local currency
(d) decline in money supply

IAS Prelims 2010 & CDS-2012


Which of the following statements is an appropriate description of deflation?
(a)It is a sudden fall in the value of a currency against other currencies
(b.) It is a persistent recession in both the financial and real sectors of economy
©It is a persistent fall in the general price level of goods and services
(d)It is a fall in the rate of inflation over a period of time

IAS Prelims 1992


A deflator is a technique of
(a) Adjusting for changes in price level
(b) Adjusting for change in commodity
(c) Accounting for decline of GNP
(d) Accounting for higher increase of GNP

Stagflation *
a situation where the inflation rate is high, the economic growth rate slows down, and
unemployment remains steadily high.

SCRA-2010
Inflation accompanied by increase in unemployment is called
(a) recession
(b) depression
(c) deflation
(d) stagflation

A subsistence economy
is a non-monetary economy which relies on natural resources to provide for basic needs,
through hunting, gathering, and subsistence agriculture.
Whatsapp No 7023213423 http://iasselfstudy.com/ 209

Green accounting or environment accounting


Green accounting is a type of accounting that attempts to factor environmental costs into the
financial results of operations

Green GDP= GDP- waste/degradation of natural resources

Minimum support price *


What is MSP MSP is a form of market intervention by the Government to insure
agricultural producers against any sharp fall in farm prices

Recommendation By Commission for Agriculture costs and prices (CACP)


of MSP

Approval of MSP By Cabinet Committee on Economic Affairs

About CACP The CACP being the expert body, its recommendations are generally
accepted as such.

IAS Prelims 1995


Which of the following are the objectives of the Commission for Agricultural Costs and Prices (CACP) ?
I. To stabilise agricultural prices.
II. To ensure meaningful real income levels to the farmers.
III. To protect the interest of the consumers by providing essential agricultural commodities at
reasonable rates through public distribution system.
IV. To ensure maximum price for the farmer.
Select the correct answer from the codes given below :
Codes:
(a) I, II and III
(b) I, II and IV
(c) I, III and IV
(d) II, III and IV

IAS Prelims 2001


The prices at which the Government purchases food grains for maintaining the public distribution
system and for building up buffer stocks is known as
(a) minimum support prices
(b) procurement prices
(c) issue prices
(d) ceiling prices

IAS Prelims 2015


The Fair and Remunerative Price of Sugarcane is approved by the
a) Cabinet Committee on Economic Affairs
b) Commission for Agricultural Costs and Prices
c) Directorate of Marketing and Inspection, Ministry of Agriculture
d) Agricultural Produce Marketing Committee
Whatsapp No 7023213423 http://iasselfstudy.com/ 210

Sacrifice ratio *
If inflation is becoming a problem, central banks try to cool economic growth in a bid to reduce
inflationary pressures.

Sacrifice ratio measures the proportionate reduction in growth rate due to reduction in inflation.

It indicate that if inflation comes down then growth rate will also fall.

Round – Tripping
Black money generated in India coming back to India as foreign investment. It increases
inflation

Laffer curve *
It says that Higher tax rate discourages the production. Consequently, revenue decreases.

Devaluation *
means reduction in value of money

Benefits -Export increases

IAS Prelims 1994


Devaluation of a currency means
(a) reduction in the value of a currency vis-a-vis major internationally traded currencies
(b) permitting the currency to seek its worth in the international market
(c) fixing the value of the currency in conjunction with the movement in the value of a basket of
predetermined currencies
(d) fixing the value of a currency in multilateral consultation with the IMF, the World Bank and major
trading partners

IAS Prelims 1999


Assertion (A): Devaluation of a currency may promote export.
Reason (R): Price of the country’s products in the international market may fall due to devaluation.
(a) Both A and R are true, and R is the correct explanation of A
(b) Both A and R are true, but R is not a correct explanation of A
(c) A is true, but R is false
(d) A is false, but R is true
Whatsapp No 7023213423 http://iasselfstudy.com/ 211

Economic growth *
Means continuous increase in total volume of production over a long period.

National Income Index measures the rate of economic growth

Economic development *
is nothing but economic growth plus qualitative changes in the economy.

Qualitative changes refer to improvement in living standards, reduction in inequality of income


and no of poor people, increase in per capita income, development of technology etc.

Per capita income index measures/indicates the rate of economic development

Price index is a measure of cost of living.

IAS Prelims 2001


The most appropriate measure of a country’s economic growth is its
(a) Gross Domestic Product
(b) Net Domestic Product
(c) Net National Product
(d) Per Capita Real Income

Consumer Price Index for industrial workers (Base year 2001=100) **


Issued Monthly

Issued by Labour bureau

When issued on the last working day of the succeeding month

Uses Computation of Dearness allowance of Govt employees

IAS Prelims 2015


Which of the following brings out the ‘Consumer Price Index Number for the Industrial Workers’?
a) The Reserve Bank of India
b) The Department of Economic Affairs
c) The Labour Bureau
d) The department of Personnel and Training
Whatsapp No 7023213423 http://iasselfstudy.com/ 212

Consumer Price Index Numbers for Agricultural Labourers (Base year 1986-
87= 100)
Issued Monthly

Issued by Labour bureau

Consumer Price Index Numbers for Rural Labourers Base year 1986-87= 100
Issued Monthly

Issued by Labour bureau

Consumer Price Index -combined (Base year 2012= 100) ***


Issued Monthly

Issued by Central Statistics office (CSO), Ministry of Statistics and Programme


Implementation

Combined means Rural & Urban

Uses Key measure of Inflation by RBI

Wholesale Price Index (Base year 2004-05=100) **


Issued Monthly on 14th of every month

Issued by Office of economic adviser, DIPP, Ministry of commerce & Industry

Commodities & Weight 1) PRIMARY ARTICLES (Weight 20.12%)

(in Primary articles, Food articles weight is highest)

2) FUEL & POWER (Weight 14.91%)

3) MANUFACTURED PRODUCTS (Weight 64.97%)


Whatsapp No 7023213423 http://iasselfstudy.com/ 213

CAPF-2010
Consider the following items of wholesale price index:
1. Primary articles
2. Fuel, power, lights and lubricants
3. Manufacturing products
Arrange the above items in descending order in terms of their weightage in calculating wholesale price
index:
(a) 1, 2, 3
(b) 1, 3, 2
(c) 3, 1, 2
(d) 3, 2, 1

CAPF-2013
Which one among the following items has maximum weight in wholesale price index in India?
a. Primary article
b. Fuel and power
c. Manufactured product
d. Food item

CISF-LDC-2009
In calculation of wholesale price index, which item has been assigned the maximum weightage ?
(a) Food articles
(b) Non-food articles
(c) Fuel, power, light and lubricants
(d) Manufactured products

IAS Prelims 2010


With reference to India, consider the following statements:
1. The Wholesale Price Index (WPI) in India is available on a monthly basis only .
2. As compared to Consumer Price Index- for Industrial Workers CPI(lW)), the WPI gives less weight to
food articles.
Which of the statements given above is/are correct?
(a.) 1 only
(b.) 2 only
(c.) Both 1 and 2
(d.) Neither, 1 nor 2

CDS-2011
At present the Wholesale Price Index (WPI) for all commodities including manufactured products is
released on a
(a) weekly basis
(b) monthly basis
(c) bi-weekly basis
(d) quarterly basis
Whatsapp No 7023213423 http://iasselfstudy.com/ 214

Index of Industrial Production (IIP) (Base 2004-05 =100) *


Issued Monthly

Issued by The Central Statistics Office (CSO) (Ministry of Statistics and


Program Implementation)

SCRA-2013
Consumer price index is designed to measure the degree to which
(a) Incomes are distributed among the poor and rich over time
(b) The cost of purchasing a bundle of consumer goods has changed with time
(c) Consumption patterns have changed with time because of higher prices
(d) Consumer prices have risen relative to wages

Index of Eight Core Industries (Base: 2004-05=100) Monthly issued *


Issued Monthly

Issued by Office of economic adviser, DIPP, Ministry of commerce & Industry

Industries & Weight Coal (weight: 4.38 %)

Crude Oil (weight: 5.22 %)

Natural Gas (weight: 1.71 %)

Refinery Products (weight: 5.94%)

Fertilizers (weight: 1.25%)

Steel (Alloy + Non-Alloy) (weight: 6.68%)

Cement (weight: 2.41%)

Electricity (weight: 10.32%)

IIP relation The Eight Core Industries comprise nearly 38 % of the weight of
items included in the Index of Industrial Production (IIP).

IAS Prelims 2015


In the Index of Eight Core Industries, which one of the following is given the highest weight?
a) Coal Production
b) Electricity generation
c) Fertilizer Production
d) Steel Production
Whatsapp No 7023213423 http://iasselfstudy.com/ 215

Maharatna PSU *

1. Coal India Limited


2. Indian Oil Corporation Limited
3. NTPC Limited
4. Oil & Natural Gas Corporation Limited
5. Steel Authority of India Limited
6. Bharat Heavy Electricals Limited
7. GAIL (India) Limited

First five PSU became Maharatna in 2010. BHEL and GAIL joined in 2013

ES-2012
Which of the following is/are the Maharatna Central Public Sector Enterprises ?
I. National Thermal Power Corporation Ltd.
2. Hindustan Fertilizers Corporation Ltd.
3. Oil and Natural Gas Corporation Ltd.
4. Steel Authority of India Ltd.
Select the correct answer using the codes given below :
(a) I and 3 only
(b) 2 and 4 only
(c) I, 2 and 4
(d) I, 3 and 4

7th Pay commission *


Chairman- Justice Shri Ashok Kumar Mathur

Its recommendations are likely to be implemented with effect from January 1, 2016.

Minimum pay 18000 pm

Maximum pay 250000 pm

One Person Company


The concept of One Person Company [OPC] is introduced by The Companies Act, 2013.

Now Entrepreneur(s) carrying on Sole-Proprietor business can enter into a Corporate


Framework and enjoy corporate benefits.
Whatsapp No 7023213423 http://iasselfstudy.com/ 216

Micro, Small & Medium Enterprises (MSME)-Definition under MSMED act, 2006
In case enterprise engaged in the manufacturing or production of goods:

Micro enterprise where investment in plant and machinery does not exceed Rs. 50 lakh

Small enterprise where the investment in plant and machinery is more than Rs. 50 lakh
but does not exceed Rs. 10crore; and

Medium enterprise where the investment in plant and machinery is more than Rs.10 crore
but does not exceed Rs.30 crore

In case enterprises engaged in providing or rendering of services:

Micro enterprise where the investment in equipment does not exceed Rs. 20 lakh;

Small enterprise where the investment in equipment is more than Rs.20 lakh but does
not exceed Rs. 5 crore; and

Medium enterprise where the investment in equipment is more than Rs. 5 crore but does
not exceed Rs. 15 crore.

ES-2012
In the context of Micro, Small and Medium Enterprises (MSME) sector in India, which among the
following is/are the correct criteria for declaring an enterprise "Micro" ?
1 The investment in plant and machinery does not exceed Rs. 50 lakhs.
2. The enterprise should fall in the category of manufacturing sector and not in service sector.
Select the correct answer using the codes given below:
(a) 1 only
(b) 2 only
(c) Both 1 and 2
(d) Neither I nor 2

IAS Prelims 1992


The definition of a small scale industry is based on
(a) The market outreach of a unit
(b) The size of the sales of a unit
(c) The fact whether the products manufactured fall in a list drawn up by the Ministry of Industry
(d) The value of the plant and machinery investment in a unit
Whatsapp No 7023213423 http://iasselfstudy.com/ 217

Consumer Disputes Redressal Agencies under Consumer Protection Act, 1986


*
District Forum Established by State Government in each district

Jurisdiction Where the value of the goods or services and the


compensation, if any, claimed ''does not exceed rupees
20 lakhs.

State Established by State Government


Commission
Jurisdiction Where the value of the goods or services and
compensation, if any, claimed exceeds rupees 20 lakhs
but does not exceed rupees 1 crore; or

Appeals against the orders of any District Forum within


the State;

National Established by Central Government


Commission
Jurisdiction Where the value of the goods or services and
compensation, if any, claimed exceeds rupees 1 crore; or

Appeals against the orders of any State Commission

IAS Prelims 2010


With reference to the Consumer Disputes Redressal at district level in India, which one of the
following statements is not correct?
(a) A State Government can establish more than one District Forum in a district if it deems fit
(b) One of the members of the District Forum shall be a woman
(c) The District Forum entertains the complaints where the value of goods or services does not exceed
rupees fifty lakhs
(d)A complaint in relation to any goods sold or any service provided may be filed with a District Forum
by the State Government as a representative of the interests of the consumers in general

4-coloured revolution representing the Indian flag *


Saffron Saffron is a colour of energy-energy revolution
revolution

White revolution increasing milk production

Green revolution improving agricultural productivity

Blue revolution blue water and skies (Development of Fisheries)


Whatsapp No 7023213423 http://iasselfstudy.com/ 218

Great depression *
Start of the Great Depression is attributed to the sudden devastating collapse
of US stock market on October 29, 1929, known as Black Tuesday.

Type of Unemployment *
Seasonal when people are unemployed at particular times of the year. E.g. off
unemployment season in winter on Hill station

Structural unemployment caused by a mismatch between the skills that workers in


unemployment the economy can offer, and the skills demanded by employers (also
known as the skills gap)

Disguised When more people are engaged in some activity than the required.
unemployment
For example : An agricultural land require 2 labourers but people
engaged in this activity is 5 then this unemployment for 3 labours is
called disguised unemployment.

Their marginal productivity, i.e., the addition to the production will be


zero

Cyclical When there is lack of aggregate demand for goods and services.
unemployment
This type of unemployment increases during a recession and decreases
during an expansion

Frictional Unemployment which occurs when workers move from one job to
unemployment another job. Also called search unemployment.

CDS-2012
When the productive capacity of the economic system of a State is inadequate to create sufficient number
of job's, it is called
(a) seasonal unemployment
(b) structural unemployment
(c) disguised unemployment
(d) cyclical unemployment

IAS Prelims 1992


Among the reasons for disguised unemployment in rural areas is
(a) Choice of a heavy industry model for economic development
(b) Low levels of technological development in the country
(c) Heavy pressure of population along with halfhearted implementation of agrarian reforms
(d) High illiteracy rates
Whatsapp No 7023213423 http://iasselfstudy.com/ 219

IAS Prelims 2013


Disguised unemployment generally means
a. large number of people remain unemployed
b. alternative employment is not available
c. marginal productivity of labour is zero
d. productivity of workers is low

IAS Prelims 1992


Unemployment which occurs when workers move from one job to another job is known as
(a) Seasonal unemployment
(b) Frictional unemployment
(c) Technological unemployment
(d) Cyclical unemployment

Renewable Energy Target **


The Government of India has set a target of 175 GW renewable power installed capacity by
the end of 2022.

This includes
60 GW from wind power,
100 GW from solar power,
10 GW from biomass power and
5 GW from small hydro power.

India’s First East Coast Economic Corridor (ECEC) **


Length 2500 KM from Kolkata (in West Bengal) to Tuticorin (in Tamil Nadu)

Benefit It will spur development on India’s eastern coast and create seamless
trade links with other parts of South and Southeast Asia

Funding Loan from Asian Development Bank

Visakhapatnam- It is a First Phase of ECEC


Chennai
The Visakhapatnam-Chennai Industrial Corridor section of the East
Industrial
Coast Economic Corridor will mark the first industrial corridor developed
Corridor
along India’s coast.
Whatsapp No 7023213423 http://iasselfstudy.com/ 220

Industrial Corridors *
Purpose To accelerate the growth in manufacturing and for ensuring
scientifically planned urbanization, Govt of India has adopted the
strategy of developing integrated Industrial Corridors in partnership
with State Governments with focus on manufacturing.

5 Corridors to be Delhi Mumbai Industrial Corridor (DMIC), (1st Corridor and Financed
developed by by Japan)
Govt.
Chennai-Bengaluru Industrial Corridor (CBIC),

Amritsar- Kolkata Industrial Corridor (AKIG),

Bengaluru- Mumbai Economic Corridor (BMEC) and

Vizag-Chennai Industrial Corridor (VCIC)

National NICDIT would be an apex body under the administrative control of


Industrial Department of Industrial Policy & Promotion (DIPP) for coordinated
Corridor and unified development of all the industrial corridors in the country
Development &
Implementation
Trust

CAPF-2016
Which one of the following is not an industrial corridor as per the policy initiaives ?
(a)Amritsar -Kolkata
(b)Delhi - Mumbai
©Kolkata - Guwahati
(d)Chennai – Bengaluru

NDA-2013
Delhi-Mumbai industrial corridor connects the political and business capital of India. Arrange the states
from highest to lowest in terms of the length of corridor passing through them.
(b) Maharashtra- Gujarat-Rajasthan-NCR of Delhi
(c) NCR of Delhi-UP- Rajasthan- Maharashtra
(d) Rajasthan- Gujarat –Maharashtra- NCR of Delhi
(e) Maharashtra- Rajasthan –UP-NCR of Delhi

UREA
UREA is a main source of Nitrogen.

Correct proportion in which Nitrogen, Phosphorus, and Potassium is used are 4:2:1.

Urea constitute 50% of total fertilizers’ consumption


Whatsapp No 7023213423 http://iasselfstudy.com/ 221

Regional Connectivity Scheme “UDAN” (Ude Desh Ka Aam Nagarik)***


Launched in October, 2016.

About scheme Scheme envisages providing connectivity to Un-served and under-


served airports of the country through revival of existing air-strips
and airports

Objective to facilitate/stimulate regional air connectivity by making it affordable


to the common man even in small towns

Market-based It is a market-based mechanism in which airlines bid for seat


subsidies. The routes and networks will be awarded to the bidders
who quote the lowest requirement of VGF per seat

Period of scheme scheme would be in operation for a period of 10 years

Proposed fare The fare for a one hour journey of appx. 500 km on a fixed wing
aircraft or for a 30 minute journey on a helicopter would be capped at
Rs. 2,500, with proportionate pricing for routes of different lengths /
flight duration

Support to Airlines 1.Concessions by Central Govt, State Govt and airport operators and
2.VGF support

Regional A Regional Connectivity Fund would be created to provide the viability


Connectivity Fund gap funding requirements under the scheme

Implementing Airport Authority of India


agency

India-Bangladesh 1320 MW Maitree Super Thermal Power Project *


About It is a joint venture between India and Bangladesh and to be executed by
project the Bangladesh-India Friendship Power Company Private Limited (BIFPCL)

Location of It is located just 65 kilometers from the Sundarbans (World Heritage Site)
project

Why in news Citing concerns over the impact on the World Heritage Site in Sundarbans,
including on the Royal Bengal Tiger, The UN has recommended
Bangladesh to scrap the proposed thermal power plant and consider another
location
Whatsapp No 7023213423 http://iasselfstudy.com/ 222

Asian Development Bank loan for building India’s longest river bridge
The Asian Development Bank (ADB) and the Government of India signed a $500 million loan
to build a 9.8 km long road bridge across the Ganges River to improve transport connectivity
between North and South Bihar and for better link between the State Capital, Patna, and the
surrounding areas.

Setting up of Financial Redressal Agency (FRA) *


Purpose to address the grievances of retail consumers against all
Financial Service Providers (FSPs).

Recommen Financial Sector Legislative Reforms Commission (FSLRC),


ded by

Resolution It will try to resolve all complaints through mediation. It will discourage court-
of like processes
complaints

Appeal Consumers will be able to appeal against the orders of FRA at Securities Appella
te Tribunal (SAT)

India becomes 5th largest economy-Forbes


India overtakes UK and becomes 5th largest GDP after USA, China, Japan & Germany.

Financial Intelligence Unit – India (Ministry of Finance) *


Purpose It was set by the Government in November 2004 as the central
national agency responsible for receiving, processing, analyzing and
disseminating information relating to suspect financial transactions.

Function to receive cash/suspicious transaction reports, analyse them and,

disseminate valuable financial information to intelligence/enforcement


agencies and regulatory authorities

Reporting FIU-IND is an independent body reporting directly to the Economic


Intelligence Council (EIC) headed by the Finance Minister.
Whatsapp No 7023213423 http://iasselfstudy.com/ 223

Enforcement Directorate (ED) *


About ED ED is a specialized financial investigation agency under the Department of
Revenue, Ministry of Finance

Area of work It enforces the following laws: -

Foreign Exchange Management Act,1999 (FEMA) and

Prevention of Money Laundering Act, 2002 (PMLA)

Financial Stability and Development Council (FSDC) *


Set up by Government as the apex level forum in December 2010

Purpose to strengthening and institutionalizing the mechanism for maintaining


financial stability, enhancing inter-regulatory coordination and promoting
financial sector development

Function Without prejudice to the autonomy of Regulators, the Council monitors


macro prudential supervision of the economy, including functioning of
large financial conglomerates, and addresses inter-regulatory coordination
and financial sector development issues.

It also focuses on financial literacy and financial inclusion.

Chairman of Finance Minister


the Council

Members Heads of financial sector Regulators (RBI, SEBI, PFRDA, IRDA & FMC)
Finance Secretary and/or Secretary, Department of Economic Affairs,
Secretary, Department of Financial Services, and Chief Economic Adviser.

The Council can invite experts to its meeting if required

IAS Prelims 2016


With reference to ‘Financial Stability and Development Council’, consider the following statements :
1. It is an organ of NITI Aayog.
2. It is headed by the Union Finance Minister.
3. It monitors macro prudential supervision of the economy.
Which of the statements given above is/are correct?
(a) 1 and 2 only
(b) 3 only
(c) 2 and 3 only
(d) 1, 2 and 3
Whatsapp No 7023213423 http://iasselfstudy.com/ 224

Varishtha Pension Bima Yojana - 2017 **


Social security It is a part of Government’s commitment for financial inclusion and
scheme social security.

Purpose to provide social security during old age and protect elderly persons
aged 60 years and above against a future fall in their interest income
due to uncertain market conditions.

Implemented Life Insurance Corporation of India (LIC)


through

Pension The scheme will provide an assured pension based on a guaranteed rate
of return of 8% per annum for 10 years, with an option to opt for
pension on a monthly / quarterly / half yearly and annual basis.

Closing date of VPBY-2017 is proposed to be open for subscription for a period of one
scheme year from the date of launch

Government issues notification of ‘Indian Skill Development Service’ **


About ISDS ISDS will be a Group ‘A’ service where induction will take place through
Indian Engineering Service Examination conducted by UPSC.

Purpose It is an attempt to attract young and talented administrators for Skill


Development.

The knowledge acquired by the engineers recruited will give new


impetus to the initiative of the government to the skill development and
also efficient and effective implementation of the schemes.

Created for This service has been created for the Training Directorate of the Ministry
of Skill Development and Entrepreneurship.
Whatsapp No 7023213423 http://iasselfstudy.com/ 225

Various Global Index **


Index Issued by

Global Innovation Index World intellectual property organization, Cornell university,


and INSEAD

Corruption perception Transparency international


index

Global Peace Index Institute for Economics and Peace

Global Slavery Index Walk free foundation

Global Terrorism Index Institute for Economics and Peace

Global Hunger Index International Food Policy Research Institute (IFPRI)

World Happiness Report United nations sustainable development solutions network

World most valuable Brand finance


Nation brands

Index of Economic The Heritage Foundation and The Wall Street Journal
Freedom

International Intellectual U.S. Chamber of Commerce


Property (IP) index

An economy for the 1% Oxfam international

Various Important Committees **


Committee on Revisiting & Revitalising the PPP chaired by Dr. V. Kelkar
model of Infrastructure Development

NITI Aayog constituted a Committee of Chief Shri Chandrababu Naidu, Hon’ble


Ministers to examine and implement measures to Chief Minister of Andhra Pradesh;-
implement digital payment systems convener

(recommended Banking Cash transaction tax)

Fiscal Responsibility and Budget Management Shri N.K. Singh


(FRBM) Review Committee to comprehensively
review and give recommendations on the FRBM
roadmap for the future

Committee on digital payments Ratan p watal


Whatsapp No 7023213423 http://iasselfstudy.com/ 226

National Institution for Transforming India (NITI Aayog) *


Established on 01-01-2015 through a cabinet resolution

Replaced NITI Aayog replaced the Planning Commission which was set up on
15-03-1950 through a Cabinet resolution

About NITI Aayog NITI Aayog is the premier policy ‘Think Tank’ of the Government of
India, providing both directional and policy inputs.

While designing strategic and long term policies and programmes for
the Government of India, NITI Aayog also provides relevant technical
advice to the Centre and States.

Chairperson Shri Narendra Modi, Hon'ble Prime Minister

Vice Chairperson Shri Arvind Panagariya

Two hubs At the core of NITI Aayog’s creation are two hubs – Team India
Hub and the Knowledge and Innovation Hub.

The Team India leads the engagement of states with the Central government
Hub

Knowledge and builds NITI’s think-tank capabilities


Innovation Hub

Future Vision and (a) A vision document keeping in view the social goals set and / or
Strategies proposed and Sustainable Development Goals (SDGs) for about 15
years for the period up to 2030 which is co-terminus with the period
to achieve SDGs.

(b) A 7-year strategy from 2017-18 to 2023-24 to convert the


longer vision into implementable policy and action as a part of
“National Development Agenda” with a mid-term review after 3 years
i.e. the year ending March 2020.

(c ) A 3-year Action Document for 2017-18 to 2019-20 aligned to the


predictability of financial resources during the 14th Finance
Commission Award period. This is also to help translate into actions
the goals of the government to be achieved by 2019.

CDS-2015
Who is the current Vice Chairperson of NITI Aayog?
(a) Raghuram Rajan (b) Arvind Panagariya (c) Bibek Debroy (d) V. K. Saraswat

IAS Prelims 2015


The Government of India has established NITI Aayog to replace the
a) Human Rights Commission
b) Finance Commission
c) Law Commission
d) Planning Commission
Whatsapp No 7023213423 http://iasselfstudy.com/ 227

Nikkei Manufacturing Purchasing Managers’ Index (PMI) *


What it measures Manufacturing PMI measures the performance of the manufacturing
sector and is derived from a survey of 500 manufacturing companies.

Weight The Manufacturing PMI is based on five individual indexes with the
following weights:

New Orders (30 percent),

Output (25 percent),

Employment (20 percent),

Suppliers’ Delivery Times (15 percent) and

Stock of Items Purchased (10 percent)

Results A reading above 50 indicates an expansion of the manufacturing


sector compared to the previous month;

below 50 represents a contraction;

while 50 indicates no change.

Eligibility of employees under EPF, ESI & Bonus act *


EPF Salary upto 15000 pm

ESI Salary upto Rs 21000 pm

Bonus Salary upto Rs 21000 pm, for the purpose of calculation of Bonus Rs. 7000pm

Individual beneficiaries having Ration Cards under National Food Security


Act, 2013 to furnish proof of possession of Aadhaar number or undergo
Aadhaar authentication to receive subsidies under NFSA **
Notification issued by Ministry of Consumer Affairs, Food and Public Distribution

Effective from 8th February, 2017 in all States and Union Territories except the
States of Assam, Meghalaya and Jammu and Kashmir

Aadhaar enrolment Beneficiaries under NFSA not yet enrolled for Aadhaar, require to
make application for Aadhaar enrolment by 30thJune, 2017
Whatsapp No 7023213423 http://iasselfstudy.com/ 228

Employment statistics
Economic Census Released by the Central Statistics Office (CSO) (Ministry of
Statistics and Program Implementation)

Annual Survey of conducted by Central Statistics Office (CSO)


Industries

Annual Employment- conducted by Labour Bureau (Ministry Of Labour & Employment)


Unemployment Survey

Classification of countries
Developed Countries Where share of Service sector is very high, industry sector is
medium and agriculture sector is minimum

Undeveloped Countries Where share of Service sector is minimum, industry sector is


medium and agriculture sector is very high

Developing Countries Where share of Service and industry sector is gradually increasing,
and agriculture sector is gradually decreasing.

Gini coefficient *
What it measures It is a measure of the deviation of the distribution of income among
individuals or households within a country from a perfectly equal
distribution.

Result A value of 0 represents absolute equality i.e. where everyone has the
equal income, a value of 100 represents absolute inequality.

IAS Prelims 2007


Consider the following statements:
1. The repo rate is the rate at which other banks borrow from the Reserve Bank of India.
2. A value of 1 for Gini Coefficient in a country implies that there is perfectly equal income for everyone
in its population.
Which of the statements given above is/are correct ?
(a) 1 only (b) 2 only
(c) Both 1 and 2 (d) Neither 1 nor 2
Whatsapp No 7023213423 http://iasselfstudy.com/ 229

Constitution of SIT on Panama Papers Leaks *


Constituted by Central Government

Purpose for facilitating coordinated and speedy investigation in the cases of


Indian persons allegedly having undisclosed foreign assets and whose
names are reportedly included in Panama Papers leaks.

Multi-Agency The Group consist of the officers of


Group
Central Board of Direct Taxes (CBDT),

Enforcement Directorate (ED),

Financial Intelligence Unit (FIU) and

Reserve Bank of India

Convener Member (Investigation), CBDT

Panama papers International Consortium of Investigative Journalists (ICIJ), a


leaked by Washington based organization

Phillips curve *
About it It shows the inverse relationship between unemployment and inflation

inverse It says that decrease in the rate of unemployment (i.e. increase in Jobs) will
relationship cause higher rate of inflation and vice versa.
means

Reason When unemployment is high salary increases slowly and when


unemployment is low salary increases rapidly

SCRA-2013
The trade-off between inflation and unemployment is known as
(a) cost-push inflation
(b) demand-pull inflation
(c) Phillips curve approach to inflation
(d) Stagflation
Whatsapp No 7023213423 http://iasselfstudy.com/ 230

Defence Acquisition Council (DAC)


the highest decision-making body in the Defence Ministry, is chaired by Defence Minister

CAPF-2015
Who heads the Defence Acquisition Council?
(a) Defence Minister
(b) Defence Secretary
(c) Chief of the Integrated Defence Staff
(d) Director General (Acquisition)

Employment News
Weekly publication Published by Publications Division under Ministry of Information
and Broadcasting.

Languages It publishes government job vacancies simultaneously in Hindi,


English & Urdu.

The Food Safety and Standards Authority of India (FSSAI)


Establishment established under Food Safety and Standards Act, 2006

Administrative Ministry of Health & Family Welfare, is the Administrative Ministry


Ministry for the implementation of FSSAI

The Unique Identification Authority of India (UIDAI) *


Statutory Established under the provisions of the Aadhaar (Targeted Delivery of
authority Financial and Other Subsidies, Benefits and Services) Act, 2016
(“Aadhaar Act 2016”)

Administrative Ministry of Electronics and Information Technology (MeitY)


Ministry
Prior to its establishment as a statutory authority, UIDAI was functioning
as an attached office of the NITI Aayog

Unique It is 12-digit Unique Identification numbers (UID), named as "Aadhaar",


Identification to be issued to all residents of India
numbers or
Aadhaar
Whatsapp No 7023213423 http://iasselfstudy.com/ 231

CDS-2016
Which of the following statements with regard to UID / Aadhar Card are correct?
1. It is a 12 digit unique form of identification for all residents of lndia
2. It is an identity number along with the biometric information of the individuals
3. It is a national identity and citizenship card
Select the correct answer using the code given below:
(a) 2 and 3 only
(b) I and 2 only
(c) I and 3 only
(d) 1, 2 and 3

Mumbai- Ahmedabad High Speed Railway (MAHSR) project *


Japan cooperation Memorandum of Cooperation (MoC) has been signed by the
Government of India and Government of Japan.

Estimated cost Rs. 1,00,000/ crore

Funding Japan has agreed to fund 81% of the total project cost through a 50
year loan at an interest rate of 0.1% .

Diamond Quadrilateral Network in Indian Railway


Six corridors on Diamond Quadrilateral connecting metropolitan cities and growth centers
of the country (Delhi, Mumbai, Chennai & Kolkata) have been identified for feasibility
studies for high speed rail connectivity viz:

(i) Delhi-Mumbai, (ii) Mumbai-Chennai, (iii) Chennai-Kolkata, (iv) Kolkata-Delhi and both
diagonals i.e. (v) Delhi-Chennai and (vi) Mumbai-Kolkata routes.

Minimum Wages *
Fixed by Under the provisions of the Minimum Wages Act, 1948, both Central and
State Governments are appropriate Governments to fix, review and revise
the minimum wages of the workers employed in the scheduled
employment consisting of agriculture and non-agricultural sector under
their respective jurisdictions

Half yearly Minimum wages also include Variable Dearness Allowance which is linked
revision to Consumer Price Index and revised half yearly in April and October
Whatsapp No 7023213423 http://iasselfstudy.com/ 232

SC bans liquor shops within 500 metres of highways from 1 April *


Purpose To reducing drink driving and road accidents

Coverage all liquor shops within 500 metres of national and state highways
across the country

Advertisement SC also ordered removal of all signboards of alcohol along highways.

Applicability From 1st April 2017

First Indian Institute of Skills at Kanpur **


Purpose In line with the vision of making India the Skill Capital of the World
by empowering its youth to be more employable and self-
sustainable

Partner Institute of Technical Education, Singapore.

The Institute has been conceptualized by the Prime Minister during


his visit to Singapore’s Institute of Technical Education.

Inspired by Singapore model of training

Target To have six such institutes all across

Prices of Petrol and Diesel *


Market based The prices of Petrol and Diesel have been made market determined
by the Government effective 26th June, 2010 and 19th October, 2014
respectively.

Who decides price Public Sector Oil Marketing Companies (OMCs)

Price of CNG/PNG fixed by the concerned City Gas Distribution (CGD) entities as per
market dynamics.
Whatsapp No 7023213423 http://iasselfstudy.com/ 233

National Pharmaceutical Pricing Authority *


About NPPA NPPA fix/ revises the prices of controlled bulk drugs and
formulations and to enforce prices and availability of the medicines
in the country, under the Drugs (Prices Control) Order, 1995.

Other area of work Recovering amounts overcharged by manufacturers for the


controlled drugs from the consumers.

Monitoring the prices of decontrolled drugs in order to keep them at


reasonable levels

Administrative Department of Pharmaceuticals, Ministry of Chemicals and Fertilizers


ministry

National Productivity Week


When celebrated 12th-18th February is celebrated as the ‘National Productivity Week’
every year

This year theme From Waste to Profits-through Reduce, Recycle and Reuse”.

About National It is an autonomous body under Department of Industrial Policy &


Productivity Council Promotion, Ministry of Commerce & Industry.

It was established on 12th February, 1958, with the objective of


stimulating and promoting productivity and quality consciousness
across all sectors in the country

Gujarat becomes the first State for establishing Cashless System for
distribution of food grains **
Government of Gujarat installed Aadhaar Enabled Payment System much before the targeted
date of 31st March, 2017

The beneficiaries under NFSA will need to carry only Aadhaar cards for getting their
foodgrains in Gujarat.

This will help in establishing the identity of beneficiaries, help in stopping the leakages of
grains at shop level and above all eradicate corruption in the Public Distribution System.
Whatsapp No 7023213423 http://iasselfstudy.com/ 234

Pradhan Mantri Gramin Digital Saksharta Abhiyan **


Purpose to make 6 crore rural households digitally literate by March, 2019

Target 25 lakh candidates will be trained in the FY 2016-17;

275 lakh in the FY 2017-18; and

300 lakh in the FY 2018-19

Coverage each of the 250,000 Gram Panchayats would be expected to register an


average of 200-300 candidates to ensure equitable geographical reach

Digitally literate persons would be able to operate computers/digital access devices (like
means tablets, smart phones, etc.), send and receive emails, browse internet,
access Government Services, search for information, undertaking
cashless transactions, etc. and hence use IT to actively participate in the
process of nation building.

Implementation Scheme would be carried out under the overall supervision of Ministry of
Electronics and IT in active collaboration with States/UTs through their
designated State Implementing Agencies, District e-Governance Society
(DeGS), etc.

Strategic Crude Oil Storage facilities *


Purpose To ensure energy security during any external supply disruptions

Phase-1 5 million metric tons (MMT) of strategic crude oil storages at three
locations namely, Visakhapatnam, Mangalore and Padur (near Udupi)

Phase-2 Budget Chandikhole in Jajpur district of Odisha and Bikaner in Rajasthan


2017
annoucement

Indian strategic The construction of the Strategic Crude Oil Storage facilities is being
petroleum managed by Indian Strategic Petroleum Reserves Limited (ISPRL), a
reserves limited Special Purpose Vehicle, which is a wholly owned subsidiary of Oil
Industry Development Board (OIDB) under the Ministry of Petroleum &
Natural Gas.

Engineers India Limited (EIL) is taken as the Project Management


Consultant for all three projects.
Whatsapp No 7023213423 http://iasselfstudy.com/ 235

Service charges on the bills of food and beverages **


The Department of Consumer Affairs has issued advisory to the State Governments to
sensitize the companies, hotels and restaurants regarding the provisions of the Consumer
Protection Act, 1986 on unfair trade practices and also to advise the hotels/restaurants to
disseminate information through display at the appropriate place in the hotels/restaurants
that the 'service charges" on the bills of food and beverages in lieu of tips which a consumer
is forced to pay irrespective of the kind of service provided to him are discretionary/
voluntary and a consumer dissatisfied with the services can have it waived off.

India Innovation Index **


Purpose To make India an innovation-driven economy

Initiative of NITI Aayog, Department of Industrial Policy & Promotion (DIPP) and
Confederation of Indian Industry (CII)

Ranking of States will be ranked on innovations from 2017 through country’s first
States online innovation index portal that will capture data on innovation from all
Indian states on innovation and regularly update it in real time

Amendment in section 6 of Payment of Wages Act **


Old provision All wages shall be paid in current coin or currency notes or in both:
in Section 6
Employer can pay the wages either by cheque or by crediting the wages in
bank account only after obtaining the written authorization of the employee

New provision All wages shall be paid in current coin or currency notes or by cheque or
in Section 6 by crediting the wages in the bank account of the employee:

Government may specify the industrial or other establishment, which shall


pay the wages only by cheque or by crediting the wages in bank account.

Effect After amendment employer can also pay wages to workers through cheque
or Bank credit
Whatsapp No 7023213423 http://iasselfstudy.com/ 236

Creation of Indian Enterprise Development Service (IEDS) **


Ministry in the Office of Development Commissioner (MSME), Ministry of Micro,
Small and Medium Enterprises(MSME)

Purpose It will help to achieve the vision of “Startup India”, “Stand-up India”
and “Make in India”

It will enhance the capacity and efficiency of the organization and also
help in achieving growth in MSME sector through a focussed and
dedicated cadre of technical officers.

BLUE ECONOMY *
About Blue Dr. Gunter Pauli introduced this concept in the book – "The Blue Economy:
Economy 10 years, 100 innovations, 100 million jobs.

The Blue Economy encompasses various traditional marine sectors such as


fisheries and aquaculture, shipping, port infrastructure, ship building and
repair, island development, seabed exploration, hydrocarbon extraction and
marine tourism industry.

Why in news The Niti Aayog has started a consultation process for integrating defence
and internal security with a 15-year vision and initiated a discussion to
leverage India's status as a major maritime nation with a long coastline and
the potential to become a significant 'blue economy'

Miscellaneous **
1. Global 2008 financial crisis was created by sub prime mortgage.
2. An increase in foreign income leads to increased exports and increases domestic output.
It also improves the trade balance.
3. Trade deficits need not be alarming if the country invests the borrowed funds yielding a
rate of growth higher than the interest rate.
4. India’s GDP growth is largely driven by the Service sector
5. when the economy has both trade deficit and budget deficit, it is said to be facing twin
deficits
6. India’s first Semi high speed Gatiman express train Delhi to Agra
Whatsapp No 7023213423 http://iasselfstudy.com/ 237

7. Foreign economic agents accept a national currency only if they are convinced that the
currency will maintain a stable purchasing power.
8. When the supply of money remain same, if demand of money increases then interest
rate will increase.
9. Capital formation increase economic growth in country.
10.Purpose of reduction in excise duty is to encourage growth in export.
11.Taxes on share market and commodities market are levied by Central govt.
12.Level of per capita income has been regarded as a summary indicator of the economic
well-being of the country.
13.Production, consumption and investments are the three basic economic activities in an
economy. These are interrelated and interdependent. These three economic activities
are responsible for generating the income flows in the economy.
14.Comprehensive employment data are made available once in five years by the national
sample survey organization (NSSO).
15.Securities Transaction Tax (STT) is levied on securities and commodity transaction tax
(CTT) is levied on non-agriculture commodities trade.
16.STT and CTT is allowed as deduction from such business income.
17. A rise in the interest rates at home often leads to an appreciation of the domestic
currency. Because foreign investment will come and demand for domestic currency will
rise.
18.When income increases, consumer spending increases. Spending on imported goods is
also likely to increase. Then there is a depreciation of the domestic currency.
19.Indian economy is set for a ’goldilocks’ period — used to describe a timeframe of high
growth and low inflation
20.Engineering Export Promotion Council of India- sponsored by Ministry of commerce &
industry
21.Interest Rate on Small Savings, Post Offices, NSC, PPF, Kisan Vikas Patra etc. decided
By Ministry of Finance and notified Quarterly
22.GIFT city in Gujarat is a India’s first International Financial Services Centre (IFSC)
23. India’s first international stock exchange- India international exchange (INX) set up in
GIFT city. India INX is a wholly owned subsidiary of BSE Ltd.
24.Sweden is becoming the world's first cashless economy
25.Interest rate on Employee provident fund is decided by the Finance ministry on the
recommendation of Central Board of Trustees of EPFO chaired by Labour minister.
Central Board recommended 8.65% interest to its subscribers for the year 2016-17
26.Tea and Spices are brand ambassador of India
27.A quarterly report on Public Debt Management is published by Public Debt Management
Cell (PDMC) in Budget Division of Department of Economic Affairs, Ministry of finance.
Whatsapp No 7023213423 http://iasselfstudy.com/ 238

28.The Centralized Processing Centre (CPC) of the Income Tax Department (ITD) at
Bengluru processes Income Tax Returns (ITRs)
29.Cabinet Committee on Economic Affairs is chaired by the Prime Minister
30.Advance Pricing Agreements pertains to taxation (Central Board of Direct Taxes)
31.The General Anti Avoidance Rule (GAAR) provisions under the Income Tax Act shall be
effective from the Assessment Year 2018-19 onwards, i.e. Financial Year 2017-18
onwards.
32.BFSI stand for Banking, Financial Service and Insurance
33.Cyrus Mistry was removed as chairman of TATA Sons and Natarajan Chandrasekaran
from TCS became the new chairman
34.Liberalization started in China 1978, Pakistan 1988 and in India 1991
35.Industry 4.0 or Fourth Industrial Revolution means Digital revolution.
36.Ministry of Electronics & Information Technology is mandated to assist in the promotion
of e-commerce.
37.Issues relating to Foreign Direct Investment (FDI) in the country are administered by
Department of Industrial Policy & Promotion.
38.One barrel of oil means 42 gallons (approximately 159 liters)
39.As a part of Digital India programme 2.5 lakh Gram Panchayats are to be connected by
National Optic Fibre Network
40.The international crude oil price of Indian Basket computed/published by Petroleum
Planning and Analysis Cell (PPAC) under the Ministry of Petroleum and Natural Gas
41. Setu Bharatam programme aims to make all National Highways free of railway level
crossings by 2019
42.LPG subsidy will not be available if the consumer or his/her spouse had taxable income
of more than Rs 10 lakh during the previous financial year
43.Greenfield projects- means setting up of new project
44.Brownfield projects- means renovation of existing project
45.DIN stand for Director Identification Number and CIN stand for Corporate Identification
Number
46.Basic exemption limit under Income tax act
For Super senior citizens (80 years or more) Rs. 5 lacs
For Senior citizens (60 years or more but less than 80 years) Rs. 3 lacs
Others Individuals Rs. 2.5 lacs
Whatsapp No 7023213423 http://iasselfstudy.com/ 239

Chapter 12

UPSC Previous Years Questions related to Economy


not covered in earlier Chapters

CDS-2010
Match List 1 with List II and select the correct answer using the code given below the Lists :
List I (Industrial policy) List II (Salient feature)

A. The industrial policy 1948 1. Began the process of state-centric economy


B. The industrial policy 1956 2. Reaffirmed faith in mixed economy
C. The industrial policy 1980 3. Initiated public-private partnership
D. The industrial policy 1991 4. Ushered in mixed economy

Code:
A B c D

(a) 4 1 2 3
(b) 4 2 1 3
(c) 3 1 2 4
(d) 3 2 1 4

CDS-2013
Whether India is experiencing inclusive growth in the true sense can be assessed by looking at
I. the economic growth indicators of the different States of India
2. whether different social groups like the SCs, STs and Minorities are getting excluded from the
development process
3. whether the government's various flagship programmes are reaching the socially excluded
4. State - by-State human development profile
Select the correct answer using the code given below.
A. 1 only
B. 2, 3 and 4
C. 2 and 4 only
D. 1, 3 and 4

CDS-2014
Corporation tax in India is levied on income of a company. Which one of the following does not include
Corporation tax?
(a) Profit from business
(b) Capital gain
(c) Interest on securities
(d) Sale proceed of assets
Whatsapp No 7023213423 http://iasselfstudy.com/ 240

CDS-2014
In India, mergers and acquisition of firms are regulated by
(a) National Manufacturing Competitiveness Council
(b) Competition Commission of India
(c) Security and Exchange Board of India
(d) Department of Industrial Policy and Promotion

CDS-2015
Private investment in Indian agriculture is mostly on labour-saving mechanization.
This could be a response to
(a) rising productivity of agricultural sector
(b) rising inequality in agriculture
(c) rising wages and tighter labour market
(d) debt write-off by the Government

CDS-2015
What is meant by 'Public Good'?
(a) A commodity produced by the Government
(b) A commodity whose benefits are indivisibly spread among the entire community
(c) A Government scheme that benefits the poor households
(d) Any commodity that is very popular among general public

CDS-2016
Which of the following is not a ' Public Good' ?
(a) Electricity
(b) National Defence
(c) Light House
(d) Public Parks

CDS-2016
Which one of the following terms is used in Economics to denote a technique for avoiding a risk by
making a counteracting transaction ?
(a) Dumping
(b) Hedging
(c) Discounting
(d) Deflating

CDS-2016
Which of the following with regard to the term 'bank run' is correct?
(a) The net balance of money a bank has in its chest at the end of the day's business
(b) The ratio of bank's total deposits and total liabilities
(c) A panic situation when the deposit holders start withdrawing cash from the banks
(d) The period in which a bank creates highest credit in the market

CDS-2016
Capital deepening refers to
(a) going for more fixed capital per worker
(b) emphasis on social overhead capital
(c) constant capital-output ratio
(d) increasing capital-output ratio
Whatsapp No 7023213423 http://iasselfstudy.com/ 241

CDS-2016
In India, the term 'hot money' is used to refer to
(a) Currency + Reserves with the RBI
(b) Net GDR
(c) Net Foreign Direct Investment
(d) Foreign Portfolio Investment

CDS-2016
Which of the following is/are credit rating agency/ agencies in India?
(a) CRISIL
(b) CARE
(c) ICRA
(d) All of the above

CAPF-2011
Which one among the following is not an institution of Indian Money Market?
(a) Reserve Bank of India
(b) Bill Brokers
(c) Merchant Bankers
(d) Non-Banking Financial Intermediaries

CAPF-2011
Which one among the following is not a component of fiscal policy?
(a) Taxation policy
(b) Public debt policy
(c) Trade policy
(d) Public expenditure policy

CAPF-2011
Import substitution implies
(a ) importing new items in place of old items of import.
(b) gradual reduction of imported goods to save foreign exchange
(c) increasing domestic supply of goods by imposing import restrictions
(d) replacing import items by domestic production of such items

CAPF-2012
In the context of agricultural revolution in India, which of the following statements is/are correct?
1. White revolution in the field of agricultural development is associated with modernization
of sheep farming.
2. Second green revolution is associated with development of horticulture.
3. Blue revolution in the field of agriculture is associated with improvement in the production of fish
and marine products.
Select the correct answer using the code given below.
(a) 1 only
(b) 3 only
(c) 2 and 3 only
(d) 1, 2 and 3

CAPF-2013
Which of the following statements related to globalization is/are correct?
1. It has resulted in the change of scale in economic activities.
2. End of Cold War facilitated globalization.
3. Flow of money in transnational corporations is discouraged.
Select the correct answer using the code given below.
Whatsapp No 7023213423 http://iasselfstudy.com/ 242

(a) 1 and 2 only


(b) 1 only
(c) 2 and 3 only
(d) 1, 2 and 3

CAPF-2013
Privatization includes
(a) sale of public enterprises to private sector
(b) disinvestment of public enterprise equity
(e) participation of private sector in management in public sector enterprises
(d) All of the above

CAPF-2013
In the current pricing policy, the price of diesel in India consists of
(a) Fuel component + Customs duty + Excise duty +" Sales VAT + Dealer's commission
(b) Fuel component + Excise duty + Sales VAT + Dealer's commission
(c) Fuel component + Customs duty + Sales VAT + Dealer's commission
(d) Fuel component + Customs duty + Excise duty + Dealer's commission

CAPF-2013
Which one among the following sectors in India has the highest share of employment?
(a) Agriculture and allied activities
(b) Manufacturing
(c) Construction
(d) Tertiary sector

CAPF-2014
Which of the following factors affects individual's demand for a commodity ?
1. Price of the commodity
2. Income of the consumer
3. Prices of related goods
Select the correct answer using the code given below :
(a) 1 and 2 only
(b) 2 and 3 only
(c) 1, 2 and 3
(d) 1 only

CISF-LDC-2012
Which one among the following price indices does not indicate net asset value of a stock ?
(a) Residex
(b) Sensex
(c) Nifty
(d) Wholesale Price Index

ES-2013
Mahatma Gandhi National Rural Employment Guarantee Act makes a paradigm shift from
the previous wage employment programmes by :
(a) focusing on all-round development of the rural people
(b) providing wage employment in rural areas a legal binding for the State Governments
(c) providing a statutory guarantee of wage employment
(d) None of the above is correct in this context
Whatsapp No 7023213423 http://iasselfstudy.com/ 243

NDA-2010
Planning in India drew on
(f) New economic programme of Lenin
(g) The Fabian socialism of Sidney and beatrice webb
(h) The British welfare mechanism
(i) New democratic development packages

NDA-2016
Areas which are engines for economic growth supported by quality infrastructure and, complemented
by an attractive fiscal package are known as
(a) Export Processing Zones
(b) Duty Free Tariff Zones
(c) Special Economic Zones
(d) Technology Parks

SCRA-2010
An increase in money supply in the economy will lead to
(a) increase in inflow of FDI
(b) appreciation of domestic currency
(c) increase in prices In the economy
(d) None of the above

SCRA-2011
In India, property tax is levied by
(a) Central Government
(b) State Governments
(c) Local Government
(d) both Central Government and State Governments

IAS Prelims 1999


From the balance sheet of a company, it is possible to
(a) judge the extent of profitability of the company
(b) assess the profitability and size of the company
(c) determine the size and composition of the assets and liabilities of the company
(d) determine the market share, debts and assets of the company

SCRA-2014
The term 'hot money' is used to refer to
(a) Currency + Reserves with the RBI
(b) Net GDR receipts
(c) Net Foreign Direct Investment
(d) Foreign Portfolio Investment

SCRA-2014
Balanced growth implies that
(a) growth of no sector is impeded by inadequate growth in another sector
(b) supply and demand grow at the same pace
(c) growth rates of income and consumption are the same
(d) growth rate of savings is equal to growth rate of investment
Whatsapp No 7023213423 http://iasselfstudy.com/ 244

SCRA-2014
In India, which one among the following taxes/duties now follows a negative list approach?
(a) Customs duty
(b) Excise duty
(e) Service tax
(d) Income tax

SCRA-2015
Gadgil formula was evolved to distribute
(a) taxes between the Union and States
(b) plan assistance among States
(c) amount of loans to needy States
(d) grants among special category States

SCRA-2015
Unbalanced growth strategy keeps in view
(a) linkages between industries
(b) prices across industries
(c) employment across industries
(d) contribution of various industries to GDP

IAS Prelims 2010


With reference to India, consider the following
1. Nationalization of Banks
2. Formation of Regional Rural Banks
3. Adoption of villages by Bank Branches
Which of the above can be considered as steps taken to achieve the "financial inclusion" in India?
(a.) 1 and 2 only
(b.) 2 and 3 only
(c.) 3 only
(d.) 1, 2 and 3

IAS Prelims 2010


In India, the tax proceeds of which one of the following as a percentage of gross tax revenue has
significantly declined in the last five years?
(a.) Service tax
(b.) Personal income tax
(c.) Excise duty
(d.) Corporation tax

IAS Prelims 2010


Consider the following statements :
In India, taxes on transactions in Stock Exchanges and Futures Markets are
1. levied by the Union
2. collected by the State
which of the statements given above is /are correct?
(a.) 1 only
(b.) 2 only
(c.) Both 1 and 2
(d.) Neither 1 nor 2
Whatsapp No 7023213423 http://iasselfstudy.com/ 245

IAS Prelims 2010


In the context of governance, consider the following:
1. Encouraging Foreign Direct Investment inflows
2. Privatization of Higher educational Institutions
3. downsizing of bureaucracy
4. selling/offloading the shares of Public Sector Undertakings
Which of the above can be used as measures to control the fiscal deficit in India?
(a.) 1, 2 and 3
(b.) 2, 3 and 4
(c.) 1, 2 and 4
(d.) 3 and 4 only

IAS Prelims 2010


In India, the interest rate on savings account in all the nationalized commercial banks is fixed by
(a.) Union Ministry of Finance
(b.) Union Finance Commission
(c.) Indian Banks' Association
(d.) None of the above

IAS Prelims 2009


In the context of independent India’s Economy, which one of the following was the earliest event to take
place?
A. Nationalisation of Insurance companies
B. Nationalisation of State Bank of India
C. Enactment of Banking Regulation Act
D. Introduction of First Five Year Plan

IAS Prelims 2006


Which one of the following statements is correct ?
Fiscal Responsibility and Budget Management Act (FRBMA) concerns
(a) fiscal deficit only
(b) revenue deficit only
(c) both fiscal and revenue deficit
(d) neither fiscal deficit nor revenue deficit.

IAS Prelims 2003


Debenture holder of a company are its
(a) Shareholders (b) Creditors (c) Debtors (d) Directors

IAS Prelims 2001


Match List I with List II and select the correct answer using the codes given below the Lists :
List I List II
(Publisher) (Publication)
I. Ministry of Industry (A) Report on Currency and Finance
II. Central Statistical Organisation (B) Economic Survey
III. Reserve Bank of India (C) Wholesale Price Index
IV. Ministry of Finance (D) National Accounts Statistics
Codes:
(a) ID, IIC, IIIB, IVA
(b) IC, IID, IIIA, IVB
(c) ID, IIC, IIIA, IVB
(d) IC, IID, IIIB, IVA
Whatsapp No 7023213423 http://iasselfstudy.com/ 246

IAS Prelims 2001


Consider the following:
I. Market borrowing
II. Treasury bills
III. Special securities issued to RBI
Which of these is/are component(s) of internal debt ?
(a) I only
(b) I and II
(c) II only
(d) I, II and III

IAS Prelims 2000


Economic liberalisation in India started with
(a) substantial changes in industrial licensing policy
(b) the convertibility of Indian rupee
(c) doing away with procedural formalities for foreign direct investment
(d) significant reduction in tax rates

IAS Prelims 2000


The growth rate of per capita income at current prices is higher than that of per capita income at constant
prices, because the latter takes into account the rate of
(a) growth of population (b) increase in price level
(c) growth of money supply (d) increase in the wage rate

IAS Prelims 2000


Match List I with List II and select the correct answer using the codes given below the Lists :
List I List II
I. Boom (A) Business activity at high level with increasing income, output and employment
at macro level
II. Recession (B) Gradual fall of income, output and employment with business activity in a low
gear
III. Depression (C) Unprecedented level of under employment and unemployment, drastic fall in
income, output and employment
IV. Recovery (D) Steady rise in the general level of prices, income, output and employment
Codes:
(a) IA, IIB, IIIC, IVD
(b) IA, IIB, IIID, IVC
(c) IB, IIA, IIID, IVC
(d) IB, IIA, IIIC, IVD

IAS Prelims 1991


The main source of financing the implementation of development programmes is
(a) Internal resources from taxes and other measures
(b) External finance as borrowings/loans from foreign governments
(c) External finance as loans from international finance agencies
(d) External finance as loans from foreign governments and international finance agencies

IAS Prelims 1993


Which one of the following modes of privatisation is the most comprehensive and complete?
(a) Introduction of private capital in public sector
(b) Contracting out management of public enterprises to the private sector
(c) Transferring ownership and management to the workers
(d) Transferring ownership and management to the private sector
Whatsapp No 7023213423 http://iasselfstudy.com/ 247

IAS Prelims 1994


To know whether the rich are getting richer and the poor getting poorer, it is necessary to compare
(a) the wholesale price index over different periods of time for different regions
(b) the distribution of income of an identical set of income recipients in different periods of time
(c) the distribution of income of different sets of income recipients at a point of time
(d) the availability of food grains among two sets of people, one rich and the other poor, over different
periods of time

IAS Prelims 1994


Which one of the following governmental steps has proved relatively effective in controlling the double
digit rate of inflation in the Indian economy during recent years ?
(a) Enhanced rate of production of all consumer goods
(b) Streamlined public distribution system
(c) Pursuing an export oriented strategy
(d) Containing budgetary deficits and unproductive expenditure

IAS Prelims 1994


The principal reason why national, economic planning is still being pursued in spite of embracing a market
economy since 1991 is that
(a) it is a Constitutional requirement
(b) the vast quantity of capital already deployed in the public sector needs to be nurtured
(c) Five Year Plans can continue to provide a long term perspective to the economy in market friendly
fashions
(d) the market economy is mainly confined to industry and commerce and central planning in agriculture
is necessary

IAS Prelims 1995


Which of the following pairs are correctly matched ?
I. Increase in foreign exchange reserves – Monetary expansion
II. Low import growth rate in India – Recession in Indian Industry
III. Euro issues – Shares held by Indian companies in European countries
IV. Portfolio investment – Foreign institutional investors
Select the correct answer by using the following codes :
Codes :
(a) I, II and IV
(b) III and IV
(c) I, II and III
(d) I, II, III and IV

IAS Prelims 1995


The main reason for low growth rate in India, in spite of high rate of savings and capital formation is
(a) high birth rate
(b) low level of foreign aid
(c) low capital / output ratio
(d) high capital / output ratio

IAS Prelims 1995


One of the reasons for India’s occupational structure remaining more or less the same over the years
has been that
(a) investment pattern has been directed towards capital intensive industries
(b) productivity in agriculture has been high enough to induce people to stay with agriculture
(c) ceilings on land holdings have enabled more people to own land and hence their preference to stay
with agriculture
(d) people are largely unaware of the significance of transition from agriculture to industry for
economic development
Whatsapp No 7023213423 http://iasselfstudy.com/ 248

IAS Prelims 1996


In India, rural incomes are generally lower than the urban incomes. Which of the following reasons
account for this ?
I. A large number of farmers are illiterate and know little about scientific agriculture.
II. Prices of primary products are lower than those of manufactured products.
III. Investment in agriculture has been low when compared to investment in industry.
Select the correct answer by using the codes given below :
Codes :
(a) I, II and III
(b) I and II
(c) I and III
(d) II and III

IAS Prelims 1996


Consider the following statements:
Most international agencies which fund development programmes in India on intergovernmental bilateral
agreements, mainly provide
I. Technical assistance.
II. Soft loans which are required to be paid back with interest.
III. Grants, not required to be paid back.
IV. Food assistance to alleviate poverty.
Of these statements
(a) II and IV and correct
(b) I, II and III are correct
(c) I, II and IV are correct
(d) III and IV are correct

IAS Prelims 1996


The emerging trading blocs in the world, such as NAFTA, ASEAN and the like, are expected to
(a) act as constrictions in free trade across the world
(b) promote free trade on the lines laid down by the WTO
(c) permit transfer of technology between member countries
(d) promote trade in agricultural commodities between countries of the North and South

IAS Prelims 1996


Assertion (A): An important policy instrument of economic liberalization is reduction in import duties on
capital goods.
Reason (R): Reduction in import duties would help the local entrepreneurs to improve technology to face
the global markets.
In the context of the above two statements, which one of the following is correct ?
(a) Both A and R are true and R is the correct explanation of A
(b) Both A and R are true and R is not a correct explanation of A
(c) A is true but R is false
(d) A is false but R is true

IAS Prelims 1996


Assertion (A): Though India’s national income has gone up several fold since 1947, there has been no
marked improvement in the per capita income level.
Reason (R): Sizeable proportion of the population of India is still living below the poverty line.
In the context of the above two statements which one of the following is correct ?
(a) Both A and R are true and R is the correct explanation of A
(b) Both A and R are true but R is not a correct explanation of A
(c) A is true but R is false
(d) A is false but R is true
Whatsapp No 7023213423 http://iasselfstudy.com/ 249

IAS Prelims 1998


A consumer is said to be in equilibrium, if
(a) he is able to fulfil his need with a given level of income
(b) he is able to live in full comforts with a given level of income
(c) he can fulfil his needs without consumption of certain items
(d) he is able to locate new sources of income

IAS Prelims 1998


The supply side economics lays greater emphasis on the point of view of
(a) producer
(b) global economy
(c) consumer
(d) middleman

IAS Prelims 1999


Consider the following statements :
Industrial development in India, to an extent, is constrained by
I. lack of adequate entrepreneurship and leadership in business.
II. lack of savings to invest.
III. lack of technology, skills and infrastructure.
IV. limited purchasing power among the larger masses.
Which of the above statements are correct ?
(a) I, II and III
(b) I, III and IV
(c) II, III and IV
(d) I, II and IV

IAS Prelims 1999


Consider the following statements :
Small scale industries are, in most cases, not as efficient and competitive as the large scale ones. Yet
the Government provides preferential treatment and reservations in a range of products to the small
firms because small scale industries
I. provide higher employment on a per unit capital deployment basis.
II. promote a regional dispersion of industries and economic activities.
III. have performed better in export of manufactured products than the large scale ones.
IV. provide jobs to low skill workers, who otherwise may not find employment avenues elsewhere.
Which of the above statements are correct ?
(a) I and IV
(b) I and II
(c) II and III
(d) III and IV

IAS Prelims 1999


The farmers are provided credit from a number of sources for their short and long term needs. The
main sources of credit to the farmers include
(a) the Primary Agricultural Cooperative Societies, commercial banks, RRBs and private money lenders
(b) the NABARD, RBI, commercial banks and private moneylenders
(c) the District Central Cooperative Banks (DCCB), the lead banks, IRDP and JRY
(d) the Large Scale Multipurpose Adivasis Programme, DCCB, IFFCO and commercial banks
Whatsapp No 7023213423 http://iasselfstudy.com/ 250

IAS Prelims 2016


There has been a persistent deficit budget year after year. Which action/actions of the following can be
taken by the Government to reduce the deficit?
1. Reducing revenue expenditure
2. Introducing new welfare schemes
3. Rationalizing subsidies
4. Reducing import duty
Select the correct answer using the code given below.
(a) 1 only (b) 2 and 3 only (c) 1 and 3 only (d) 1, 2, 3 and 4

IAS Prelims 2015


There has been a persistent deficit budget year after year. Which of the following actions can be taken
by the government to reduce the deficit?
1. Reducing revenue expenditure
2. Introducing new welfare schemes
3. Rationalizing subsidies
4. Expanding industries
Select the correct answer using the code given below.
(a) 1 and 3 only
(b) 2 and 3 only
(c) 1 only
(d) 1,2,3 and 4

IAS Prelims 2016


Which of the following best describes the term ‘import cover’, sometimes seen in the news?
(a) It is the ratio of value of imports to the Gross Domestic Product of a country
(b) It is the total value of imports of a country in a year
(c) It is the ratio between the value of exports and that of imports between two countries
(d) It is the number of months of imports that could be paid for by a country’s international reserves

IAS Prelims 2016


Which of the following is/are the indicator/indicators used by IFPRI to compute the Global Hunger
Index Report?
1. Undernourishment
2. Child stunting
3. Child mortality
Select the correct answer using the code given below.
(a) 1 only
(b) 2 and 3 only
(c) 1 , 2 and 3
(d) 1 and 3 only

IAS Prelims 2015


‘Basel III Accord’ or simply ‘Basel III’, often seen in the news, seeks to
(a) develop national strategies for the conservation and sustainable use of biological diversity
(b) improve banking sector’s ability to deal with financial and economic stress and improve risk
management
(c) reduce the greenhouse gas emissions but places a heavier burden on developed countries
(d) transfer technology from developed Countries to poor countries to enable them to replace the use
of chlorofluorocarbons in refrigeration with harmless chemicals
Whatsapp No 7023213423 http://iasselfstudy.com/ 251

IAS Prelims 2015


The problem of international liquidity is related to the non-availability of
(a) goods and services
(b) gold and silver
(c) dollars and other hard currencies
(d) exportable surplus

IAS Prelims 2015


A decrease in tax to GDP ratio of a country indicates which of the following?
1. Slowing economic growth rates
2. Less equitable distribution of national income
Select the correct answer using the code given below.
a) 1 only
b) 2 only
c) Both 1 and 2
d) Neither 1 nor 2

IAS Prelims 2015


The terms ‘Agreement on Agriculture’, ‘Agreement on the Application of Sanitary and Phytosanitary
Measures’ and Peace Clause’ appear in the news frequently in the context of the affairs of the:
a) Food and Agriculture Organization
b) United Nations Framework Conference on Climate Change
c) World Trade Organization
d) United Nations Environment Programme

IAS Prelims 2014


If the interest rate is decreased in an economy, it will
A. decrease the consumption expenditure in the economy
B. increase the tax collection of the Government
C. increase the investment expenditure in the economy
D. increase the total savings in the economy

IAS Prelims 2013


Which of the following grants/ grant direct credit assistance to rural households?
1. Regional Rural Banks
2.National Bank for Agriculture and Rural Development
3.Land Development Banks
Select the correct answer using the codes given below:
a. 1 and 2 only
b. 2 only
c. 1 and 3 only
d. 1, 2 and 3

IAS Prelims 2013


Supply of money remaining the same when there is an increase in demand for money, there will be
a. a fall in the level of prices
b. an increase in the rate of interest
c. a decrease in the rate of interest
d. an increase in the level of income and employment
Whatsapp No 7023213423 http://iasselfstudy.com/ 252

IAS Prelims 2013


Economic growth in country X will necessarily have to occur if
a. there is technical progress in the world economy
b. there is population growth in X
c. there is capital formation in X
d. the volume of trade grows in the world economy

IAS Prelims 2012


Under which of the following circumstances may ‘capital gains ‘arise?
1 when there is an increase in the sales of a product
2 when there is a natural increase in the value of the property owned.
3 when you purchase a painting and there is a growth in its value due to increase in its popularity.
Select the correct answer using the codes given below:
a) 1 only
b) 2 and 3 only
c) 2 only
d) 1, 2 and 3

IAS Prelims 2012


Which of the following measures would result in an increase in the money supply in the economy?
1 Purchase of govt securities from the public by the central bank
2 Deposit of currency in commercial banks by the public
3 borrowing by the govt. from the central bank
4 Sale of govt. securities to the public by the central bank
Select the correct answer using the codes given below:
a )1
b )2 and 4 only
c )1 and 3
d )2, 3 and 4

IAS Prelims 1998 & 2012


Consider the following statement:
The price of any currency in international market is decided by the
1. World Bank
2. Demand for goods/services provided by the country concerned
3. Stability of the government of the concerned country
4. Economic potential of the country in question.
Which of the statements given above are correct?
a) 1, 2 ,3 and 4
b) 2 and 3 only
c) 3 and 4 only
d) 1 and 4 only

IAS Prelims 2012


Which of the following can be said to be essentially the parts of ‘Inclusive Governance’?
1. Permitting the Non-Banking Financial Companies to do banking
2. Establishing effective District Planning Committees in all the districts
3.Increasing the government spending on public health
4. Strengthening the Mid-day Meal Scheme
Select the correct answer using the codes given below:
a. 1 and 2 only
b. 3 and 4 only
c. 2,3 and 4 only
d. 1, 2, 3 and 4
Whatsapp No 7023213423 http://iasselfstudy.com/ 253

IAS Prelims 2011


In terms of economy, the visit by foreign nationals to witness the XIX common wealth games in India
amounted to?
(a) Export.
(b) Import.
(c) Production.
(d) Consumption.

IAS Prelims 2011


Which one of the following statements appropriately describes the “fiscal stimulus” ?
(a) It is a massive investment by the government in manufacturing sector to ensure the supply of
goods to meet the demand surge caused by rapid economic growth.
(b) It is an intense affirmative action of the government to boost economic activity in the country.
(c) It is government’s intensive action on financial institutions to ensure disbursement of loans to
agriculture and allied sectors to promote greater food production and contain food inflation.
(d) It is an extreme affirmative action by the government to pursue its policy of financial inclusion

IAS Prelims 2011


A rapid increase in the rate of inflation is sometimes attributed to the “ base effect”. What is “base
effect” ?
(a) It is the impact of drastic deficiency in supply due to failure of crops.
(b) It is the impact of the surge in demand due to rapid economic growth.
(c) It is the impact of the price levels of previous year on the calculation of inflation rate.
(d) None of the statements (a), (b) and (c) given above is correct in this context.

IAS Prelims 2011


In India, which of the following have the highest share in the disbursement of credit to agriculture and
allied activities ?
(a) Commercial banks.
(b) Cooperative banks.
(c) Regional rural banks.
(d) Microfinance institutions.

IAS Prelims 2011


Which of the following can aid in furthering the government’s objective of inclusive growth ?
1. Promoting self-help groups.
2. Promoting micro, Small and medium enterprises.
3. Implementing the right to education act.
Select the correct answer using the codes given below :
(a) 1 only.
(b) 1 and 2 only.
(c) 2 and 3 only.
(d) 1, 2 and 3

IAS Prelims 2011


Economic growth is usually coupled with ?
(a) Deflation.
(b) Inflation.
(c) Stagflation.
(d) Hyperinflation.
Whatsapp No 7023213423 http://iasselfstudy.com/ 254

CDS-2014
Which of the following statements about India's unorganized sector are true?
I. Labour is more in number than that in the organized sector.
2. Job security and work regulation are better in unorganized sector.
3. They are usually not organized into trade unions.
4. Workers are usually employed for a limited number of days.
Select the correct answer using the code given below.
(a) I, 2 and 4
(b) I, 3 and 4
(c) 3 and 4 only
(d) 1 and 3 only

CDS-2013
Which one among the following is a fixed cost to a manufacturing firm in the short run ?
1. insurance on buildings
2. overtime payment to workers
3. cost of energy
4. cost of raw materials

CDS-2013
The value of money varies:
(a) directly with the interest rate
(b) directly with the price level
(c) directly with the volume of employment
(d) inversely with the price level

CDS-2013
Corporation tax is imposed by :
(a) State Government
(b) Central Government
(c) Local Government
(d) State as well as Central Government

CDS-2013
In India, the price of petroleum products has been deregulated mainly to :
(a) reduce the burden of subsidies given to the oil companies
(b) discourage the exploration of oil reserves in the country
(c) discourage the demand for private vehicles
(d) curb the use of black money in the Economy

CDS-2012
Which of the following statements is / are correct?
l. High growth will lead to inflation.
2. High growth will lead to deflation.
Select the correct answer using the code given below :
(a) 1 only
(b) 2 only
(c) Both 1 and 2
(d) Neither 1 nor 2
Whatsapp No 7023213423 http://iasselfstudy.com/ 255

CDS-2012
Which of the following measures should be taken when an economy is going through inflationary
pressures?
1. The direct taxes should be increased.
2. The interest rate should be reduced.
3. The public spending should be increased.
Select the correct answer using the code given below:
Code:
(a) 1 only
(b) 2 only
(c) 2 and 3
(d) 1 and 2

CDS-2012
Fiscal policy in India is formulated by
(a) the Reserve Bank of India
(b) the Planning Commission
© the Finance Ministry
(d) the Securities and Exchange Board of India

Note: Fiscal policy is implemented through the Budget (i.e. Finance ministry).

CDS-2014
A market in which there are large numbers of sellers of a particular product, but each seller sells
somewhat differentiated but close products is termed as
(a) Perfect competition
(b) Monopoly
(c) Monopolistic competition
(d) Oligopoly

CDS-2014
Which of the following statements are correct ?
1. When marginal revenue is positive, total revenue increases with increase in output.
2. When marginal revenue is zero, total revenue is maximum.
3. When marginal revenue becomes negative, total revenue falls with increase in output. "
Select the correct answer using the code given below:
(a) 1 and 2 only
(b) 2 and 3 only
(c) 1 and 3 only
(d) 1, 2 and 3

CDS-2014
Which one among the following is not a source of tax revenue for the Central Government in India?
(a) Income tax
(b) Customs duties
(c) Service tax
(d) Motor Vehicle tax

CDS -2011
Brent index is associated with
(a) crude oil prices (b) copper future prices (c) gold future prices (d) shipping rate index
Whatsapp No 7023213423 http://iasselfstudy.com/ 256

ES-2013
Brent index is associated with :
(a) Crude oil prices
(b) Copper future prices
© Eco-friendly status of processed foods
(d) Energy efficiency status of electrical goods

CDS-2011
According to Goldman Sachs' review of emerging economies, by 2050 which one of the following would
be the order of the largest economies in the world?
(a) China-USA-India-Brazil-Mexico
(b) USA-China-India-Brazil-Mexico
(c) China-USA-Brazil-India-Mexico
(d) USA-Mexico-China-India-Brazil

CDS-2011
Consider the following statements about Sinking Fund:
1. It is a method of repayment of public debt.
2. It is created by the government out of budgetary revenues every year.
Which of the statements given above is/are correct ?
(a) 1 only
(b) 2 only
(c) Both 1 and 2
(d) Neither 1 nor 2

CDS-2015
Demand for a commodity refers to
(a) Desire for that commodity
(b) Need for that commodity
(c) Quantity demanded of that commodity
(d) Quantity demanded at certain price during any particular period of time

CDS-2015
Rise in the price of a commodity means
(a)rise in the value of currency only
(b) fall in the value of currency only
(c) rise in the value of commodity only
(d) fall in the value of currency and rise in the value of commodity

IAS Prelims 2010


In the context of Indian economy, consider the following pairs:
Term Most appropriate description
1. Melt down .. . .. . .. . . Fall in stock prices
2. Recession .................Fall in growth rate
3. Slow down ............... Fall in GDP
Which of the pairs given above is/are correctly matched?
(a.) 1 only
(b.) 2 and 3 only
(c.) 1 and 3 only
(d.) 1, 2 and 3
Whatsapp No 7023213423 http://iasselfstudy.com/ 257

ES 2017
Which one of the following Committees was set up by the Government of India in September 2014 to
restructure the Railways and to suggest ways for resource mobilization ?
(a) Bibek Debroy Committee
(b) C. Rangarajan Committee
(c) Parthasarathi Shome Committee
(d) Sundar Committee

CDS-2011
Along with goods manufactured using child labour, some of the developed countries have started to
boycott goods manufactured in developing countries using 'sweat labour'. 'Sweat labour' implies goods
produced by
(a) female labourers at a very low wage
(b) labourers working in inhuman/unhealthy working conditions
(C) labourers working for more than eight hours a day without any break
(d) labourers where there is a wage discrimination between male and female labourers

IAS Prelims 2002


A country is said to be in debt trap if
(a) it has to abide by the conditionalities imposed by the International Monetary Fund
(b) it has to borrow to make interest payments to on outstanding loans
(c) it has been refused loans or aid by creditors abroad
(d) the World Bank charges a very high rate of interest on outstanding as well as new loans

Potrebbero piacerti anche